*NURSING > TEST BANK > Advanced Health Assessment and Differential Diagnosis Essentials for Clinical Practice 1st Edition M (All)

Advanced Health Assessment and Differential Diagnosis Essentials for Clinical Practice 1st Edition Myrick Test Bank

Document Content and Description Below

Advanced Health Assessment and Differential Diagnosis Essentials for Clinical Practice 1st Edition Myrick Test Bank Chapter 1. Health History, The Patient Interview, And Motivational Interviewing MUL... TIPLE CHOICE 1. The nurse is preparing to conduct a health history. Which of these statements best describes the purpose of a health history? a . To provide an opportunity for interaction between the patient and the nurse b . To provide a form for obtaining the patients biographic information c . To document the normal and abnormal findings of a physical assessment d . To provide a database of subjective information about the patients past and current health ANS: D The purpose of the health history is to collect subjective datawhat the person says about him or herself. The other options are not correct. DIF: Cognitive Level: Understanding (Comprehension) REF: dm. 49 MSC: Client Needs: Safe and Effective Care Environment: Management of Care 2. When the nurse is evaluating the reliability of a patients responses, which of these statements would be correct? The patient: a . Has a history of drug abuse and therefore is not reliable. b . Provided consistent information and therefore is reliable. c . Smiled throughout interview and therefore is assumed reliable. d . Would not answer questions concerning stress and therefore is not reliable. ANS: B A reliable person always gives the same answers, even when questions are rephrased or are repeated later in the interview. The other statements are not correct. DIF: Cognitive Level: Applying (Application) REF: dm. 49 MSC: Client Needs: Safe and Effective Care Environment: Management of Care 3. A 59-year-old patient tells the nurse that he has ulcerative colitis. He has been having black stools for the last 24 hours. How would the nurse best document his reason for seeking care? a . J.M. is a 59-year-old man seeking treatment for ulcerative colitis. b . J.M. came into the clinic complaining of having black stools for the past 24 hours. c . J.M. is a 59-year-old man who states that he has ulcerative colitis and wants it checked. d . J.M. is a 59-year-old man who states that he has been having black stools for the past 24 hours. ANS: D The reason for seeking care is a brief spontaneous statement in the persons own words that describes the reason for the visit. It states one (possibly two) signs or symptoms and their duration. It is enclosed in quotation marks to indicate the persons exact words. DIF: Cognitive Level: Applying (Application) REF: dm. 50 MSC: Client Needs: Safe and Effective Care Environment: Management of Care 4. A patient tells the nurse that she has had abdominal pain for the past week. What would be the nurses best response? a . Can you point to where it hurts? b . Well talk more about that later in the interview. c . What have you had to eat in the last 24 hours? d . Have you ever had any surgeries on your abdomen? ANS: A A final summary of any symptom the person has should include, along with seven other critical characteristics, Location: specific. The person is asked to point to the location. DIF: Cognitive Level: Applying (Application) REF: dm. 50 MSC: Client Needs: Safe and Effective Care Environment: Management of Care 5. A 29-year-old woman tells the nurse that she has excruciating pain in her back. Which would be the nurses appropriate response to the womans statement? a . How does your family react to your pain? b . The pain must be terrible. You probably pinched a nerve. c . Ive had back pain myself, and it can be excruciating. d . How would you say the pain affects your ability to do your daily activities? ANS: D The symptom of pain is difficult to quantify because of individual interpretation. With pain, adjectives should be avoided and the patient should be asked how the pain affects his or her daily activities. The other responses are not appropriate. DIF: Cognitive Level: Applying (Application) REF: dm. 50 MSC: Client Needs: Safe and Effective Care Environment: Management of Care 6. In recording the childhood illnesses of a patient who denies having had any, which note by the nurse would be most accurate? a . Patient denies usual childhood illnesses. b . Patient states he was a very healthy child. c . Patient states his sister had measles, but he didnt. d . Patient denies measles, mumps, rubella, chickenpox, pertussis, and strep throat. ANS: D Childhood illnesses include measles, mumps, rubella, chickenpox, pertussis, and strep throat. Avoid recording usual childhood illnesses because an illness common in the persons childhood may be unusual today (e.g., measles). DIF: Cognitive Level: Remembering (Knowledge) REF: dm. 51 MSC: Client Needs: Safe and Effective Care Environment: Management of Care 7. A female patient tells the nurse that she has had six pregnancies, with four live births at term and two spontaneous abortions. Her four children are still living. How would the nurse record this information? a P-6, B-4, (S)Ab-2 . b . Grav 6, Term 4, (S)Ab-2, Living 4 c . Patient has had four living babies. d . Patient has been pregnant six times. ANS: B Obstetric history includes the number of pregnancies (gravidity), number of deliveries in which the fetus reached term (term), number of preterm pregnancies (preterm), number of incomplete pregnancies (abortions), and number of children living (living). This is recorded: Grav Term Preterm Ab Living . For any incomplete pregnancies, the duration is recorded and whether the pregnancy resulted in a spontaneous (S) or an induced (I) abortion. DIF: Cognitive Level: Applying (Application) REF: dm. 51 MSC: Client Needs: Safe and Effective Care Environment: Management of Care 8. A patient tells the nurse that he is allergic to penicillin. What would be the nurses best response to this information? a Are you allergic to any other drugs? . b How often have you received penicillin? . c Ill write your allergy on your chart so you wont receive any penicillin. . d Describe what happens to you when you take penicillin. . ANS: D Note both the allergen (medication, food, or contact agent, such as fabric or environmental agent) and the reaction (rash, itching, runny nose, watery eyes, or difficulty breathing). With a drug, this symptom should not be a side effect but a true allergic reaction. DIF: Cognitive Level: Understanding (Comprehension) REF: dm. 52 MSC: Client Needs: Safe and Effective Care Environment: Management of Care 9. The nurse is taking a family history. Important diseases or problems about which the patient should be specifically asked include: a Emphysema. . b . Head trauma. c . Mental illness. d . Fractured bones. ANS: C Questions concerning any family history of heart disease, high blood pressure, stroke, diabetes, obesity, blood disorders, breast and ovarian cancers, colon cancer, sickle cell anemia, arthritis, allergies, alcohol or drug addiction, mental illness, suicide, seizure disorder, kidney disease, and tuberculosis should be asked. DIF: Cognitive Level: Remembering (Knowledge) REF: dm. 53-54 MSC: Client Needs: Safe and Effective Care Environment: Management of Care 10. The review of systems provides the nurse with: a Physical findings related to each system. . b Information regarding health promotion practices. . c An opportunity to teach the patient medical terms. . d Information necessary for the nurse to diagnose the patients medical problem. . ANS: B The purposes of the review of systems are to: (1) evaluate the past and current health state of each body system, (2) double check facts in case any significant data were omitted in the present illness section, and (3) evaluate health promotion practices. DIF: Cognitive Level: Remembering (Knowledge) REF: dm. 54 MSC: Client Needs: Safe and Effective Care Environment: Management of Care 11. Which of these statements represents subjective data the nurse obtained from the patient regarding the patients skin? a Skin appears dry. . b No lesions are obvious. . c Patient denies any color change. . d Lesion is noted on the lateral aspect of the right arm. . ANS: C The history should be limited to patient statements or subjective datafactors that the person says were or were not present. DIF: Cognitive Level: Understanding (Comprehension) REF: dm. 54 MSC: Client Needs: Safe and Effective Care Environment: Management of Care 12. The nurse is obtaining a history from a 30-year-old male patient and is concerned about health promotion activities. Which of these questions would be appropriate to use to assess health promotion activities for this patient? a Do you perform testicular self-examinations? . b Have you ever noticed any pain in your testicles? . c Have you had any problems with passing urine? . d Do you have any history of sexually transmitted diseases? . ANS: A Health promotion for a man would include the performance of testicular self-examinations. The other questions are asking about possible disease or illness issues. DIF: Cognitive Level: Understanding (Comprehension) REF: dm. 56 MSC: Client Needs: Safe and Effective Care Environment: Management of Care 13. Which of these responses might the nurse expect during a functional assessment of a patient whose leg is in a cast? a . I broke my right leg in a car accident 2 weeks ago. b . The pain is decreasing, but I still need to take acetaminophen. c . I check the color of my toes every evening just like I was taught. d Im able to transfer myself from the wheelchair to the bed without help. . ANS: D Functional assessment measures a persons self-care ability in the areas of general physical health or absence of illness. The other statements concern health or illness issues. DIF: Cognitive Level: Applying (Application) REF: dm. 56 MSC: Client Needs: Safe and Effective Care Environment: Management of Care 14. In response to a question about stress, a 39-year-old woman tells the nurse that her husband and mother both died in the past year. Which response by the nurse is most appropriate? a . This has been a difficult year for you. b . I dont know how anyone could handle that much stress in 1 year! c . What did you do to cope with the loss of both your husband and mother? d . That is a lot of stress; now lets go on to the next section of your history. ANS: C Questions about coping and stress management include questions regarding the kinds of stresses in ones life, especially in the last year, any changes in lifestyle or any current stress, methods tried to relieve stress, and whether these methods have been helpful. DIF: Cognitive Level: Applying (Application) REF: dm. 57 MSC: Client Needs: Safe and Effective Care Environment: Management of Care 15. In response to a question regarding the use of alcohol, a patient asks the nurse why the nurse needs to know. What is the reason for needing this information? a . This information is necessary to determine the patients reliability. b . Alcohol can interact with all medications and can make some diseases worse. c . The nurse needs to be able to teach the patient about the dangers of alcohol use. d . This information is not necessary unless a drinking problem is obvious. ANS: B Alcohol adversely interacts with all medications and is a factor in many social problems such as child or sexual abuse, automobile accidents, and assaults; alcohol also contributes to many illnesses and disease processes. Therefore, assessing for signs of hazardous alcohol use is important. The other options are not correct. DIF: Cognitive Level: Understanding (Comprehension) REF: dm. 58 MSC: Client Needs: Safe and Effective Care Environment: Management of Care 16. The mother of a 16-month-old toddler tells the nurse that her daughter has an earache. What would be an appropriate response? a . Maybe she is just teething. b . I will check her ear for an ear infection. c . Are you sure she is really having pain? d . Describe what she is doing to indicate she is having pain. ANS: D With a very young child, the parent is asked, How do you know the child is in pain? A young child pulling at his or her ears should alert parents to the childs ear pain. Statements about teething and questioning whether the child is really having pain do not explore the symptoms, which should be done before a physical examination. 17. During a visit to the clinic, a patient states, The doctor just told me he thought I ought to stop smoking. He doesnt understand how hard Ive tried. I just dont know the best way to do it. What should I do? The nurses most appropriate response in this case would be: a . Id quit if I were you. The doctor really knows what he is talking about. b . Would you like some information about the different ways a person can quit smoking? c . Stopping your dependence on cigarettes can be very difficult. I understand how you feel. d . Why are you confused? Didnt the doctor give you the information about the smoking cessation program we offer? ANS: B Clarification should be used when the persons word choice is ambiguous or confusing. Clarification is also used to summarize the persons words or to simplify the words to make them clearer; the nurse should then ask if he or she is on the right track. The other responses give unwanted advice or do not offer a helpful response. DIF: Cognitive Level: Applying (Application) REF: dm. 33 MSC: Client Needs: Psychosocial Integrity 18. As the nurse enters a patients room, the nurse finds her crying. The patient states that she has just found out that the lump in her breast is cancer and says, Im so afraid of, um, you know. The nurses most therapeutic response would be to say in a gentle manner: a . Youre afraid you might lose your breast? b . No, Im not sure what you are talking about. c . Ill wait here until you get yourself under control, and then we can talk. d . I can see that you are very upset. Perhaps we should discuss this later. ANS: A Reflection echoes the patients words, repeating part of what the person has just said. Reflection can also help express the feelings behind a persons words. DIF: Cognitive Level: Applying (Application) REF: dm. 33 MSC: Client Needs: Psychosocial Integrity 19. A nurse is taking complete health histories on all of the patients attending a wellness workshop. On the history form, one of the written questions asks, You dont smoke, drink, or take drugs, do you? This question is an example of: a . Talking too much. b . Using confrontation. c . Using biased or leading questions. d . Using blunt language to deal with distasteful topics. ANS: C This question is an example of using leading or biased questions. Asking, You dont smoke, do you? implies that one answer is better than another. If the person wants to please someone, then he or she is either forced to answer in a way that corresponds to his or her implied values or is made to feel guilty when admitting the other answer. DIF: Cognitive Level: Understanding (Comprehension) REF: dm. 34 MSC: Client Needs: Psychosocial Integrity 20. When observing a patients verbal and nonverbal communication, the nurse notices a discrepancy. Which statement is true regarding this situation? The nurse should: a . Ask someone who knows the patient well to help interpret this discrepancy. b . Focus on the patients verbal message, and try to ignore the nonverbal behaviors. c . Try to integrate the verbal and nonverbal messages and then interpret them as an average. d . Focus on the patients nonverbal behaviors, because these are often more reflective of a patients true feelings. ANS: D When nonverbal and verbal messages are congruent, the verbal message is reinforced. When they are incongruent, the nonverbal message tends to be the true one because it is under less conscious control. Thus studying the nonverbal messages of the patients and examiners and understanding their meanings are important. The other statements are not true. DIF: Cognitive Level: Applying (Application) REF: dm. 28 MSC: Client Needs: Psychosocial Integrity 21. During an interview, a parent of a hospitalized child is sitting in an open position. As the interviewer begins to discuss his sons treatment, however, he suddenly crosses his arms against his chest and crosses his legs. This changed posture would suggest that the parent is: a . Simply changing positions. b . More comfortable in this position. c . Tired and needs a break from the interview. d . Uncomfortable talking about his sons treatment. ANS: D The persons position is noted. An open position with the extension of large muscle groups shows relaxation, physical comfort, and a willingness to share information. A closed position with the arms and legs crossed tends to look defensive and anxious. Any change in posture should be noted. If a person in a relaxed position suddenly tenses, then this change in posture suggests possible discomfort with the new topic. DIF: Cognitive Level: Analyzing (Analysis) REF: dm. 35 MSC: Client Needs: Psychosocial Integrity 22. A mother brings her 28-month-old daughter into the clinic for a well-child visit. At the beginning of the visit, the nurse focuses attention away from the toddler, but as the interview progresses, the toddler begins to warm up and is smiling shyly at the nurse. The nurse will be most successful in interacting with the toddler if which is done next? a . Tickle the toddler, and get her to laugh. b . Stoop down to her level, and ask her about the toy she is holding. c . Continue to ignore her until it is time for the physical examination. d . Ask the mother to leave during the examination of the toddler, because toddlers often fuss less if their parent is not in view. ANS: B Although most of the communication is with the parent, the nurse should not completely ignore the child. Making contact will help ease the toddler later during the physical examination. The nurse should begin by asking about the toys the child is playing with or about a special doll or teddy bear brought from home. Does your doll have a name? or What can your truck do? Stoop down to meet the child at his or her eye level. DIF: Cognitive Level: Applying (Application) REF: dm. 37 MSC: Client Needs: Psychosocial Integrity 23. During an examination of a 3-year-old child, the nurse will need to take her blood pressure. What might the nurse do to try to gain the childs full cooperation? a . Tell the child that the blood pressure cuff is going to give her arm a big hug. b . Tell the child that the blood pressure cuff is asleep and cannot wake up. c . Give the blood pressure cuff a name and refer to it by this name during the assessment. d . Tell the child that by using the blood pressure cuff, we can see how strong her muscles are. ANS: D Take the time to give a short, simple explanation with a concrete explanation for any unfamiliar equipment that will be used on the child. Preschoolers are animistic; they imagine inanimate objects can come alive and have human characteristics. Thus a blood pressure cuff can wake up and bite or pinch. DIF: Cognitive Level: Applying (Application) REF: dm. 38-39 MSC: Client Needs: Psychosocial Integrity 24. A 16-year-old boy has just been admitted to the unit for overnight observation after being in an automobile accident. What is the nurses best approach to communicating with him? a . Use periods of silence to communicate respect for him. b . Be totally honest with him, even if the information is unpleasant. c . Tell him that everything that is discussed will be kept totally confidential. d . Use slang language when possible to help him open up. ANS: B Successful communication with an adolescent is possible and can be rewarding. The guidelines are simple. The first consideration is ones attitude, which must be one of respect. Second, communication must be totally honest. An adolescents intuition is highly tuned and can detect phoniness or the withholding of information. Always tell him or her the truth. DIF: Cognitive Level: Applying (Application) REF: dm. 39 MSC: Client Needs: Psychosocial Integrity 25. A 75-year-old woman is at the office for a preoperative interview. The nurse is aware that the interview may take longer than interviews with younger persons. What is the reason for this? a . An aged person has a longer story to tell. b . An aged person is usually lonely and likes to have someone with whom to talk. c . Aged persons lose much of their mental abilities and require longer time to complete an interview. d . As a person ages, he or she is unable to hear; thus the interviewer usually needs to repeat much of what is said. ANS: A The interview usually takes longer with older adults because they have a longer story to tell. It is not necessarily true that all older adults are lonely, have lost mental abilities, or are hard of hearing. DIF: Cognitive Level: Understanding (Comprehension) REF: dm. 40 MSC: Client Needs: Psychosocial Integrity 26. The nurse is interviewing a male patient who has a hearing impairment. What techniques would be most beneficial in communicating with this patient? a . Determine the communication method he prefers. b . Avoid using facial and hand gestures because most hearing-impaired people find this degrading. c . Request a sign language interpreter before meeting with him to help facilitate the communication. d . Speak loudly and with exaggerated facial movement when talking with him because doing so will help him lip read. ANS: A The nurse should ask the deaf person the preferred way to communicateby signing, lip reading, or writing. If the person prefers lip reading, then the nurse should be sure to face him squarely and have good lighting on the nurses face. The nurse should not exaggerate lip movements because this distorts words. Similarly, shouting distorts the reception of a hearing aid the person may wear. The nurse should speak slowly and supplement his or her voice with appropriate hand gestures or pantomime. DIF: Cognitive Level: Understanding (Comprehension) REF: dm. 41 MSC: Client Needs: Psychosocial Integrity 27. During a prenatal check, a patient begins to cry as the nurse asks her about previous pregnancies. She states that she is remembering her last pregnancy, which ended in miscarriage. The nurses best response to her crying would be: a . Im so sorry for making you cry! b . I can see that you are sad remembering this. It is all right to cry. c . Why dont I step out for a few minutes until youre feeling better? d . I can see that you feel sad about this; why dont we talk about something else? ANS: B A beginning examiner usually feels horrified when the patient starts crying. When the nurse says something that makes the person cry, the nurse should not think he or she has hurt the person. The nurse has simply hit on an important topic; therefore, moving on to a new topic is essential. The nurse should allow the person to cry and to express his or her feelings fully. The nurse can offer a tissue and wait until the crying subsides to talk. DIF: Cognitive Level: Applying (Application) REF: dm. 42 MSC: Client Needs: Psychosocial Integrity 28. A female nurse is interviewing a man who has recently immigrated. During the course of the interview, he leans forward and then finally moves his chair close enough that his knees are nearly touching the nurses knees. The nurse begins to feel uncomfortable with his proximity. Which statement most closely reflects what the nurse should do next? a . The nurse should try to relax; these behaviors are culturally appropriate for this person. b . The nurse should discreetly move his or her chair back until the distance is more comfortable, and then continue with the interview. c . These behaviors are indicative of sexual aggression, and the nurse should confront this person about his behaviors. d . The nurse should laugh but tell him that he or she is uncomfortable with his proximity and ask him to move away. ANS: A Both the patients and the nurses sense of spatial distance are significant throughout the interview and physical examination, with culturally appropriate distance zones varying widely. Some cultural groups value close physical proximity and may perceive a health care provider who is distancing him or herself as being aloof and unfriendly. DIF: Cognitive Level: Analyzing (Analysis) REF: dm. 29 MSC: Client Needs: Psychosocial Integrity 29. A female American Indian has come to the clinic for follow-up diabetic teaching. During the interview, the nurse notices that she never makes eye contact and speaks mostly to the floor. Which statement is true regarding this situation? a . The woman is nervous and embarrassed. b . She has something to hide and is ashamed. c . The woman is showing inconsistent verbal and nonverbal behaviors. d . She is showing that she is carefully listening to what the nurse is saying. ANS: D Eye contact is perhaps among the most culturally variable nonverbal behaviors. Asian, American Indian, Indochinese, Arabian, and Appalachian people may consider direct eye contact impolite or aggressive, and they may avert their eyes during the interview. American Indians often stare at the floor during the interview, which is a culturally appropriate behavior, indicating that the listener is paying close attention to the speaker. DIF: Cognitive Level: Analyzing (Analysis) REF: dm. 36 MSC: Client Needs: Psychosocial Integrity 30. The nurse is performing a health interview on a patient who has a language barrier, and no interpreter is available. Which is the best example of an appropriate question for the nurse to ask in this situation? a . Do you take medicine? b . Do you sterilize the bottles? c . Do you have nausea and vomiting? d . You have been taking your medicine, havent you? ANS: A In a situation during which a language barrier exists and no interpreter is available, simple words should be used, avoiding medical jargon. The use of contractions and pronouns should also be avoided. Nouns should be repeatedly used, and one topic at a time should be discussed. DIF: Cognitive Level: Analyzing (Analysis) REF: dm. 46 MSC: Client Needs: Psychosocial Integrity 31. A man arrives at the clinic for his annual wellness physical. He is experiencing no acute health problems. Which question or statement by the nurse is most appropriate when beginning the interview? a . How is your family? b . How is your job? c . Tell me about your hypertension. d How has your health been since your last visit? . ANS: D Open-ended questions are used for gathering narrative information. This type of questioning should be used to begin the interview, to introduce a new section of questions, and whenever the person introduces a new topic. DIF: Cognitive Level: Applying (Application) REF: dm. 31 MSC: Client Needs: Psychosocial Integrity 32. The nurse makes this comment to a patient, I know it may be hard, but you should do what the doctor ordered because she is the expert in this field. Which statement is correct about the nurses comment? a . This comment is inappropriate because it shows the nurses bias. b . This comment is appropriate because members of the health care team are experts in their area of patient care. c . This type of comment promotes dependency and inferiority on the part of the patient and is best avoided in an interview situation. d . Using authority statements when dealing with patients, especially when they are undecided about an issue, is necessary at times. ANS: C Using authority responses promotes dependency and inferiority. Avoiding the use of authority is best. Although the health care provider and patient do not have equal professional knowledge, both have equally worthy roles in the health process. The other statements are not correct. DIF: Cognitive Level: Applying (Application) REF: dm. 34 MSC: Client Needs: Psychosocial Integrity 33. A female patient does not speak English well, and the nurse needs to choose an interpreter. Which of the following would be the most appropriate choice? a . Trained interpreter b . Male family member c . Female family member d . Volunteer college student from the foreign language studies department ANS: A Whenever possible, the nurse should use a trained interpreter, preferably one who knows medical terminology. In general, an older, more mature interpreter is preferred to a younger, less experienced one, and the same gender is preferred when possible. DIF: Cognitive Level: Understanding (Comprehension) REF: dm. 44 MSC: Client Needs: Psychosocial Integrity 34. During a follow-up visit, the nurse discovers that a patient has not been taking his insulin on a regular basis. The nurse asks, Why havent you taken your insulin? Which statement is an appropriate evaluation of this question? a . This question may place the patient on the defensive. b . This question is an innocent search for information. c . Discussing his behavior with his wife would have been better. d . A direct question is the best way to discover the reasons for his behavior. ANS: A The adults use of why questions usually implies blame and condemnation and places the person on the defensive. The other statements are not correct. DIF: Cognitive Level: Analyzing (Analysis) REF: dm. 34 MSC: Client Needs: Psychosocial Integrity 35. The nurse is nearing the end of an interview. Which statement is appropriate at this time? a . Did we forget something? b . Is there anything else you would like to mention? c . I need to go on to the next patient. Ill be back. d . While Im here, lets talk about your upcoming surgery. ANS: B This question offers the person a final opportunity for self-expression. No new topic should be introduced. The other questions are not appropriate. DIF: Cognitive Level: Understanding (Comprehension) REF: dm. 36 MSC: Client Needs: Psychosocial Integrity 36. During the interview portion of data collection, the nurse collects data. a Physical . b Historical . c Objective . d Subjective . ANS: D The interview is the first, and really the most important, part of data collection. During the interview, the nurse collects subjective data; that is, what the person says about him or herself. DIF: Cognitive Level: Remembering (Knowledge) REF: dm. 27 MSC: Client Needs: Psychosocial Integrity 37. During an interview, the nurse would expect that most of the interview will take place at what distance? a . Intimate zone b . Personal distance c . Social distance d . Public distance ANS: C Social distance, 4 to 12 feet, is usually the distance category for most of the interview. Public distance, over 12 feet, is too much distance; the intimate zone is inappropriate, and the personal distance will be used for the physical assessment. DIF: Cognitive Level: Understanding (Comprehension) REF: dm. 29 MSC: Client Needs: Psychosocial Integrity 38. A female nurse is interviewing a male patient who is near the same age as the nurse. During the interview, the patient makes an overtly sexual comment. The nurses best reaction would be: a . Stop that immediately! b . Oh, you are too funny. Lets keep going with the interview. c . Do you really think I would be interested? d . It makes me uncomfortable when you talk that way. Please stop. ANS: D The nurses response must make it clear that she is a health professional who can best care for the person by maintaining a professional relationship. At the same time, the nurse should communicate that he or she accepts the person and understands the persons need to be self- assertive but that sexual advances cannot be tolerated. Chapter 2. Advanced Health Assessment of the Head, Neck, and Lymphatic System MULTIPLE CHOICE 1. A physician tells the nurse that a patients vertebra prominens is tender and asks the nurse to reevaluate the area in 1 hour. The area of the body the nurse will assess is: a . Just above the diaphragm. b . Just lateral to the knee cap. c . At the level of the C7 vertebra. d . At the level of the T11 vertebra. ANS: C The C7 vertebra has a long spinous process, called the vertebra prominens, which is palpable when the head is flexed. DIF: Cognitive Level: Applying (Application) REF: dm. 251 MSC: Client Needs: Physiologic Integrity: Physiologic Adaptation 2. A mother brings her 2-month-old daughter in for an examination and says, My daughter rolled over against the wall, and now I have noticed that she has this spot that is soft on the top of her head. Is something terribly wrong? The nurses best response would be: a . Perhaps that could be a result of your dietary intake during pregnancy. b . Your baby may have craniosynostosis, a disease of the sutures of the brain. c . That soft spot may be an indication of cretinism or congenital hypothyroidism. d . That soft spot is normal, and actually allows for growth of the brain during the first year of your babys life. ANS: D Membrane-covered soft spots allow for growth of the brain during the first year of life. They gradually ossify; the triangular-shaped posterior fontanel is closed by 1 to 2 months, and the diamond-shaped anterior fontanel closes between 9 months and 2 years. DIF: Cognitive Level: Applying (Application) REF: dm. 255 MSC: Client Needs: Health Promotion and Maintenance 3. The nurse notices that a patients palpebral fissures are not symmetric. On examination, the nurse may find that damage has occurred to which cranial nerve (CN)? a III . b V . c VII . d VIII . ANS: C Facial muscles are mediated by CN VII; asymmetry of palpebral fissures may be attributable to damage to CN VII (Bell palsy). DIF: Cognitive Level: Applying (Application) REF: dm. 259 MSC: Client Needs: Physiologic Integrity: Physiologic Adaptation 4. A patient is unable to differentiate between sharp and dull stimulation to both sides of her face. The nurse suspects: a Bell palsy. . b . Damage to the trigeminal nerve. c . Frostbite with resultant paresthesia to the cheeks. d . Scleroderma. ANS: B Facial sensations of pain or touch are mediated by CN V, which is the trigeminal nerve. Bell palsy is associated with CN VII damage. Frostbite and scleroderma are not associated with this problem. DIF: Cognitive Level: Applying (Application) REF: dm. 252-253 MSC: Client Needs: Physiologic Integrity: Physiologic Adaptation 5. When examining the face of a patient, the nurse is aware that the two pairs of salivary glands that are accessible to examination are the and glands. a Occipital; submental . b Parotid; jugulodigastric . c Parotid; submandibular . d Submandibular; occipital . ANS: C Two pairs of salivary glands accessible to examination on the face are the parotid glands, which are in the cheeks over the mandible, anterior to and below the ear; and the submandibular glands, which are beneath the mandible at the angle of the jaw. The parotid glands are normally nonpalpable. DIF: Cognitive Level: Understanding (Comprehension) REF: dm. 253 MSC: Client Needs: Safe and Effective Care Environment: Management of Care 6. A patient comes to the clinic complaining of neck and shoulder pain and is unable to turn her head. The nurse suspects damage to CN and proceeds with the examination by . a XI; palpating the anterior and posterior triangles . b . XI; asking the patient to shrug her shoulders against resistance c . XII; percussing the sternomastoid and submandibular neck muscles d . XII; assessing for a positive Romberg sign ANS: B The major neck muscles are the sternomastoid and the trapezius. They are innervated by CN XI, the spinal accessory. The innervated muscles assist with head rotation and head flexion, movement of the shoulders, and extension and turning of the head. DIF: Cognitive Level: Analyzing (Analysis) REF: dm. 260 MSC: Client Needs: Safe and Effective Care Environment: Management of Care 7. When examining a patients CN function, the nurse remembers that the muscles in the neck that are innervated by CN XI are the: a Sternomastoid and trapezius. . b Spinal accessory and omohyoid. . c Trapezius and sternomandibular. . d Sternomandibular and spinal accessory. . ANS: A The major neck muscles are the sternomastoid and the trapezius. They are innervated by CN XI, the spinal accessory. DIF: Cognitive Level: Remembering (Knowledge) REF: dm. 253 MSC: Client Needs: General 8. A patients laboratory data reveal an elevated thyroxine (T4) level. The nurse would proceed with an examination of the gland. a Thyroid . b Parotid . c Adrenal . d Parathyroid . ANS: A The thyroid gland is a highly vascular endocrine gland that secretes T4 and triiodothyronine (T3). The other glands do not secrete T4. DIF: Cognitive Level: Understanding (Comprehension) REF: dm. 253 MSC: Client Needs: Safe and Effective Care Environment: Management of Care 9. A patient says that she has recently noticed a lump in the front of her neck below her Adams apple that seems to be getting bigger. During the assessment, the finding that leads the nurse to suspect that this may not be a cancerous thyroid nodule is that the lump (nodule): a Is tender. . b Is mobile and not hard. . c Disappears when the patient smiles. . d Is hard and fixed to the surrounding structures. . ANS: B Painless, rapidly growing nodules may be cancerous, especially the appearance of a single nodule in a young person. However, cancerous nodules tend to be hard and fixed to surrounding structures, not mobile. DIF: Cognitive Level: Applying (Application) REF: dm. 262 MSC: Client Needs: Physiologic Integrity: Physiologic Adaptation 10. The nurse notices that a patients submental lymph nodes are enlarged. In an effort to identify the cause of the node enlargement, the nurse would assess the patients: a . Infraclavicular area. b . Supraclavicular area. c . Area distal to the enlarged node. d Area proximal to the enlarged node. . ANS: D When nodes are abnormal, the nurse should check the area into which they drain for the source of the problem. The area proximal (upstream) to the location of the abnormal node should be explored. DIF: Cognitive Level: Analyzing (Analysis) REF: dm. 255 MSC: Client Needs: Safe and Effective Care Environment: Management of Care 11. The nurse is aware that the four areas in the body where lymph nodes are accessible are the: a . Head, breasts, groin, and abdomen. b . Arms, breasts, inguinal area, and legs. c . Head and neck, arms, breasts, and axillae. d . Head and neck, arms, inguinal area, and axillae. ANS: D Nodes are located throughout the body, but they are accessible to examination only in four areas: head and neck, arms, inguinal region, and axillae. DIF: Cognitive Level: Remembering (Knowledge) REF: dm. 255 MSC: Client Needs: Safe and Effective Care Environment: Management of Care 12. A mother brings her newborn in for an assessment and asks, Is there something wrong with my baby? His head seems so big. Which statement is true regarding the relative proportions of the head and trunk of the newborn? a . At birth, the head is one fifth the total length. b . Head circumference should be greater than chest circumference at birth. c . The head size reaches 90% of its final size when the child is 3 years old. d . When the anterior fontanel closes at 2 months, the head will be more proportioned to the body. ANS: B The nurse recognizes that during the fetal period, head growth predominates. Head size is greater than chest circumference at birth, and the head size grows during childhood, reaching 90% of its final size when the child is age 6 years. DIF: Cognitive Level: Understanding (Comprehension) REF: dm. 264 MSC: Client Needs: Health Promotion and Maintenance 13. A patient, an 85-year-old woman, is complaining about the fact that the bones in her face have become more noticeable. What explanation should the nurse give her? a . Diets low in protein and high in carbohydrates may cause enhanced facial bones. b . Bones can become more noticeable if the person does not use a dermatologically approved moisturizer. c . More noticeable facial bones are probably due to a combination of factors related to aging, such as decreased elasticity, subcutaneous fat, and moisture in her skin. d . Facial skin becomes more elastic with age. This increased elasticity causes the skin to be more taught, drawing attention to the facial bones. ANS: C The facial bones and orbits appear more prominent in the aging adult, and the facial skin sags, which is attributable to decreased elasticity, decreased subcutaneous fat, and decreased moisture in the skin. DIF: Cognitive Level: Understanding (Comprehension) REF: dm. 255 MSC: Client Needs: Health Promotion and Maintenance 14. A patient reports excruciating headache pain on one side of his head, especially around his eye, forehead, and cheek that has lasted approximately to 2 hours, occurring once or twice each day. The nurse should suspect: a . Hypertension. b . Cluster headaches. c . Tension headaches. d . Migraine headaches. ANS: B Cluster headaches produce pain around the eye, temple, forehead, and cheek and are unilateral and always on the same side of the head. They are excruciating and occur once or twice per day and last to 2 hours each. DIF: Cognitive Level: Applying (Application) REF: dm. 256 MSC: Client Needs: Physiologic Integrity: Physiologic Adaptation 15. A patient complains that while studying for an examination he began to notice a severe headache in the frontotemporal area of his head that is throbbing and is somewhat relieved when he lies down. He tells the nurse that his mother also had these headaches. The nurse suspects that he may be suffering from: a . Hypertension. b . Cluster headaches. c . Tension headaches. d . Migraine headaches. ANS: D Migraine headaches tend to be supraorbital, retroorbital, or frontotemporal with a throbbing quality. They are severe in quality and are relieved by lying down. Migraines are associated with a family history of migraine headaches. DIF: Cognitive Level: Applying (Application) REF: dm. 256 MSC: Client Needs: Physiologic Integrity: Physiologic Adaptation 16. A 19-year-old college student is brought to the emergency department with a severe headache he describes as, Like nothing Ive ever had before. His temperature is 40 C, and he has a stiff neck. The nurse looks for other signs and symptoms of which problem? a . Head injury b . Cluster headache c . Migraine headache d . Meningeal inflammation ANS: D The acute onset of neck stiffness and pain along with headache and fever occurs with meningeal inflammation. A severe headache in an adult or child who has never had it before is a red flag. Head injury and cluster or migraine headaches are not associated with a fever or stiff neck. DIF: Cognitive Level: Analyzing (Analysis) REF: dm. 258 MSC: Client Needs: Physiologic Integrity: Physiologic Adaptation 17. During a well-baby checkup, the nurse notices that a 1-week-old infants face looks small compared with his cranium, which seems enlarged. On further examination, the nurse also notices dilated scalp veins and downcast or setting sun eyes. The nurse suspects which condition? a . Craniotabes b . Microcephaly c . Hydrocephalus d . Caput succedaneum ANS: C Hydrocephalus occurs with the obstruction of drainage of cerebrospinal fluid that results in excessive accumulation, increasing intracranial pressure, and an enlargement of the head. The face looks small, compared with the enlarged cranium, and dilated scalp veins and downcast or setting sun eyes are noted. Craniotabes is a softening of the skulls outer layer. Microcephaly is an abnormally small head. A caput succedaneum is edematous swelling and ecchymosis of the presenting part of the head caused by birth trauma. DIF: Cognitive Level: Applying (Application) REF: dm. 272 MSC: Client Needs: Health Promotion and Maintenance 18. The nurse needs to palpate the temporomandibular joint for crepitation. This joint is located just below the temporal artery and anterior to the: a Hyoid bone. . b Vagus nerve. . c Tragus. . d Mandible. . ANS: C The temporomandibular joint is just below the temporal artery and anterior to the tragus. DIF: Cognitive Level: Understanding (Comprehension) REF: dm. 259 MSC: Client Needs: General 19. A patient has come in for an examination and states, I have this spot in front of my ear lobe on my cheek that seems to be getting bigger and is tender. What do you think it is? The nurse notes swelling below the angle of the jaw and suspects that it could be an inflammation of his: a Thyroid gland. . b Parotid gland. . c Occipital lymph node. . d Submental lymph node. . ANS: B Swelling of the parotid gland is evident below the angle of the jaw and is most visible when the head is extended. Painful inflammation occurs with mumps, and swelling also occurs with abscesses or tumors. Swelling occurs anterior to the lower ear lobe. DIF: Cognitive Level: Applying (Application) REF: dm. 253 MSC: Client Needs: Physiologic Integrity: Physiologic Adaptation 20. A male patient with a history of acquired immunodeficiency syndrome (AIDS) has come in for an examination and he states, I think that I have the mumps. The nurse would begin by examining the: a Thyroid gland. . b Parotid gland. . c Cervical lymph nodes. . d Mouth and skin for lesions. . ANS: B The parotid gland may become swollen with the onset of mumps, and parotid enlargement has been found with human immunodeficiency virus (HIV). DIF: Cognitive Level: Applying (Application) REF: dm. 276 MSC: Client Needs: Physiologic Integrity: Physiologic Adaptation 21. The nurse suspects that a patient has hyperthyroidism, and the laboratory data indicate that the patients T4 and T3 hormone levels are elevated. Which of these findings would the nurse most likely find on examination? a Tachycardia . b Constipation . c Rapid dyspnea . d Atrophied nodular thyroid gland . ANS: A T4 and T3 are thyroid hormones that stimulate the rate of cellular metabolism, resulting in tachycardia. With an enlarged thyroid gland as in hyperthyroidism, the nurse might expect to find diffuse enlargement (goiter) or a nodular lump but not an atrophied gland. Dyspnea and constipation are not findings associated with hyperthyroidism. DIF: Cognitive Level: Analyzing (Analysis) REF: dm. 277 MSC: Client Needs: Physiologic Integrity: Physiologic Adaptation 22. A visitor from Poland who does not speak English seems to be somewhat apprehensive about the nurse examining his neck. He would probably be more comfortable with the nurse examining his thyroid gland from: a Behind with the nurses hands placed firmly around his neck. . b The side with the nurses eyes averted toward the ceiling and thumbs on his . neck. c The front with the nurses thumbs placed on either side of his trachea and his . head tilted forward. d The front with the nurses thumbs placed on either side of his trachea and his . head tilted backward. ANS: C Examining this patients thyroid gland from the back may be unsettling for him. It would be best to examine his thyroid gland using the anterior approach, asking him to tip his head forward and to the right and then to the left. DIF: Cognitive Level: Applying (Application) REF: dm. 263 MSC: Client Needs: Safe and Effective Care Environment: Management of Care 23. A patients thyroid gland is enlarged, and the nurse is preparing to auscultate the thyroid gland for the presence of a bruit. A bruit is a sound that is heard best with the of the stethoscope. a Low gurgling; diaphragm . b Loud, whooshing, blowing; bell . c Soft, whooshing, pulsatile; bell . d High-pitched tinkling; diaphragm . ANS: C If the thyroid gland is enlarged, then the nurse should auscultate it for the presence of a bruit, which is a soft, pulsatile, whooshing, blowing sound heard best with the bell of the stethoscope. DIF: Cognitive Level: Understanding (Comprehension) REF: dm. 264 MSC: Client Needs: Safe and Effective Care Environment: Management of Care 24. The nurse notices that an infant has a large, soft lump on the side of his head and that his mother is very concerned. She tells the nurse that she noticed the lump approximately 8 hours after her babys birth and that it seems to be getting bigger. One possible explanation for this is: a Hydrocephalus. . b Craniosynostosis. . c Cephalhematoma. . d Caput succedaneum. . ANS: C A cephalhematoma is a subperiosteal hemorrhage that is the result of birth trauma. It is soft, fluctuant, and well defined over one cranial bone. It appears several hours after birth and gradually increases in size. DIF: Cognitive Level: Analyzing (Analysis) REF: dm. 265 MSC: Client Needs: Physiologic Integrity: Physiologic Adaptation 25. A mother brings in her newborn infant for an assessment and tells the nurse that she has noticed that whenever her newborns head is turned to the right side, she straightens out the arm and leg on the same side and flexes the opposite arm and leg. After observing this on examination, the nurse tells her that this reflex is: a Abnormal and is called the atonic neck reflex. . b Normal and should disappear by the first year of life. . c Normal and is called the tonic neck reflex, which should disappear between 3 . and 4 months of age. d Abnormal. The baby should be flexing the arm and leg on the right side of his . body when the head is turned to the right. ANS: C By 2 weeks, the infant shows the tonic neck reflex when supine and the head is turned to one side (extension of same arm and leg, flexion of opposite arm and leg). The tonic neck reflex disappears between 3 and 4 months of age. DIF: Cognitive Level: Analyzing (Analysis) REF: dm. 266 MSC: Client Needs: Health Promotion and Maintenance 26. During an admission assessment, the nurse notices that a male patient has an enlarged and rather thick skull. The nurse suspects acromegaly and would further assess for: a Exophthalmos. . b Bowed long bones. . c Coarse facial features. . d Acorn-shaped cranium. . ANS: C Acromegaly is excessive secretion of growth hormone that creates an enlarged skull and thickened cranial bones. Patients will have elongated heads, massive faces, prominent noses and lower jaws, heavy eyebrow ridges, and coarse facial features. Exophthalmos is associated with hyperthyroidism. Bowed long bones and an acorn-shaped cranium result from Paget disease. DIF: Cognitive Level: Analyzing (Analysis) REF: dm. 278 MSC: Client Needs: Physiologic Integrity: Physiologic Adaptation 27. When examining children affected with Down syndrome (trisomy 21), the nurse looks for the possible presence of: a Ear dysplasia. . b Long, thin neck. . c Protruding thin tongue. . d Narrow and raised nasal bridge. . ANS: A With the chromosomal aberration trisomy 21, also known as Down syndrome, head and face characteristics may include upslanting eyes with inner epicanthal folds, a flat nasal bridge, a small broad flat nose, a protruding thick tongue, ear dysplasia, a short broad neck with webbing, and small hands with a single palmar crease. DIF: Cognitive Level: Understanding (Comprehension) REF: dm. 272 MSC: Client Needs: Physiologic Integrity: Physiologic Adaptation 28. A patient visits the clinic because he has recently noticed that the left side of his mouth is paralyzed. He states that he cannot raise his eyebrow or whistle. The nurse suspects that he has: a Cushing syndrome. . b Parkinson disease. . c Bell palsy. . d Experienced a cerebrovascular accident (CVA) or stroke. . ANS: D With an upper motor neuron lesion, as with a CVA, the patient will have paralysis of lower facial muscles, but the upper half of the face will not be affected owing to the intact nerve from the unaffected hemisphere. The person is still able to wrinkle the forehead and close the eyes. (See Table 13-4, Abnormal Facial Appearances with Chronic Illnesses, for descriptions of the other responses.) DIF: Cognitive Level: Applying (Application) REF: dm. 278 MSC: Client Needs: Physiologic Integrity: Physiologic Adaptation 29. A woman comes to the clinic and states, Ive been sick for so long! My eyes have gotten so puffy, and my eyebrows and hair have become coarse and dry. The nurse will assess for other signs and symptoms of: a Cachexia. . b Parkinson syndrome. . c Myxedema. . d Scleroderma. . ANS: C Myxedema (hypothyroidism) is a deficiency of thyroid hormone that, when severe, causes a nonpitting edema or myxedema. The patient will have a puffy edematous face, especially around the eyes (periorbital edema); coarse facial features; dry skin; and dry, coarse hair and eyebrows. (See Table 13-4, Abnormal Facial Appearances with Chronic Illnesses, for descriptions of the other responses.) DIF: Cognitive Level: Applying (Application) REF: dm. 277 MSC: Client Needs: Physiologic Integrity: Physiologic Adaptation 30. During an examination of a female patient, the nurse notes lymphadenopathy and suspects an acute infection. Acutely infected lymph nodes would be: a Clumped. . b Unilateral. . c Firm but freely movable. . d Firm and nontender. . ANS: C Acutely infected lymph nodes are bilateral, enlarged, warm, tender, and firm but freely movable. Unilaterally enlarged nodes that are firm and nontender may indicate cancer. DIF: Cognitive Level: Understanding (Comprehension) REF: dm. 262 MSC: Client Needs: Physiologic Integrity: Physiologic Adaptation 31. The physician reports that a patient with a neck tumor has a tracheal shift. The nurse is aware that this means that the patients trachea is: a Pulled to the affected side. . b Pushed to the unaffected side. . c Pulled downward. . d Pulled downward in a rhythmic pattern. . ANS: B The trachea is pushed to the unaffected side with an aortic aneurysm, a tumor, unilateral thyroid lobe enlargement, or a pneumothorax. The trachea is pulled to the affected side with large atelectasis, pleural adhesions, or fibrosis. Tracheal tug is a rhythmic downward pull that is synchronous with systole and occurs with aortic arch aneurysm. DIF: Cognitive Level: Understanding (Comprehension) REF: dm. 263 MSC: Client Needs: Physiologic Integrity: Physiologic Adaptation 32. During an assessment of an infant, the nurse notes that the fontanels are depressed and sunken. The nurse suspects which condition? a Rickets . b Dehydration . c Mental retardation . d Increased intracranial pressure . ANS: B Depressed and sunken fontanels occur with dehydration or malnutrition. Mental retardation and rickets have no effect on the fontanels. Increased intracranial pressure would cause tense or bulging and possibly pulsating fontanels. DIF: Cognitive Level: Applying (Application) REF: dm. 266 MSC: Client Needs: Physiologic Integrity: Physiologic Adaptation 33. The nurse is performing an assessment on a 7-year-old child who has symptoms of chronic watery eyes, sneezing, and clear nasal drainage. The nurse notices the presence of a transverse line across the bridge of the nose, dark blue shadows below the eyes, and a double crease on the lower eyelids. These findings are characteristic of: a Allergies. . b Sinus infection. . c Nasal congestion. . d Upper respiratory infection. . ANS: A Chronic allergies often develop chronic facial characteristics and include blue shadows below the eyes, a double or single crease on the lower eyelids, open-mouth breathing, and a transverse line on the nose. DIF: Cognitive Level: Analyzing (Analysis) REF: dm. 273 MSC: Client Needs: Physiologic Integrity: Physiologic Adaptation 34. While performing a well-child assessment on a 5 year old, the nurse notes the presence of palpable, bilateral, cervical, and inguinal lymph nodes. They are approximately 0.5 cm in size, round, mobile, and nontender. The nurse suspects that this child: a Has chronic allergies. . b May have an infection. . c Is exhibiting a normal finding for a well child of this age. . d Should be referred for additional evaluation. . ANS: C Palpable lymph nodes are normal in children until puberty when the lymphoid tissue begins to atrophy. Lymph nodes may be up to 1 cm in size in the cervical and inguinal areas but are discrete, movable, and nontender. DIF: Cognitive Level: Analyzing (Analysis) REF: dm. 267 MSC: Client Needs: Health Promotion and Maintenance 35. The nurse has just completed a lymph node assessment on a 60-year-old healthy female patient. The nurse knows that most lymph nodes in healthy adults are normally: a Shotty. . b Nonpalpable. . c Large, firm, and fixed to the tissue. . d Rubbery, discrete, and mobile. . ANS: B Most lymph nodes are nonpalpable in adults. The palpability of lymph nodes decreases with age. Normal nodes feel movable, discrete, soft, and nontender. DIF: Cognitive Level: Applying (Application) REF: dm. 262 MSC: Client Needs: Physiologic Integrity: Physiologic Adaptation 36. During an examination of a patient in her third trimester of pregnancy, the nurse notices that the patients thyroid gland is slightly enlarged. No enlargement had been previously noticed. The nurse suspects that the patient: a Has an iodine deficiency. . b Is exhibiting early signs of goiter. . c Is exhibiting a normal enlargement of the thyroid gland during pregnancy. . d Needs further testing for possible thyroid cancer. . ANS: C The thyroid gland enlarges slightly during pregnancy because of hyperplasia of the tissue and increased vascularity. DIF: Cognitive Level: Applying (Application) REF: dm. 255 MSC: Client Needs: Safe and Effective Care Environment: Management of Care 37. During an examination, the nurse knows that the best way to palpate the lymph nodes in the neck is described by which statement? a Using gentle pressure, palpate with both hands to compare the two sides. . b . Using strong pressure, palpate with both hands to compare the two sides. c . Gently pinch each node between ones thumb and forefinger, and then move down the neck muscle. d . Using the index and middle fingers, gently palpate by applying pressure in a rotating pattern. ANS: A Using gentle pressure is recommended because strong pressure can push the nodes into the neck muscles. Palpating with both hands to compare the two sides symmetrically is usually most efficient. DIF: Cognitive Level: Understanding (Comprehension) REF: dm. 260 MSC: Client Needs: Safe and Effective Care Environment: Management of Care 38. During a well-baby checkup, a mother is concerned because her 2-month-old infant cannot hold her head up when she is pulled to a sitting position. Which response by the nurse is appropriate? a Head control is usually achieved by 4 months of age. . b You shouldnt be trying to pull your baby up like that until she is older. . c Head control should be achieved by this time. . d This inability indicates possible nerve damage to the neck muscles. . ANS: A Head control is achieved by 4 months when the baby can hold the head erect and steady when pulled to a vertical position. The other responses are not appropriate. Chapter 3. Advanced Health Assessment of the Nose, Mouth, and Throat MULTIPLE CHOICE 1. The primary purpose of the ciliated mucous membrane in the nose is to: a Warm the inhaled air. . b Filter out dust and bacteria. . c Filter coarse particles from inhaled air. . d Facilitate the movement of air through the nares. . ANS: B The nasal hairs filter the coarsest matter from inhaled air, whereas the mucous blanket filters out dust and bacteria. The rich blood supply of the nasal mucosa warms the inhaled air. DIF: Cognitive Level: Remembering (Knowledge) REF: dm. 353 MSC: Client Needs: General 2. The projections in the nasal cavity that increase the surface area are called the: a Meatus. . b Septum. . c Turbinates. . d Kiesselbach plexus. . ANS: C The lateral walls of each nasal cavity contain three parallel bony projections: the superior, middle, and inferior turbinates. These increase the surface area, making more blood vessels and mucous membrane available to warm, humidify, and filter the inhaled air. DIF: Cognitive Level: Remembering (Knowledge) REF: dm. 353 MSC: Client Needs: General 3. The nurse is reviewing the development of the newborn infant. Regarding the sinuses, which statement is true in relation to a newborn infant? a Sphenoid sinuses are full size at birth. . b Maxillary sinuses reach full size after puberty. . c Frontal sinuses are fairly well developed at birth. . d Maxillary and ethmoid sinuses are the only sinuses present at birth. . ANS: D Only the maxillary and ethmoid sinuses are present at birth. The sphenoid sinuses are minute at birth and develop after puberty. The frontal sinuses are absent at birth, are fairly well developed at age 7 to 8 years, and reach full size after puberty. DIF: Cognitive Level: Remembering (Knowledge) REF: dm. 355 MSC: Client Needs: General 4. The tissue that connects the tongue to the floor of the mouth is the: a Uvula. . b Palate. . c Papillae. . d Frenulum. . ANS: D The frenulum is a midline fold of tissue that connects the tongue to the floor of the mouth. The uvula is the free projection hanging down from the middle of the soft palate. The palate is the arching roof of the mouth. Papillae are the rough, bumpy elevations on the tongues dorsal surface. DIF: Cognitive Level: Remembering (Knowledge) REF: dm. 355 MSC: Client Needs: General 5. The salivary gland that is the largest and located in the cheek in front of the ear is the gland. a Parotid . b Stensens . c Sublingual . d Submandibular . ANS: A The mouth contains three pairs of salivary glands. The largest, the parotid gland, lies within the cheeks in front of the ear extending from the zygomatic arch down to the angle of the jaw. The Stensens duct (not gland) drains the parotid gland onto the buccal mucosa opposite the second molar. The sublingual gland is located within the floor of the mouth under the tongue. The submandibular gland lies beneath the mandible at the angle of the jaw. DIF: Cognitive Level: Remembering (Knowledge) REF: dm. 356 MSC: Client Needs: General 6. In assessing the tonsils of a 30 year old, the nurse notices that they are involuted, granular in appearance, and appear to have deep crypts. What is correct response to these findings? a Refer the patient to a throat specialist. . b No response is needed; this appearance is normal for the tonsils. . c Continue with the assessment, looking for any other abnormal findings. . d Obtain a throat culture on the patient for possible streptococcal (strep) infection. . ANS: B The tonsils are the same color as the surrounding mucous membrane, although they look more granular and their surface shows deep crypts. Tonsillar tissue enlarges during childhood until puberty and then involutes. DIF: Cognitive Level: Applying (Application) REF: dm. 356 MSC: Client Needs: Safe and Effective Care Environment: Management of Care 7. The nurse is obtaining a health history on a 3-month-old infant. During the interview, the mother states, I think she is getting her first tooth because she has started drooling a lot. The nurses best response would be: a Youre right, drooling is usually a sign of the first tooth. . b It would be unusual for a 3 month old to be getting her first tooth. . c This could be the sign of a problem with the salivary glands. . d She is just starting to salivate and hasnt learned to swallow the saliva. . ANS: D In the infant, salivation starts at 3 months. The baby will drool for a few months before learning to swallow the saliva. This drooling does not herald the eruption of the first tooth, although many parents think it does. DIF: Cognitive Level: Understanding (Comprehension) REF: dm. 356 MSC: Client Needs: Health Promotion and Maintenance 8. The nurse is assessing an 80-year-old patient. Which of these findings would be expected for this patient? a Hypertrophy of the gums . b Increased production of saliva . c Decreased ability to identify odors . d Finer and less prominent nasal hair . ANS: C The sense of smell may be reduced because of a decrease in the number of olfactory nerve fibers. Nasal hairs grow coarser and stiffer with aging. The gums may recede with aging, not hypertrophy, and saliva production decreases. DIF: Cognitive Level: Understanding (Comprehension) REF: dm. 357 MSC: Client Needs: Health Promotion and Maintenance 9. The nurse is performing an oral assessment on a 40-year-old Black patient and notices the presence of a 1 cm, nontender, grayish-white lesion on the left buccal mucosa. Which one of these statements is true? This lesion is: a Leukoedema and is common in dark-pigmented persons. . b The result of hyperpigmentation and is normal. . c Torus palatinus and would normally be found only in smokers. . d Indicative of cancer and should be immediately tested. . ANS: A Leukoedema, a grayish-white benign lesion occurring on the buccal mucosa, is most often observed in Blacks. DIF: Cognitive Level: Understanding (Comprehension) REF: dm. 358 MSC: Client Needs: Physiologic Integrity: Physiologic Adaptation 10. While obtaining a health history, a patient tells the nurse that he has frequent nosebleeds and asks the best way to get them to stop. What would be the nurses best response? a While sitting up, place a cold compress over your nose. . b Sit up with your head tilted forward and pinch your nose. . c Just allow the bleeding to stop on its own, but dont blow your nose. . d Lie on your back with your head tilted back and pinch your nose. . ANS: B With a nosebleed, the person should sit up with the head tilted forward and pinch the nose between the thumb and forefinger for 5 to 15 minutes. DIF: Cognitive Level: Applying (Application) REF: dm. 359 MSC: Client Needs: Physiologic Integrity: Physiologic Adaptation 11. A 92-year-old patient has had a stroke. The right side of his face is drooping. The nurse might also suspect which of these assessment findings? a Epistaxis . b Rhinorrhea . c Dysphagia . d Xerostomia . ANS: C Dysphagia is difficulty with swallowing and may occur with a variety of disorders, including stroke and other neurologic diseases. Rhinorrhea is a runny nose, epistaxis is a bloody nose, and xerostomia is a dry mouth. DIF: Cognitive Level: Analyzing (Analysis) REF: dm. 359 MSC: Client Needs: Physiologic Integrity: Physiologic Adaptation 12. While obtaining a health history from the mother of a 1-year-old child, the nurse notices that the baby has had a bottle in his mouth the entire time. The mother states, It makes a great pacifier. The best response by the nurse would be: a Youre right. Bottles make very good pacifiers. . b Using a bottle as a pacifier is better for the teeth than thumb-sucking. . c Its okay to use a bottle as long as it contains milk and not juice. . d Prolonged use of a bottle can increase the risk for tooth decay and ear . infections. ANS: D Prolonged bottle use during the day or when going to sleep places the infant at risk for tooth decay and middle ear infections. DIF: Cognitive Level: Applying (Application) REF: dm. 360 MSC: Client Needs: Health Promotion and Maintenance 13. A 72-year-old patient has a history of hypertension and chronic lung disease. An important question for the nurse to include in the health history would be: a Do you use a fluoride supplement? . b Have you had tonsillitis in the last year? . c At what age did you get your first tooth? . d Have you noticed any dryness in your mouth? . ANS: D Xerostomia (dry mouth) is a side effect of many drugs taken by older people, including antidepressants, anticholinergics, antispasmodics, antihypertensives, antipsychotics, and bronchodilators. DIF: Cognitive Level: Applying (Application) REF: dm. 360 MSC: Client Needs: Physiologic Integrity: Physiologic Adaptation 14. The nurse is using an otoscope to assess the nasal cavity. Which of these techniques is correct? a Inserting the speculum at least 3 cm into the vestibule . b Avoiding touching the nasal septum with the speculum . c Gently displacing the nose to the side that is being examined . d Keeping the speculum tip medial to avoid touching the floor of the nares . ANS: B The correct technique for using an otoscope is to insert the apparatus into the nasal vestibule, avoiding pressure on the sensitive nasal septum. The tip of the nose should be lifted up before inserting the speculum. DIF: Cognitive Level: Understanding (Comprehension) REF: dm. 362 MSC: Client Needs: Safe and Effective Care Environment: Management of Care 15. The nurse is performing an assessment on a 21-year-old patient and notices that his nasal mucosa appears pale, gray, and swollen. What would be the most appropriate question to ask the patient? a Are you aware of having any allergies? . b Do you have an elevated temperature? . c Have you had any symptoms of a cold? . d Have you been having frequent nosebleeds? . ANS: A With chronic allergies, the mucosa looks swollen, boggy, pale, and gray. Elevated body temperature, colds, and nosebleeds do not cause these mucosal changes. DIF: Cognitive Level: Applying (Application) REF: dm. 362 MSC: Client Needs: Safe and Effective Care Environment: Management of Care 16. The nurse is palpating the sinus areas. If the findings are normal, then the patient should report which sensation? a No sensation . b Firm pressure . c Pain during palpation . d Pain sensation behind eyes . ANS: B The person should feel firm pressure but no pain. Sinus areas are tender to palpation in persons with chronic allergies or an acute infection (sinusitis). DIF: Cognitive Level: Remembering (Knowledge) REF: dm. 363 MSC: Client Needs: Safe and Effective Care Environment: Management of Care 17. During an oral assessment of a 30-year-old Black patient, the nurse notices bluish lips and a dark line along the gingival margin. What action would the nurse perform in response to this finding? a Check the patients hemoglobin for anemia. . b Assess for other signs of insufficient oxygen supply. . c Proceed with the assessment, knowing that this appearance is a normal finding. . d Ask if he has been exposed to an excessive amount of carbon monoxide. . ANS: C Some Blacks may have bluish lips and a dark line on the gingival margin; this appearance is a normal finding. DIF: Cognitive Level: Applying (Application) REF: dm. 363 MSC: Client Needs: Safe and Effective Care Environment: Management of Care 18. During an assessment of a 20-year-old patient with a 3-day history of nausea and vomiting, the nurse notices dry mucosa and deep vertical fissures in the tongue. These findings are reflective of: a Dehydration. . b . Irritation by gastric juices. c . A normal oral assessment. d . Side effects from nausea medication. ANS: A Dry mouth occurs with dehydration or fever. The tongue has deep vertical fissures. DIF: Cognitive Level: Applying (Application) REF: dm. 364 MSC: Client Needs: Physiologic Integrity: Physiologic Adaptation 19. A 32-year-old woman is at the clinic for little white bumps in my mouth. During the assessment, the nurse notes that she has a 0.5 cm white, nontender papule under her tongue and one on the mucosa of her right cheek. What would the nurse tell the patient? a These spots indicate an infection such as strep throat. . b These bumps could be indicative of a serious lesion, so I will refer you to a . specialist. c This condition is called leukoplakia and can be caused by chronic irritation such . as with smoking. d These bumps are Fordyce granules, which are sebaceous cysts and are not a . serious condition. ANS: D Fordyce granules are small, isolated white or yellow papules on the mucosa of the cheek, tongue, and lips. These little sebaceous cysts are painless and are not significant. Chalky, white raised patches would indicate leukoplakia. In strep throat, the examiner would see tonsils that are bright red, swollen, and may have exudates or white spots. DIF: Cognitive Level: Applying (Application) REF: dm. 366 MSC: Client Needs: Physiologic Integrity: Physiologic Adaptation 20. A 10 year old is at the clinic for a sore throat that has lasted 6 days. Which of these findings would be consistent with an acute infection? a . Tonsils 1+/1-4+ and pink; the same color as the oral mucosa b . Tonsils 2+/1-4+ with small plugs of white debris c Tonsils 3+/1-4+ with large white spots . d Tonsils 3+/1-4+ with pale coloring . ANS: C With an acute infection, tonsils are bright red and swollen and may have exudate or large white spots. Tonsils are enlarged to 2+, 3+, or 4+ with an acute infection. DIF: Cognitive Level: Understanding (Comprehension) REF: dm. 367 MSC: Client Needs: Physiologic Integrity: Physiologic Adaptation 21. Immediately after birth, the nurse is unable to suction the nares of a newborn. An attempt is made to pass a catheter through both nasal cavities with no success. What should the nurse do next? a . Attempt to suction again with a bulb syringe. b . Wait a few minutes, and try again once the infant stops crying. c . Recognize that this situation requires immediate intervention. d . Contact the physician to schedule an appointment for the infant at his or her next hospital visit. ANS: C Determining the patency of the nares in the immediate newborn period is essential because most newborns are obligate nose breathers. Nares blocked with amniotic fluid are gently suctioned with a bulb syringe. If obstruction is suspected, then a small lumen (5 to 10 Fr) catheter is passed down each naris to confirm patency. The inability to pass a catheter through the nasal cavity indicates choanal atresia, which requires immediate intervention. DIF: Cognitive Level: Analyzing (Analysis) REF: dm. 370 MSC: Client Needs: Physiologic Integrity: Physiologic Adaptation 22. The nurse notices that the mother of a 2-year-old boy brings him into the clinic quite frequently for various injuries and suspects there may be some child abuse involved. During an inspection of his mouth, the nurse should look for: a Swollen, red tonsils. . b Ulcerations on the hard palate. . c Bruising on the buccal mucosa or gums. . d Small yellow papules along the hard palate. . ANS: C The nurse should notice any bruising or laceration on the buccal mucosa or gums of an infant or young child. Trauma may indicate child abuse from a forced feeding of a bottle or spoon. DIF: Cognitive Level: Applying (Application) REF: dm. 370 MSC: Client Needs: Physiologic Integrity: Reduction of Risk Potential 23. The nurse is assessing a 3 year old for drainage from the nose. On assessment, a purulent drainage that has a very foul odor is noted from the left naris and no drainage is observed from the right naris. The child is afebrile with no other symptoms. What should the nurse do next? a Refer to the physician for an antibiotic order. . b Have the mother bring the child back in 1 week. . c Perform an otoscopic examination of the left nares. . d Tell the mother that this drainage is normal for a child of this age. . ANS: C Children are prone to put an object up the nose, producing unilateral purulent drainage with a foul odor. Because some risk for aspiration exists, removal should be prompt. DIF: Cognitive Level: Analyzing (Analysis) REF: dm. 376 MSC: Client Needs: Physiologic Integrity: Physiologic Adaptation 24. During an assessment of a 26 year old at the clinic for a spot on my lip I think is cancer, the nurse notices a group of clear vesicles with an erythematous base around them located at the lip- skin border. The patient mentions that she just returned from Hawaii. What would be the most appropriate response by the nurse? a . Tell the patient she needs to see a skin specialist. b . Discuss the benefits of having a biopsy performed on any unusual lesion. c . Tell the patient that these vesicles are indicative of herpes simplex I or cold sores and that they will heal in 4 to 10 days. d Tell the patient that these vesicles are most likely the result of a riboflavin . deficiency and discuss nutrition. ANS: C Cold sores are groups of clear vesicles with a surrounding erythematous base. These evolve into pustules or crusts and heal in 4 to 10 days. The most likely site is the lip-skin junction. Infection often recurs in the same site. Recurrent herpes infections may be precipitated by sunlight, fever, colds, or allergy. DIF: Cognitive Level: Analyzing (Analysis) REF: dm. 377 MSC: Client Needs: Physiologic Integrity: Physiologic Adaptation 25. While performing an assessment of the mouth, the nurse notices that the patient has a 1-cm ulceration that is crusted with an elevated border and located on the outer third of the lower lip. What other information would be most important for the nurse to assess? a . Nutritional status b . When the patient first noticed the lesion c . Whether the patient has had a recent cold d . Whether the patient has had any recent exposure to sick animals ANS: B With carcinoma, the initial lesion is round and indurated, but then it becomes crusted and ulcerated with an elevated border. Most cancers occur between the outer and middle thirds of the lip. Any lesion that is still unhealed after 2 weeks should be referred. DIF: Cognitive Level: Applying (Application) REF: dm. 365 MSC: Client Needs: Physiologic Integrity: Reduction of Risk Potential 26. A pregnant woman states that she is concerned about her gums because she has noticed they are swollen and have started bleeding. What would be an appropriate response by the nurse? a . Your condition is probably due to a vitamin C deficiency. b . Im not sure what causes swollen and bleeding gums, but let me know if its not better in a few weeks. c . You need to make an appointment with your dentist as soon as possible to have this checked. d Swollen and bleeding gums can be caused by the change in hormonal balance in . your system during pregnancy. ANS: D Gum margins are red and swollen and easily bleed with gingivitis. A changing hormonal balance may cause this condition to occur in pregnancy and puberty. DIF: Cognitive Level: Applying (Application) REF: dm. 357 MSC: Client Needs: Physiologic Integrity: Physiologic Adaptation 27. A 40-year-old patient who has just finished chemotherapy for breast cancer tells the nurse that she is concerned about her mouth. During the assessment the nurse finds areas of buccal mucosa that are raw and red with some bleeding, as well as other areas that have a white, cheesy coating. The nurse recognizes that this abnormality is: a . Aphthous ulcers. b . Candidiasis. c . Leukoplakia. d . Koplik spots. ANS: B Candidiasis is a white, cheesy, curdlike patch on the buccal mucosa and tongue. It scrapes off, leaving a raw, red surface that easily bleeds. It also occurs after the use of antibiotics or corticosteroids and in persons who are immunosuppressed. (See Table 16-4 for descriptions of the other lesions.) DIF: Cognitive Level: Applying (Application) REF: dm. 380 MSC: Client Needs: Physiologic Integrity: Physiologic Adaptation 28. The nurse is assessing a patient in the hospital who has received numerous antibiotics and notices that his tongue appears to be black and hairy. In response to his concern, what would the nurse say? a . We will need to get a biopsy to determine the cause. b . This is an overgrowth of hair and will go away in a few days. c . Black, hairy tongue is a fungal infection caused by all the antibiotics you have received. d This is probably caused by the same bacteria you had in your lungs. . ANS: C A black, hairy tongue is not really hair but the elongation of filiform papillae and painless overgrowth of mycelial threads of fungus infection on the tongue. It occurs after the use of antibiotics, which inhibit normal bacteria and allow a proliferation of fungus. DIF: Cognitive Level: Analyzing (Analysis) REF: dm. 381 MSC: Client Needs: Physiologic Integrity: Physiologic Adaptation 29. The nurse is assessing a patient with a history of intravenous drug abuse. In assessing his mouth, the nurse notices a dark red confluent macule on the hard palate. This could be an early sign of: a . Acquired immunodeficiency syndrome (AIDS). b . Measles. c . Leukemia. d . Carcinoma. ANS: A Oral Kaposis sarcoma is a bruiselike, dark red or violet, confluent macule that usually occurs on the hard palate. It may appear on the soft palate or gingival margin. Oral lesions may be among the earliest lesions to develop with AIDS. DIF: Cognitive Level: Understanding (Comprehension) REF: dm. 383 MSC: Client Needs: Physiologic Integrity: Physiologic Adaptation 30. A mother brings her 4-month-old infant to the clinic with concerns regarding a small pad in the middle of the upper lip that has been there since 1 month of age. The infant has no health problems. On physical examination, the nurse notices a 0.5-cm, fleshy, elevated area in the middle of the upper lip. No evidence of inflammation or drainage is observed. What would the nurse tell this mother? a This area of irritation is caused from teething and is nothing to worry about. . b This finding is abnormal and should be evaluated by another health care . provider. c This area of irritation is the result of chronic drooling and should resolve within . the next month or two. d This elevated area is a sucking tubercle caused from the friction of . breastfeeding or bottle-feeding and is normal. ANS: D A normal finding in infants is the sucking tubercle, a small pad in the middle of the upper lip from the friction of breastfeeding or bottle-feeding. This condition is not caused by irritation, teething, or excessive drooling, and evaluation by another health care provider is not warranted. DIF: Cognitive Level: Analyzing (Analysis) REF: dm. 370 MSC: Client Needs: Health Promotion and Maintenance 31. A mother is concerned because her 18-month-old toddler has 12 teeth. She is wondering if this is normal for a child of this age. The nurses best response would be: a How many teeth did you have at this age? . b All 20 deciduous teeth are expected to erupt by age 4 years. . c This is a normal number of teeth for an 18 month old. . d Normally, by age 2 years, 16 deciduous teeth are expected. . ANS: C The guidelines for the number of teeth for children younger than 2 years old are as follows: the childs age in months minus the number 6 should be equal to the expected number of deciduous teeth. Normally, all 20 teeth are in by 2 years old. In this instance, the child is 18 months old, minus 6, equals 12 deciduous teeth expected. DIF: Cognitive Level: Applying (Application) REF: dm. 356 MSC: Client Needs: Health Promotion and Maintenance 32. When examining the mouth of an older patient, the nurse recognizes which finding is due to the aging process? a . Teeth appearing shorter b . Tongue that looks smoother in appearance c . Buccal mucosa that is beefy red in appearance d Small, painless lump on the dorsum of the tongue . ANS: B In the aging adult, the tongue looks smoother because of papillary atrophy. The teeth are slightly yellowed and appear longer because of the recession of gingival margins. DIF: Cognitive Level: Remembering (Knowledge) REF: dm. 371 MSC: Client Needs: Health Promotion and Maintenance 33. When examining the nares of a 45-year-old patient who has complaints of rhinorrhea, itching of the nose and eyes, and sneezing, the nurse notices the following: pale turbinates, swelling of the turbinates, and clear rhinorrhea. Which of these conditions is most likely the cause? a . Nasal polyps b . Acute sinusitis c . Allergic rhinitis d . Acute rhinitis ANS: C Rhinorrhea, itching of the nose and eyes, and sneezing are present with allergic rhinitis. On physical examination, serous edema is noted, and the turbinates usually appear pale with a smooth, glistening surface. (See Table 16-1 for descriptions of the other conditions.) DIF: Cognitive Level: Analyzing (Analysis) REF: dm. 375 MSC: Client Needs: Physiologic Integrity: Physiologic Adaptation 34. When assessing the tongue of an adult, the nurse knows that an abnormal finding would be: a . Smooth glossy dorsal surface. b . Thin white coating over the tongue. c . Raised papillae on the dorsal surface. d . Visible venous patterns on the ventral surface. ANS: A The dorsal surface of the tongue is normally roughened from papillae. A thin white coating may be present. The ventral surface may show veins. Smooth, glossy areas may indicate atrophic glossitis (see Table 16-5). DIF: Cognitive Level: Understanding (Comprehension) REF: dm. 381 MSC: Client Needs: Physiologic Integrity: Physiologic Adaptation 35. The nurse is performing an assessment. Which of these findings would cause the greatest concern? a . Painful vesicle inside the cheek for 2 days b . Presence of moist, nontender Stensens ducts c . Stippled gingival margins that snugly adhere to the teeth d . Ulceration on the side of the tongue with rolled edges ANS: D Ulceration on the side or base of the tongue or under the tongue raises the suspicion of cancer and must be investigated. The risk of early metastasis is present because of rich lymphatic drainage. The vesicle may be an aphthous ulcer, which is painful but not dangerous. The other responses are normal findings. DIF: Cognitive Level: Applying (Application) REF: dm. 382 MSC: Client Needs: Safe and Effective Care Environment: Management of Care 36. A patient has been diagnosed with strep throat. The nurse is aware that without treatment, which complication may occur? a . Rubella b . Leukoplakia c . Rheumatic fever d . Scarlet fever ANS: C Untreated strep throat may lead to rheumatic fever. When performing a health history, the patient should be asked whether his or her sore throat has been documented as streptococcal. DIF: Cognitive Level: Understanding (Comprehension) REF: dm. 383 MSC: Client Needs: Physiologic Integrity: Reduction of Risk Potential 37. During a checkup, a 22-year-old woman tells the nurse that she uses an over-the-counter nasal spray because of her allergies. She also states that it does not work as well as it used to when she first started using it. The best response by the nurse would be: a . You should never use over-the-counter nasal sprays because of the risk of addiction. b . You should try switching to another brand of medication to prevent this problem. c . Continuing to use this spray is important to keep your allergies under control. d . Using these nasal medications irritates the lining of the nose and may cause rebound swelling. ANS: D The misuse of over-the-counter nasal medications irritates the mucosa, causing rebound swelling, which is a common problem. DIF: Cognitive Level: Analyzing (Analysis) REF: dm. 359 MSC: Client Needs: Physiologic Integrity: Reduction of Risk Potential 38. During an oral examination of a 4-year-old Native-American child, the nurse notices that her uvula is partially split. Which of these statements is accurate? a . This condition is a cleft palate and is common in Native Americans. b . A bifid uvula may occur in some Native-American groups. c . This condition is due to an injury and should be reported to the authorities. d . A bifid uvula is palatinus, which frequently occurs in Native Americans. ANS: B Bifid uvula, a condition in which the uvula is split either completely or partially, occurs in some Native-American groups. DIF: Cognitive Level: Applying (Application) REF: dm. 382 MSC: Client Needs: Physiologic Integrity: Physiologic Adaptation 39. A patient comes into the clinic complaining of facial pain, fever, and malaise. On examination, the nurse notes swollen turbinates and purulent discharge from the nose. The patient also complains of a dull, throbbing pain in his cheeks and teeth on the right side and pain when the nurse palpates the areas. The nurse recognizes that this patient has: a . Posterior epistaxis. b . Frontal sinusitis. c . Maxillary sinusitis. d . Nasal polyps. ANS: C Signs of maxillary sinusitis include facial pain after upper respiratory infection, red swollen nasal mucosa, swollen turbinates, and purulent discharge. The person also has fever, chills, and malaise. With maxillary sinusitis, dull throbbing pain occurs in the cheeks and teeth on the same side, and pain with palpation is present. With frontal sinusitis, pain is above the supraorbital ridge. DIF: Cognitive Level: Analyzing (Analysis) REF: dm. 375 MSC: Client Needs: Physiologic Integrity: Physiologic Adaptation 40. A woman who is in the second trimester of pregnancy mentions that she has had more nosebleeds than ever since she became pregnant. The nurse recognizes that this is a result of: a . A problem with the patients coagulation system. b . Increased vascularity in the upper respiratory tract as a result of the pregnancy. c . Increased susceptibility to colds and nasal irritation. d . Inappropriate use of nasal sprays. ANS: B Nasal stuffiness and epistaxis may occur during pregnancy as a result of increased vascularity in the upper respiratory tract. DIF: Cognitive Level: Applying (Application) REF: dm. 357 MSC: Client Needs: Health Promotion and Maintenance MULTIPLE RESPONSE 1. The nurse is teaching a health class to high-school boys. When discussing the topic of using smokeless tobacco (SLT), which of these statements are accurate? Select all that apply. a . One pinch of SLT in the mouth for 30 minutes delivers the equivalent of one cigarette. b . Using SLT has been associated with a greater risk of oral cancer than smoking. c . Pain is an early sign of oral cancer. d . Pain is rarely an early sign of oral cancer. e . Tooth decay is another risk of SLT because of the use of sugar as a sweetener. f . SLT is considered a healthy alternative to smoking. ANS: B, D, E One pinch of SLT in the mouth for 30 minutes delivers the equivalent of three cigarettes. Pain is rarely an early sign of oral cancer. Many brands of SLT are sweetened with sugars, which promotes tooth decay. SLT is not considered a healthy alternative to smoking, and the use of SLT has been associated with a greater risk of oral cancer than smoking. DIF: Cognitive Level: Analyzing (Analysis) REF: dm. 372 MSC: Client Needs: Health Promotion and Maintenance 2. During an assessment, a patient mentions that I just cant smell like I used to. I can barely smell the roses in my garden. Why is that? For which possible causes of changes in the sense of smell will the nurse assess? Select all that apply. a . Chronic alcohol use b . Cigarette smoking c . Frequent episodes of strep throat d . Chronic allergies e . Aging f Herpes simplex virus I . ANS: B, D, E Sen The sense of smell diminishes with cigarette smoking, chronic allergies, and aging. Chronic alcohol use, a history of strep throat, and herpes simplex virus I are not associated with changes in the sense of smell. Chapter 4. Advanced Health Assessment of the Eyes and Ears MULTIPLE CHOICE 1. When examining the eye, the nurse notices that the patients eyelid margins approximate completely. The nurse recognizes that this assessment finding: a . Is expected. b . May indicate a problem with extraocular muscles. c . May result in problems with tearing. d . Indicates increased intraocular pressure. ANS: A The palpebral fissure is the elliptical open space between the eyelids, and, when closed, the lid margins approximate completely, which is a normal finding. DIF: Cognitive Level: Understanding (Comprehension) REF: dm. 281 MSC: Client Needs: Physiologic Integrity: Physiologic Adaptation 2. During ocular examinations, the nurse keeps in mind that movement of the extraocular muscles is: a . Decreased in the older adult. b . Impaired in a patient with cataracts. c . Stimulated by cranial nerves (CNs) I and II. d Stimulated by CNs III, IV, and VI. . ANS: D Movement of the extraocular muscles is stimulated by three CNs: III, IV, and VI. DIF: Cognitive Level: Remembering (Knowledge) REF: dm. 283 MSC: Client Needs: Physiologic Integrity: Physiologic Adaptation 3. The nurse is performing an external eye examination. Which statement regarding the outer layer of the eye is true? a . The outer layer of the eye is very sensitive to touch. b . The outer layer of the eye is darkly pigmented to prevent light from reflecting internally. c . The trigeminal nerve (CN V) and the trochlear nerve (CN IV) are stimulated when the outer surface of the eye is stimulated. d . The visual receptive layer of the eye in which light waves are changed into nerve impulses is located in the outer layer of the eye. ANS: A The cornea and the sclera make up the outer layer of the eye. The cornea is very sensitive to touch. The middle layer, the choroid, has dark pigmentation to prevent light from reflecting internally. The trigeminal nerve (CN V) and the facial nerve (CN VII) are stimulated when the outer surface of the eye is stimulated. The retina, in the inner layer of the eye, is where light waves are changed into nerve impulses. DIF: Cognitive Level: Understanding (Comprehension) REF: dm. 283 MSC: Client Needs: Physiologic Integrity: Physiologic Adaptation 4. When examining a patients eyes, the nurse recalls that stimulation of the sympathetic branch of the autonomic nervous system: a . Causes pupillary constriction. b . Adjusts the eye for near vision. c . Elevates the eyelid and dilates the pupil. d . Causes contraction of the ciliary body. ANS: C Stimulation of the sympathetic branch of the autonomic nervous system dilates the pupil and elevates the eyelid. Parasympathetic nervous system stimulation causes the pupil to constrict. The muscle fibers of the iris contract the pupil in bright light to accommodate for near vision. The ciliary body controls the thickness of the lens. DIF: Cognitive Level: Understanding (Comprehension) REF: dm. 283 MSC: Client Needs: Physiologic Integrity: Physiologic Adaptation 5. The nurse is reviewing causes of increased intraocular pressure. Which of these factors determines intraocular pressure? a . Thickness or bulging of the lens b . Posterior chamber as it accommodates increased fluid c . Contraction of the ciliary body in response to the aqueous within the eye d . Amount of aqueous produced and resistance to its outflow at the angle of the anterior chamber ANS: D Intraocular pressure is determined by a balance between the amount of aqueous produced and the resistance to its outflow at the angle of the anterior chamber. The other responses are incorrect. DIF: Cognitive Level: Remembering (Knowledge) REF: dm. 284 MSC: Client Needs: Physiologic Integrity: Physiologic Adaptation 6. The nurse is conducting a visual examination. Which of these statements regarding visual pathways and visual fields is true? a . The right side of the brain interprets the vision for the right eye. b . The image formed on the retina is upside down and reversed from its actual appearance in the outside world. c . Light rays are refracted through the transparent media of the eye before striking the pupil. d . Light impulses are conducted through the optic nerve to the temporal lobes of the brain. ANS: B The image formed on the retina is upside down and reversed from its actual appearance in the outside world. The light rays are refracted through the transparent media of the eye before striking the retina, and the nerve impulses are conducted through the optic nerve tract to the visual cortex of the occipital lobe of the brain. The left side of the brain interprets vision for the right eye. DIF: Cognitive Level: Remembering (Knowledge) REF: dm. 284 MSC: Client Needs: Physiologic Integrity: Physiologic Adaptation 7. The nurse is testing a patients visual accommodation, which refers to which action? a . Pupillary constriction when looking at a near object b . Pupillary dilation when looking at a far object c . Changes in peripheral vision in response to light d . Involuntary blinking in the presence of bright light ANS: A The muscle fibers of the iris contract the pupil in bright light and accommodate for near vision, which also results in pupil constriction. The other responses are not correct. DIF: Cognitive Level: Remembering (Knowledge) REF: dm. 296 MSC: Client Needs: Physiologic Integrity: Physiologic Adaptation 8. A patient has a normal pupillary light reflex. The nurse recognizes that this reflex indicates that: a . The eyes converge to focus on the light. b . Light is reflected at the same spot in both eyes. c . The eye focuses the image in the center of the pupil. d . Constriction of both pupils occurs in response to bright light. ANS: D The pupillary light reflex is the normal constriction of the pupils when bright light shines on the retina. The other responses are not correct. DIF: Cognitive Level: Understanding (Comprehension) REF: dm. 296 MSC: Client Needs: Physiologic Integrity: Physiologic Adaptation 9. A mother asks when her newborn infants eyesight will be developed. The nurse should reply: a . Vision is not totally developed until 2 years of age. b . Infants develop the ability to focus on an object at approximately 8 months of age. c . By approximately 3 months of age, infants develop more coordinated eye movements and can fixate on an object. d . Most infants have uncoordinated eye movements for the first year of life. ANS: C Eye movements may be poorly coordinated at birth, but by 3 to 4 months of age, the infant should establish binocularity and should be able to fixate simultaneously on a single image with both eyes. DIF: Cognitive Level: Applying (Application) REF: dm. 302 MSC: Client Needs: Health Promotion and Maintenance 10. The nurse is reviewing in age-related changes in the eye for a class. Which of these physiologic changes is responsible for presbyopia? a . Degeneration of the cornea b . Loss of lens elasticity c . Decreased adaptation to darkness d . Decreased distance vision abilities ANS: B The lens loses elasticity and decreases its ability to change shape to accommodate for near vision. This condition is called presbyopia. DIF: Cognitive Level: Understanding (Comprehension) REF: dm. 286 MSC: Client Needs: Health Promotion and Maintenance 11. Which of these assessment findings would the nurse expect to see when examining the eyes of a black patient? a . Increased night vision b . Dark retinal background c . Increased photosensitivity d . Narrowed palpebral fissures ANS: B An ethnically based variability in the color of the iris and in retinal pigmentation exists, with darker irides having darker retinas behind them. DIF: Cognitive Level: Understanding (Comprehension) REF: dm. 286 MSC: Client Needs: Safe and Effective Care Environment: Management of Care 12. A 52-year-old patient describes the presence of occasional floaters or spots moving in front of his eyes. The nurse should: a . Examine the retina to determine the number of floaters. b . Presume the patient has glaucoma and refer him for further testing. c . Consider these to be abnormal findings, and refer him to an ophthalmologist. d . Know that floaters are usually insignificant and are caused by condensed vitreous fibers. ANS: D Floaters are a common sensation with myopia or after middle age and are attributable to condensed vitreous fibers. Floaters or spots are not usually significant, but the acute onset of floaters may occur with retinal detachment. DIF: Cognitive Level: Analyzing (Analysis) REF: dm. 287 MSC: Client Needs: Health Promotion and Maintenance 13. The nurse is preparing to assess the visual acuity of a 16-year-old patient. How should the nurse proceed? a Perform the confrontation test. . b . Ask the patient to read the print on a handheld Jaeger card. c . Use the Snellen chart positioned 20 feet away from the patient. d . Determine the patients ability to read newsprint at a distance of 12 to 14 inches. ANS: C The Snellen alphabet chart is the most commonly used and most accurate measure of visual acuity. The confrontation test is a gross measure of peripheral vision. The Jaeger card or newspaper tests are used to test near vision. DIF: Cognitive Level: Analyzing (Analysis) REF: dm. 289 MSC: Client Needs: Safe and Effective Care Environment: Management of Care 14. A patients vision is recorded as 20/30 when the Snellen eye chart is used. The nurse interprets these results to indicate that: a At 30 feet the patient can read the entire chart. . b The patient can read at 20 feet what a person with normal vision can read at 30 . feet. c The patient can read the chart from 20 feet in the left eye and 30 feet in the right . eye. d The patient can read from 30 feet what a person with normal vision can read . from 20 feet. ANS: B The top number indicates the distance the person is standing from the chart; the denominator gives the distance at which a normal eye can see. DIF: Cognitive Level: Applying (Application) REF: dm. 290 MSC: Client Needs: Physiologic Integrity: Physiologic Adaptation 15. A patient is unable to read even the largest letters on the Snellen chart. The nurse should take which action next? a Refer the patient to an ophthalmologist or optometrist for further evaluation. . b Assess whether the patient can count the nurses fingers when they are placed in . front of his or her eyes. c Ask the patient to put on his or her reading glasses and attempt to read the . Snellen chart again. d Shorten the distance between the patient and the chart until the letters are seen, . and record that distance. ANS: D If the person is unable to see even the largest letters when standing 20 feet from the chart, then the nurse should shorten the distance to the chart until the letters are seen, and record that distance (e.g., 10/200). If visual acuity is even lower, then the nurse should assess whether the person can count fingers when they are spread in front of the eyes or can distinguish light perception from a penlight. If vision is poorer than 20/30, then a referral to an ophthalmologist or optometrist is necessary, but the nurse must first assess the visual acuity. DIF: Cognitive Level: Analyzing (Analysis) REF: dm. 290 MSC: Client Needs: Physiologic Integrity: Physiologic Adaptation 16. A patients vision is recorded as 20/80 in each eye. The nurse interprets this finding to mean that the patient: a Has poor vision. . b Has acute vision. . c Has normal vision. . d Is presbyopic. . ANS: A Normal visual acuity is 20/20 in each eye; the larger the denominator, the poorer the vision. DIF: Cognitive Level: Applying (Application) REF: dm. 290 MSC: Client Needs: Physiologic Integrity: Physiologic Adaptation 17. When performing the corneal light reflex assessment, the nurse notes that the light is reflected at 2 oclock in each eye. The nurse should: a . Consider this a normal finding. b . Refer the individual for further evaluation. c . Document this finding as an asymmetric light reflex. d Perform the confrontation test to validate the findings. . ANS: A Reflection of the light on the corneas should be in exactly the same spot on each eye, or symmetric. If asymmetry is noted, then the nurse should administer the cover test. DIF: Cognitive Level: Analyzing (Analysis) REF: dm. 292 MSC: Client Needs: Safe and Effective Care Environment: Management of Care 18. The nurse is performing the diagnostic positions test. Normal findings would be which of these results? a . Convergence of the eyes b . Parallel movement of both eyes c . Nystagmus in extreme superior gaze d . Slight amount of lid lag when moving the eyes from a superior to an inferior position ANS: B A normal response for the diagnostic positions test is parallel tracking of the object with both eyes. Eye movement that is not parallel indicates a weakness of an extraocular muscle or dysfunction of the CN that innervates it. DIF: Cognitive Level: Applying (Application) REF: dm. 292 MSC: Client Needs: Safe and Effective Care Environment: Management of Care 19. During an assessment of the sclera of a black patient, the nurse would consider which of these an expected finding? a . Yellow fatty deposits over the cornea b . Pallor near the outer canthus of the lower lid c . Yellow color of the sclera that extends up to the iris d . Presence of small brown macules on the sclera ANS: D Normally in dark-skinned people, small brown macules may be observed in the sclera. DIF: Cognitive Level: Applying (Application) REF: dm. 294 MSC: Client Needs: Safe and Effective Care Environment: Management of Care 20. A 60-year-old man is at the clinic for an eye examination. The nurse suspects that he has ptosis of one eye. How should the nurse check for this? a . Perform the confrontation test. b . Assess the individuals near vision. c . Observe the distance between the palpebral fissures. d . Perform the corneal light test, and look for symmetry of the light reflex. ANS: C Ptosis is a drooping of the upper eyelid that would be apparent by observing the distance between the upper and lower eyelids. The confrontation test measures peripheral vision. Measuring near vision or the corneal light test does not check for ptosis. Chapter 5 Advanced Health Assessment of Skin, Hair, and Nails 21. In performing an examination of a 3-year-old child with a suspected ear infection, the nurse would: a . Omit the otoscopic examination if the child has a fever. b . Pull the ear up and back before inserting the speculum. c . Ask the mother to leave the room while examining the child. d . Perform the otoscopic examination at the end of the assessment. ANS: D In addition to its place in the complete examination, eardrum assessment is mandatory for any infant or child requiring care for an illness or fever. For the infant or young child, the timing of the otoscopic examination is best toward the end of the complete examination. DIF: Cognitive Level: Analyzing (Analysis) REF: dm. 336 MSC: Client Needs: Safe and Effective Care Environment: Management of Care 22. The nurse is preparing to perform an otoscopic examination of a newborn infant. Which statement is true regarding this examination? a . Immobility of the drum is a normal finding. b . An injected membrane would indicate an infection. c . The normal membrane may appear thick and opaque. d . The appearance of the membrane is identical to that of an adult. ANS: C During the first few days after the birth, the tympanic membrane of a newborn often appears thickened and opaque. It may look injected and have a mild redness from increased vascularity. The other statements are not correct. DIF: Cognitive Level: Understanding (Comprehension) REF: dm. 337 MSC: Client Needs: Health Promotion and Maintenance 23. The nurse assesses the hearing of a 7-month-old by clapping hands. What is the expected response? The infant: a . Turns his or her head to localize the sound. b . Shows no obvious response to the noise. c . Shows a startle and acoustic blink reflex. d . Stops any movement, and appears to listen for the sound. ANS: A With a loud sudden noise, the nurse should notice the infant turning his or her head to localize the sound and to respond to his or her own name. A startle reflex and acoustic blink reflex is expected in newborns; at age 3 to 4 months, the infant stops any movement and appears to listen. DIF: Cognitive Level: Understanding (Comprehension) REF: dm. 338 MSC: Client Needs: Health Promotion and Maintenance 24. The nurse is performing an ear examination of an 80-year-old patient. Which of these findings would be considered normal? a . High-tone frequency loss b . Increased elasticity of the pinna c . Thin, translucent membrane d . Shiny, pink tympanic membrane ANS: A A high-tone frequency hearing loss is apparent for those affected with presbycusis, the hearing loss that occurs with aging. The pinna loses elasticity, causing earlobes to be pendulous. The eardrum may be whiter in color and more opaque and duller in the older person than in the younger adult. DIF: Cognitive Level: Applying (Application) REF: dm. 339 MSC: Client Needs: Health Promotion and Maintenance 25. An assessment of a 23-year-old patient reveals the following: an auricle that is tender and reddish-blue in color with small vesicles. The nurse would need to know additional information that includes which of these? a . Any change in the ability to hear b . Any recent drainage from the ear c . Recent history of trauma to the ear d . Any prolonged exposure to extreme cold ANS: D Frostbite causes reddish-blue discoloration and swelling of the auricle after exposure to extreme cold. Vesicles or bullae may develop, and the person feels pain and tenderness. DIF: Cognitive Level: Analyzing (Analysis) REF: dm. 342 MSC: Client Needs: Physiologic Integrity: Physiologic Adaptation 26. While performing the otoscopic examination of a 3-year-old boy who has been pulling on his left ear, the nurse finds that his left tympanic membrane is bright red and that the light reflex is not visible. The nurse interprets these findings to indicate a(n): a . Fungal infection. b . Acute otitis media. c . Perforation of the eardrum. d . Cholesteatoma. ANS: B Absent or distorted light reflex and a bright red color of the eardrum are indicative of acute otitis media. (See Table 15-5 for descriptions of the other conditions.) DIF: Cognitive Level: Analyzing (Analysis) REF: dm. 348 MSC: Client Needs: Physiologic Integrity: Physiologic Adaptation 27. The mother of a 2-year-old toddler is concerned about the upcoming placement of tympanostomy tubes in her sons ears. The nurse would include which of these statements in the teaching plan? a . The tubes are placed in the inner ear. b . The tubes are used in children with sensorineural loss. c . The tubes are permanently inserted during a surgical procedure. d . The purpose of the tubes is to decrease the pressure and allow for drainage. ANS: D Polyethylene tubes are surgically inserted into the eardrum to relieve middle ear pressure and to promote drainage of chronic or recurrent middle ear infections. Tubes spontaneously extrude in 6 months to 1 year. DIF: Cognitive Level: Understanding (Comprehension) REF: dm. 348 MSC: Client Needs: Physiologic Integrity: Physiologic Adaptation 28. In an individual with otitis externa, which of these signs would the nurse expect to find on assessment? a Rhinorrhea . b . Periorbital edema c . Pain over the maxillary sinuses d . Enlarged superficial cervical nodes ANS: D The lymphatic drainage of the external ear flows to the parotid, mastoid, and superficial cervical nodes. The signs are severe swelling of the canal, inflammation, and tenderness. Rhinorrhea, periorbital edema, and pain over the maxillary sinuses do not occur with otitis externa. DIF: Cognitive Level: Understanding (Comprehension) REF: dm. 342 MSC: Client Needs: Physiologic Integrity: Physiologic Adaptation 29. When performing an otoscopic examination of a 5-year-old child with a history of chronic ear infections, the nurse sees that his right tympanic membrane is amber-yellow in color and that air bubbles are visible behind the tympanic membrane. The child reports occasional hearing loss and a popping sound with swallowing. The preliminary analysis based on this information is that the child: a Most likely has serous otitis media. . b Has an acute purulent otitis media. . c Has evidence of a resolving cholesteatoma. . d Is experiencing the early stages of perforation. . ANS: A An amber-yellow color to the tympanic membrane suggests serum or pus in the middle ear. Air or fluid or bubbles behind the tympanic membrane are often visible. The patient may have feelings of fullness, transient hearing loss, and a popping sound with swallowing. These findings most likely suggest that the child has serous otitis media. The other responses are not correct. DIF: Cognitive Level: Analyzing (Analysis) REF: dm. 347 MSC: Client Needs: Physiologic Integrity: Physiologic Adaptation 30. The nurse is performing an assessment on a 65-year-old man. He reports a crusty nodule behind the pinna. It intermittently bleeds and has not healed over the past 6 months. On physical assessment, the nurse finds an ulcerated crusted nodule with an indurated base. The preliminary analysis in this situation is that this: a Is most likely a benign sebaceous cyst. . b Is most likely a keloid. . c Could be a potential carcinoma, and the patient should be referred for a biopsy. . d Is a tophus, which is common in the older adult and is a sign of gout. . ANS: C An ulcerated crusted nodule with an indurated base that fails to heal is characteristic of a carcinoma. These lesions fail to heal and intermittently bleed. Individuals with such symptoms should be referred for a biopsy (see Table 15-2). The other responses are not correct. DIF: Cognitive Level: Analyzing (Analysis) REF: dm. 344 MSC: Client Needs: Physiologic Integrity: Physiologic Adaptation 31. The nurse suspects that a patient has otitis media. Early signs of otitis media include which of these findings of the tympanic membrane? a Red and bulging . b Hypomobility . c Retraction with landmarks clearly visible . d Flat, slightly pulled in at the center, and moves with insufflation . ANS: B An early sign of otitis media is hypomobility of the tympanic membrane. As pressure increases, the tympanic membrane begins to bulge. DIF: Cognitive Level: Remembering (Knowledge) REF: dm. 348 MSC: Client Needs: Safe and Effective Care Environment: Management of Care 32. The nurse is performing a middle ear assessment on a 15-year-old patient who has had a history of chronic ear infections. When examining the right tympanic membrane, the nurse sees the presence of dense white patches. The tympanic membrane is otherwise unremarkable. It is pearly, with the light reflex at 5 oclock and landmarks visible. The nurse should: a Refer the patient for the possibility of a fungal infection. . b Know that these are scars caused from frequent ear infections. . c Consider that these findings may represent the presence of blood in the middle . ear. d Be concerned about the ability to hear because of this abnormality on the . tympanic membrane. ANS: B Dense white patches on the tympanic membrane are sequelae of repeated ear infections. They do not necessarily affect hearing. DIF: Cognitive Level: Analyzing (Analysis) REF: dm. 349 MSC: Client Needs: Safe and Effective Care Environment: Management of Care 33. The nurse is preparing to do an otoscopic examination on a 2-year-old child. Which one of these reflects the correct procedure? a Pulling the pinna down . b Pulling the pinna up and back . c Slightly tilting the childs head toward the examiner . d Instructing the child to touch his chin to his chest . ANS: A For an otoscopic examination on an infant or on a child under 3 years of age, the pinna is pulled down. The other responses are not part of the correct procedure. DIF: Cognitive Level: Applying (Application) REF: dm. 337 MSC: Client Needs: Health Promotion and Maintenance 34. The nurse is conducting a child safety class for new mothers. Which factor places young children at risk for ear infections? a Family history . b Air conditioning . c Excessive cerumen . d Passive cigarette smoke . ANS: D Exposure to passive and gestational smoke is a risk factor for ear infections in infants and children. DIF: Cognitive Level: Understanding (Comprehension) REF: dm. 331 MSC: Client Needs: Physiologic Integrity: Reduction of Risk Potential 35. During an otoscopic examination, the nurse notices an area of black and white dots on the tympanic membrane and the ear canal wall. What does this finding suggest? a Malignancy . b Viral infection . c Blood in the middle ear . d Yeast or fungal infection . ANS: D A colony of black or white dots on the drum or canal wall suggests a yeast or fungal infection (otomycosis). DIF: Cognitive Level: Understanding (Comprehension) REF: dm. 349 MSC: Client Needs: Physiologic Integrity: Basic Care and Comfort 36. A 17-year-old student is a swimmer on her high schools swim team. She has had three bouts of otitis externa this season and wants to know what to do to prevent it. The nurse instructs her to: a . Use a cotton-tipped swab to dry the ear canals thoroughly after each swim. b . Use rubbing alcohol or 2% acetic acid eardrops after every swim. c . Irrigate the ears with warm water and a bulb syringe after each swim. d Rinse the ears with a warmed solution of mineral oil and hydrogen peroxide. . ANS: B With otitis externa (swimmers ear), swimming causes the external canal to become waterlogged and swell; skinfolds are set up for infection. Otitis externa can be prevented by using rubbing alcohol or 2% acetic acid eardrops after every swim. DIF: Cognitive Level: Analyzing (Analysis) REF: dm. 342 MSC: Client Needs: Health Promotion and Maintenance 37. During an examination, the patient states he is hearing a buzzing sound and says that it is driving me crazy! The nurse recognizes that this symptom indicates: a Vertigo. . b Pruritus. . c Tinnitus. . d Cholesteatoma. . ANS: C Tinnitus is a sound that comes from within a person; it can be a ringing, crackling, or buzzing sound. It accompanies some hearing or ear disorders. DIF: Cognitive Level: Understanding (Comprehension) REF: dm. 330 MSC: Client Needs: Physiologic Integrity: Basic Care and Comfort 38. During an examination, the nurse notices that the patient stumbles a little while walking, and, when she sits down, she holds on to the sides of the chair. The patient states, It feels like the room is spinning! The nurse notices that the patient is experiencing: a . Objective vertigo. b . Subjective vertigo. c . Tinnitus. d . Dizziness. ANS: A With objective vertigo, the patient feels like the room spins; with subjective vertigo, the person feels like he or she is spinning. Tinnitus is a sound that comes from within a person; it can be a ringing, crackling, or buzzing sound. It accompanies some hearing or ear disorders. Dizziness is not the same as true vertigo; the person who is dizzy may feel unsteady and lightheaded. DIF: Cognitive Level: Applying (Application) REF: dm. 331 MSC: Client Needs: Physiologic Integrity: Physiologic Adaptation 39. A patient has been admitted after an accident at work. During the assessment, the patient is having trouble hearing and states, I dont know what the matter is. All of a sudden, I cant hear you out of my left ear! What should the nurse do next? a . Make note of this finding for the report to the next shift. b . Prepare to remove cerumen from the patients ear. c . Notify the patients health care provider. d . Irrigate the ear with rubbing alcohol. ANS: C Any sudden loss of hearing in one or both ears that is not associated with an upper respiratory infection needs to be reported at once to the patients health care provider. Hearing loss associated with trauma is often sudden. Irrigating the ear or removing cerumen is not appropriate at this time. 40. The nurse is testing the hearing of a 78-year-old man and is reminded of the changes in hearing that occur with aging that include which of the following? Select all that apply. a . Hearing loss related to aging begins in the mid 40s. b . Progression of hearing loss is slow. c . The aging person has low-frequency tone loss. d . The aging person may find it harder to hear consonants than vowels. e . Sounds may be garbled and difficult to localize. f Hearing loss reflects nerve degeneration of the middle ear. . ANS: B, D, E Presbycusis is a type of hearing loss that occurs with aging and is found in 60% of those older than 65 years. It is a gradual sensorineural loss caused by nerve degeneration in the inner ear or auditory nerve, and it slowly progresses after the age of 50 years. The person first notices a high- frequency tone loss; it is harder to hear consonants (high-pitched components of speech) than vowels, which makes words sound garbled. The ability to localize sound is also impaired. Chapter 6. Advanced Health Assessment of the Cardiovascular System MULTIPLE CHOICE 1. The sac that surrounds and protects the heart is called the: a . Pericardium. b . Myocardium. c . Endocardium. d . Pleural space. ANS: A The pericardium is a tough, fibrous double-walled sac that surrounds and protects the heart. It has two layers that contain a few milliliters of serous pericardial fluid. DIF: Cognitive Level: Remembering (Knowledge) REF: dm. 460 MSC: Client Needs: General 2. The direction of blood flow through the heart is best described by which of these? a . Vena cava right atrium right ventricle lungs pulmonary artery left atrium left ventricle b . Right atrium right ventricle pulmonary artery lungs pulmonary vein left atrium left ventricle c . Aorta right atrium right ventricle lungs pulmonary vein left atrium left ventricle vena cava d . Right atrium right ventricle pulmonary vein lungs pulmonary artery left atrium left ventricle ANS: B Returning blood from the body empties into the right atrium and flows into the right ventricle and then goes to the lungs through the pulmonary artery. The lungs oxygenate the blood, and it is then returned to the left atrium through the pulmonary vein. The blood goes from there to the left ventricle and then out to the body through the aorta. DIF: Cognitive Level: Remembering (Knowledge) REF: dm. 461|dm. 463 MSC: Client Needs: General 3. The nurse is reviewing the anatomy and physiologic functioning of the heart. Which statement best describes what is meant by atrial kick? a . The atria contract during systole and attempt to push against closed valves. b . Contraction of the atria at the beginning of diastole can be felt as a palpitation. c . Atrial kick is the pressure exerted against the atria as the ventricles contract during systole. d . The atria contract toward the end of diastole and push the remaining blood into the ventricles. ANS: D Toward the end of diastole, the atria contract and push the last amount of blood (approximately 25% of stroke volume) into the ventricles. This active filling phase is called presystole, or atrial systole, or sometimes theatrial kick. DIF: Cognitive Level: Remembering (Knowledge) REF: dm. 463 MSC: Client Needs: General 4. When listening to heart sounds, the nurse knows the valve closures that can be heard best at the base of the heart are: a . Mitral and tricuspid. b . Tricuspid and aortic. c . Aortic and pulmonic. d . Mitral and pulmonic. ANS: C The second heart sound (S2) occurs with the closure of the semilunar (aortic and pulmonic) valves and signals the end of systole. Although it is heard over all the precordium, the S2 is loudest at the base of the heart. DIF: Cognitive Level: Understanding (Comprehension) REF: dm. 464 MSC: Client Needs: General 5. Which of these statements describes the closure of the valves in a normal cardiac cycle? a . The aortic valve closes slightly before the tricuspid valve. b . The pulmonic valve closes slightly before the aortic valve. c . The tricuspid valve closes slightly later than the mitral valve. d . Both the tricuspid and pulmonic valves close at the same time. ANS: C Events occur just slightly later in the right side of the heart because of the route of myocardial depolarization. As a result, two distinct components to each of the heart sounds exist, and sometimes they can be heard separately. In the first heart sound, the mitral component (M1) closes just before the tricuspid component (T1). DIF: Cognitive Level: Understanding (Comprehension) REF: dm. 463 MSC: Client Needs: General 6. The component of the conduction system referred to as the pacemaker of the heart is the: a . Atrioventricular (AV) node. b . Sinoatrial (SA) node. c . Bundle of His. d . Bundle branches. ANS: B Specialized cells in the SA node near the superior vena cava initiate an electrical impulse. Because the SA node has an intrinsic rhythm, it is called the pacemaker of the heart. DIF: Cognitive Level: Remembering (Knowledge) REF: dm. 464 MSC: Client Needs: General 7. The electrical stimulus of the cardiac cycle follows which sequence? a . AV node SA node bundle of His b . Bundle of His AV node SA node c . SA node AV node bundle of His bundle branches d . AV node SA node bundle of His bundle branches ANS: D Specialized cells in the SA node near the superior vena cava initiate an electrical impulse. The current flows in an orderly sequence, first across the atria to the AV node low in the atrial septum. There it is delayed slightly, allowing the atria the time to contract before the ventricles are stimulated. Then the impulse travels to the bundle of His, the right and left bundle branches, and then through the ventricles. DIF: Cognitive Level: Understanding (Comprehension) REF: dm. 464 MSC: Client Needs: General 8. The findings from an assessment of a 70-year-old patient with swelling in his ankles include jugular venous pulsations 5 cm above the sternal angle when the head of his bed is elevated 45 degrees. The nurse knows that this finding indicates: a . Decreased fluid volume. b . Increased cardiac output. c . Narrowing of jugular veins. d . Elevated pressure related to heart failure. ANS: D Because no cardiac valve exists to separate the superior vena cava from the right atrium, the jugular veins give information about the activity on the right side of the heart. They reflect filling pressures and volume changes. Normal jugular venous pulsation is 2 cm or less above the sternal angle. Elevated pressure is more than 3 cm above the sternal angle at 45 degrees and occurs with heart failure. DIF: Cognitive Level: Analyzing (Analysis) REF: pp. 466-467 MSC: Client Needs: Physiologic Integrity: Physiologic Adaptation 9. When assessing a newborn infant who is 5 minutes old, the nurse knows which of these statements to be true? a . The left ventricle is larger and weighs more than the right ventricle. b . The circulation of a newborn is identical to that of an adult. c . Blood can flow into the left side of the heart through an opening in the atrial septum. d . The foramen ovale closes just minutes before birth, and the ductus arteriosus closes immediately after. ANS: C First, approximately two thirds of the blood is shunted through an opening in the atrial septum, the foramen ovale, into the left side of the heart, where it is pumped out through the aorta. The foramen ovale closes within the first hour after birth because the pressure in the right side of the heart is now lower than in the left side. DIF: Cognitive Level: Applying (Application) REF: dm. 467 MSC: Client Needs: Health Promotion and Maintenance 10. A 25-year-old woman in her fifth month of pregnancy has a blood pressure of 100/70 mm Hg. In reviewing her previous examination, the nurse notes that her blood pressure in her second month was 124/80 mm Hg. In evaluating this change, what does the nurse know to be true? a . This decline in blood pressure is the result of peripheral vasodilatation and is an expected change. b . Because of increased cardiac output, the blood pressure should be higher at this time. c . This change in blood pressure is not an expected finding because it means a decrease in cardiac output. d . This decline in blood pressure means a decrease in circulating blood volume, which is dangerous for the fetus. ANS: A Despite the increased cardiac output, arterial blood pressure decreases in pregnancy because of peripheral vasodilatation. The blood pressure drops to its lowest point during the second trimester and then rises after that. DIF: Cognitive Level: Analyzing (Analysis) REF: dm. 467 MSC: Client Needs: Physiologic Integrity: Physiologic Adaptation 11. In assessing a 70-year-old man, the nurse finds the following: blood pressure 140/100 mm Hg; heart rate 104 beats per minute and slightly irregular; and the split S2 heart sound. Which of these findings can be explained by expected hemodynamic changes related to age? a . Increase in resting heart rate b . Increase in systolic blood pressure c . Decrease in diastolic blood pressure d . Increase in diastolic blood pressure ANS: B With aging, an increase in systolic blood pressure occurs. No significant change in diastolic pressure and no change in the resting heart rate occur with aging. Cardiac output at rest is does not changed with aging. DIF: Cognitive Level: Analyzing (Analysis) REF: dm. 469 MSC: Client Needs: Health Promotion and Maintenance 12. A 45-year-old man is in the clinic for a routine physical examination. During the recording of his health history, the patient states that he has been having difficulty sleeping. Ill be sleeping great, and then I wake up and feel like I cant get my breath. The nurses best response to this would be: a . When was your last electrocardiogram? b . Its probably because its been so hot at night. c . Do you have any history of problems with your heart? d . Have you had a recent sinus infection or upper respiratory infection? ANS: C Paroxysmal nocturnal dyspnea (shortness of breath generally occurring at night) occurs with heart failure. Lying down increases the volume of intrathoracic blood, and the weakened heart cannot accommodate the increased load. Classically, the person awakens after 2 hours of sleep, arises, and flings open a window with the perception of needing fresh air. DIF: Cognitive Level: Applying (Application) REF: dm. 472 MSC: Client Needs: Physiologic Integrity: Physiologic Adaptation 13. In assessing a patients major risk factors for heart disease, which would the nurse want to include when taking a history? a . Family history, hypertension, stress, and age b . Personality type, high cholesterol, diabetes, and smoking c . Smoking, hypertension, obesity, diabetes, and high cholesterol d . Alcohol consumption, obesity, diabetes, stress, and high cholesterol ANS: C To assess for major risk factors of coronary artery disease, the nurse should collect data regarding elevated serum cholesterol, elevated blood pressure, blood glucose levels above 100 mg/dL or known diabetes mellitus, obesity, any length of hormone replacement therapy for post menopausal women, cigarette smoking, and low activity level. DIF: Cognitive Level: Applying (Application) REF: dm. 473 MSC: Client Needs: Physiologic Integrity: Reduction of Risk Potential 14. The mother of a 3-month-old infant states that her baby has not been gaining weight. With further questioning, the nurse finds that the infant falls asleep after nursing and wakes up after a short time, hungry again. What other information would the nurse want to have? a . Infants sleeping position b . Sibling history of eating disorders c . Amount of background noise when eating d . Presence of dyspnea or diaphoresis when sucking ANS: D To screen for heart disease in an infant, the focus should be on feeding. Fatigue during feeding should be noted. An infant with heart failure takes fewer ounces each feeding, becomes dyspneic with sucking, may be diaphoretic, and then falls into exhausted sleep and awakens after a short time hungry again. DIF: Cognitive Level: Analyzing (Analysis) REF: dm. 473 MSC: Client Needs: Physiologic Integrity: Physiologic Adaptation 15. In assessing the carotid arteries of an older patient with cardiovascular disease, the nurse would: a . Palpate the artery in the upper one third of the neck. b . Listen with the bell of the stethoscope to assess for bruits. c . Simultaneously palpate both arteries to compare amplitude. d . Instruct the patient to take slow deep breaths during auscultation. ANS: B If cardiovascular disease is suspected, then the nurse should auscultate each carotid artery for the presence of a bruit. The nurse should avoid compressing the artery, which could create an artificial bruit and compromise circulation if the carotid artery is already narrowed by atherosclerosis. Excessive pressure on the carotid sinus area high in the neck should be avoided, and excessive vagal stimulation could slow down the heart rate, especially in older adults. Palpating only one carotid artery at a time will avoid compromising arterial blood to the brain. DIF: Cognitive Level: Applying (Application) REF: dm. 476 MSC: Client Needs: Safe and Effective Care Environment: Management of Care 16. During an assessment of a 68-year-old man with a recent onset of right-sided weakness, the nurse hears a blowing, swishing sound with the bell of the stethoscope over the left carotid artery. This finding would indicate: a . Valvular disorder. b . Blood flow turbulence. c . Fluid volume overload. d . Ventricular hypertrophy. ANS: B A bruit is a blowing, swishing sound indicating blood flow turbulence; normally, none is present. DIF: Cognitive Level: Analyzing (Analysis) REF: dm. 476 MSC: Client Needs: Safe and Effective Care Environment: Management of Care 17. During an inspection of the precordium of an adult patient, the nurse notices the chest moving in a forceful manner along the sternal border. This finding most likely suggests a(n): a . Normal heart. b . Systolic murmur. c . Enlargement of the left ventricle. d . Enlargement of the right ventricle. ANS: D Normally, the examiner may or may not see an apical impulse; when visible, it occupies the fourth or fifth intercostal space at or inside the midclavicular line. A heave or lift is a sustained forceful thrusting of the ventricle during systole. It occurs with ventricular hypertrophy as a result of increased workload. A right ventricular heave is seen at the sternal border; a left ventricular heave is seen at the apex. DIF: Cognitive Level: Applying (Application) REF: dm. 479 MSC: Client Needs: Safe and Effective Care Environment: Management of Care 18. During an assessment of a healthy adult, where would the nurse expect to palpate the apical impulse? a . Third left intercostal space at the midclavicular line b . Fourth left intercostal space at the sternal border c . Fourth left intercostal space at the anterior axillary line d . Fifth left intercostal space at the midclavicular line ANS: D The apical impulse should occupy only one intercostal space, the fourth or fifth, and it should be at or medial to the midclavicular line. DIF: Cognitive Level: Remembering (Knowledge) REF: dm. 479 MSC: Client Needs: Safe and Effective Care Environment: Management of Care 19. The nurse is examining a patient who has possible cardiac enlargement. Which statement about percussion of the heart is true? a . Percussion is a useful tool for outlining the hearts borders. b . Percussion is easier in patients who are obese. c . Studies show that percussed cardiac borders do not correlate well with the true cardiac border. d . Only expert health care providers should attempt percussion of the heart. ANS: C Numerous comparison studies have shown that the percussed cardiac border correlates only moderately with the true cardiac border. Percussion is of limited usefulness with the female breast tissue, in a person who is obese, or in a person with a muscular chest wall. Chest x-ray images or echocardiographic examinations are significantly more accurate in detecting heart enlargement. DIF: Cognitive Level: Understanding (Comprehension) REF: dm. 480 MSC: Client Needs: Safe and Effective Care Environment: Management of Care 20. The nurse is preparing to auscultate for heart sounds. Which technique is correct? a . Listening to the sounds at the aortic, tricuspid, pulmonic, and mitral areas b . Listening by inching the stethoscope in a rough Z pattern, from the base of the heart across and down, then over to the apex c . Listening to the sounds only at the site where the apical pulse is felt to be the strongest d . Listening for all possible sounds at a time at each specified area ANS: B Auscultation of breath sounds should not be limited to only four locations. Sounds produced by the valves may be heard all over the precordium. The stethoscope should be inched in a rough Z pattern from the base of the heart across and down, then over to the apex; or, starting at the apex, it should be slowly worked up (see Figure 19-22). Listening selectively to one sound at a time is best. DIF: Cognitive Level: Understanding (Comprehension) REF: dm. 481 MSC: Client Needs: Safe and Effective Care Environment: Management of Care 21. While counting the apical pulse on a 16-year-old patient, the nurse notices an irregular rhythm. His rate speeds up on inspiration and slows on expiration. What would be the nurses response? a . Talk with the patient about his intake of caffeine. b . Perform an electrocardiogram after the examination. c . No further response is needed because sinus arrhythmia can occur normally. d . Refer the patient to a cardiologist for further testing. ANS: C The rhythm should be regular, although sinus arrhythmia occurs normally in young adults and children. With sinus arrhythmia, the rhythm varies with the persons breathing, increasing at the peak of inspiration and slowing with expiration. DIF: Cognitive Level: Analyzing (Analysis) REF: dm. 481 MSC: Client Needs: Health Promotion and Maintenance 22. When listening to heart sounds, the nurse knows that the S1: a . Is louder than the S2 at the base of the heart. b . Indicates the beginning of diastole. c . Coincides with the carotid artery pulse. d . Is caused by the closure of the semilunar valves. ANS: C The S1 coincides with the carotid artery pulse, is the start of systole, and is louder than the S2 at the apex of the heart; the S2 is louder than the S1 at the base. The nurse should gently feel the carotid artery pulse while auscultating at the apex; the sound heard as each pulse is felt is the S1. DIF: Cognitive Level: Understanding (Comprehension) REF: dm. 482 MSC: Client Needs: General 23. During the cardiac auscultation, the nurse hears a sound immediately occurring after the S2 at the second left intercostal space. To further assess this sound, what should the nurse do? a . Have the patient turn to the left side while the nurse listens with the bell of the stethoscope. b . Ask the patient to hold his or her breath while the nurse listens again. c . No further assessment is needed because the nurse knows this sound is an S3. d . Watch the patients respirations while listening for the effect on the sound. ANS: D A split S2 is a normal phenomenon that occurs toward the end of inspiration in some people. A split S2 is heard only in the pulmonic valve area, the second left interspace. When the split S2 is first heard, the nurse should not be tempted to ask the person to hold his or her breath so that the nurse can concentrate on the sounds. Breath holding will only equalize ejection times in the right and left sides of the heart and cause the split to go away. Rather, the nurse should concentrate on the split while watching the persons chest rise up and down with breathing. DIF: Cognitive Level: Analyzing (Analysis) REF: dm. 483 MSC: Client Needs: Safe and Effective Care Environment: Management of Care 24. Which of these findings would the nurse expect to notice during a cardiac assessment on a 4- year-old child? a . S3 when sitting up b . Persistent tachycardia above 150 beats per minute c . Murmur at the second left intercostal space when supine d . Palpable apical impulse in the fifth left intercostal space lateral to midclavicular line ANS: C Some murmurs are common in healthy children or adolescents and are termed innocent or functional. The innocent murmur is heard at the second or third left intercostal space and disappears with sitting, and the young person has no associated signs of cardiac dysfunction. DIF: Cognitive Level: Applying (Application) REF: dm. 488 MSC: Client Needs: Health Promotion and Maintenance 25. While auscultating heart sounds on a 7-year-old child for a routine physical examination, the nurse hears an S3, a soft murmur at the left midsternal border, and a venous hum when the child is standing. What would be a correct interpretation of these findings? a . S3 is indicative of heart disease in children. b . These findings can all be normal in a child. c . These findings are indicative of congenital problems. d . The venous hum most likely indicates an aneurysm. ANS: B A physiologic S3 is common in children. A venous hum, caused by turbulence of blood flow in the jugular venous system, is common in healthy children and has no pathologic significance. Heart murmurs that are innocent (or functional) in origin are very common through childhood. DIF: Cognitive Level: Analyzing (Analysis) REF: dm. 488 MSC: Client Needs: Health Promotion and Maintenance 26. During the precordial assessment on an patient who is 8 months pregnant, the nurse palpates the apical impulse at the fourth left intercostal space lateral to the midclavicular line. This finding would indicate: a . Right ventricular hypertrophy. b . Increased volume and size of the heart as a result of pregnancy. c . Displacement of the heart from elevation of the diaphragm. d . Increased blood flow through the internal mammary artery. ANS: C Palpation of the apical impulse is higher and more lateral, compared with the normal position, because the enlarging uterus elevates the diaphragm and displaces the heart up and to the left and rotates it on its long axis. DIF: Cognitive Level: Analyzing (Analysis) REF: dm. 488 MSC: Client Needs: Health Promotion and Maintenance 27. In assessing for an S4 heart sound with a stethoscope, the nurse would listen with the: a Bell of the stethoscope at the base with the patient leaning forward. . b . Bell of the stethoscope at the apex with the patient in the left lateral position. c . Diaphragm of the stethoscope in the aortic area with the patient sitting. d . Diaphragm of the stethoscope in the pulmonic area with the patient supine. ANS: B The S4 is a ventricular filling sound that occurs when the atria contract late in diastole and is heard immediately before the S1. The S4 is a very soft sound with a very low pitch. The nurse needs a good bell and must listen for this sound. An S4 is heard best at the apex, with the person in the left lateral position. DIF: Cognitive Level: Understanding (Comprehension) REF: dm. 485 MSC: Client Needs: Safe and Effective Care Environment: Management of Care 28. A 70-year-old patient with a history of hypertension has a blood pressure of 180/100 mm Hg and a heart rate of 90 beats per minute. The nurse hears an extra heart sound at the apex immediately before the S1. The sound is heard only with the bell of the stethoscope while the patient is in the left lateral position. With these findings and the patients history, the nurse knows that this extra heart sound is most likely a(n): a Split S1. . b Atrial gallop. . c Diastolic murmur. . d Summation sound. . ANS: B A pathologic S4 is termed an atrial gallop or an S4 gallop. It occurs with decreased compliance of the ventricle and with systolic overload (afterload), including outflow obstruction to the ventricle (aortic stenosis) and systemic hypertension. A left-sided S4 occurs with these conditions and is heard best at the apex with the patient in the left lateral position. DIF: Cognitive Level: Analyzing (Analysis) REF: dm. 482 MSC: Client Needs: Health Promotion and Maintenance 29. The nurse is performing a cardiac assessment on a 65-year-old patient 3 days after her myocardial infarction (MI). Heart sounds are normal when she is supine, but when she is sitting and leaning forward, the nurse hears a high-pitched, scratchy sound with the diaphragm of the stethoscope at the apex. It disappears on inspiration. The nurse suspects: a Increased cardiac output. . b Another MI. . c Inflammation of the precordium. . d Ventricular hypertrophy resulting from muscle damage. . ANS: C Inflammation of the precordium gives rise to a friction rub. The sound is high pitched and scratchy, similar to sandpaper being rubbed. A friction rub is best heard with the diaphragm of the stethoscope, with the person sitting up and leaning forward, and with the breath held in expiration. A friction rub can be heard any place on the precordium. Usually, however, the sound is best heard at the apex and left lower sternal border, which are places where the pericardium comes in close contact with the chest wall. DIF: Cognitive Level: Analyzing (Analysis) REF: dm. 500 MSC: Client Needs: Physiologic Integrity: Physiologic Adaptation 30. The mother of a 10-month-old infant tells the nurse that she has noticed that her son becomes blue when he is crying and that the frequency of this is increasing. He is also not crawling yet. During the examination the nurse palpates a thrill at the left lower sternal border and auscultates a loud systolic murmur in the same area. What would be the most likely cause of these findings? a Tetralogy of Fallot . b Atrial septal defect . c Patent ductus arteriosus . d Ventricular septal defect . ANS: A The cause of these findings is tetralogy of Fallot. Its subjective findings include: (1) severe cyanosis, not in the first months of life but developing as the infant grows, and right ventricle outflow (i.e., pulmonic) stenosis that gets worse; (2) cyanosis with crying and exertion at first and then at rest; and (3) slowed development. Its objective findings include: (1) thrill palpable at the left lower sternal border; (2) the S1 is normal, the S2 has a loud A2, and the P2 is diminished or absent; and (3) the murmur is systolic, loud, and crescendo-decrescendo. DIF: Cognitive Level: Analyzing (Analysis) REF: dm. 503 MSC: Client Needs: Health Promotion and Maintenance 31. A 30-year-old woman with a history of mitral valve problems states that she has been very tired. She has started waking up at night and feels like her heart is pounding. During the assessment, the nurse palpates a thrill and lift at the fifth left intercostal space midclavicular line. In the same area, the nurse also auscultates a blowing, swishing sound right after the S1. These findings would be most consistent with: a Heart failure. . b Aortic stenosis. . c Pulmonary edema. . d Mitral regurgitation. . ANS: D These findings are consistent with mitral regurgitation. Its subjective findings include fatigue, palpitation, and orthopnea, and its objective findings are: (1) a thrill in systole at the apex; (2) a lift at the apex; (3) the apical impulse displaced down and to the left; (4) the S1 is diminished, the S2 is accentuated, and the S3 at the apex is often present; and (5) a pansystolic murmur that is often loud, blowing, best heard at the apex, and radiating well to the left axilla. DIF: Cognitive Level: Analyzing (Analysis) REF: dm. 505 MSC: Client Needs: Physiologic Integrity: Physiologic Adaptation 32. During a cardiac assessment on a 38-year-old patient in the hospital for chest pain, the nurse finds the following: jugular vein pulsations 4 cm above the sternal angle when the patient is elevated at 45 degrees, blood pressure 98/60 mm Hg, heart rate 130 beats per minute, ankle edema, difficulty breathing when supine, and an S3 on auscultation. Which of these conditions best explains the cause of these findings? a Fluid overload . b Atrial septal defect . c MI . d Heart failure . ANS: D Heart failure causes decreased cardiac output when the heart fails as a pump and the circulation becomes backed up and congested. Signs and symptoms include dyspnea, orthopnea, paroxysmal nocturnal dyspnea, decreased blood pressure, dependent and pitting edema; anxiety; confusion; jugular vein distention; and fatigue. The S3 is associated with heart failure and is always abnormal after 35 years of age. The S3 may be the earliest sign of heart failure. DIF: Cognitive Level: Analyzing (Analysis) REF: dm. 495 MSC: Client Needs: Physiologic Integrity: Physiologic Adaptation 33. The nurse knows that normal splitting of the S2 is associated with: a Expiration. . b Inspiration. . c Exercise state. . d Low resting heart rate. . ANS: B Normal or physiologic splitting of the S2 is associated with inspiration because of the increased blood return to the right side of the heart, delaying closure of the pulmonic valve. DIF: Cognitive Level: Understanding (Comprehension) REF: dm. 483 MSC: Client Needs: Safe and Effective Care Environment: Management of Care 34. During a cardiovascular assessment, the nurse knows that a thrill is: a Vibration that is palpable. . b Palpated in the right epigastric area. . c Associated with ventricular hypertrophy. . d Murmur auscultated at the third intercostal space. . ANS: A A thrill is a palpable vibration that signifies turbulent blood flow and accompanies loud murmurs. The absence of a thrill does not rule out the presence of a murmur. DIF: Cognitive Level: Remembering (Knowledge) REF: dm. 480 MSC: Client Needs: Safe and Effective Care Environment: Management of Care 35. During a cardiovascular assessment, the nurse knows that an S4 heart sound is: a Heard at the onset of atrial diastole. . b Usually a normal finding in the older adult. . c Heard at the end of ventricular diastole. . d Heard best over the second left intercostal space with the individual sitting . upright. ANS: C An S4 heart sound is heard at the end of diastole when the atria contract (atrial systole) and when the ventricles are resistant to filling. The S4 occurs just before the S1. DIF: Cognitive Level: Understanding (Comprehension) REF: dm. 500 MSC: Client Needs: Safe and Effective Care Environment: Management of Care 36. During an assessment, the nurse notes that the patients apical impulse is laterally displaced and is palpable over a wide area. This finding indicates: a Systemic hypertension. . b Pulmonic hypertension. . c Pressure overload, as in aortic stenosis. . d Volume overload, as in heart failure. . ANS: D With volume overload, as in heart failure and cardiomyopathy, cardiac enlargement laterally displaces the apical impulse and is palpable over a wider area when left ventricular hypertrophy and dilation are present. DIF: Cognitive Level: Applying (Application) REF: dm. 501 MSC: Client Needs: Safe and Effective Care Environment: Management of Care 37. When the nurse is auscultating the carotid artery for bruits, which of these statements reflects the correct technique? a While listening with the bell of the stethoscope, the patient is asked to take a . deep breath and hold it. b While auscultating one side with the bell of the stethoscope, the carotid artery is . palpated on the other side to check pulsations. c While lightly applying the bell of the stethoscope over the carotid artery and . listening, the patient is asked to take a breath, exhale, and briefly hold it. d While firmly placing the bell of the stethoscope over the carotid artery and . listening, the patient is asked to take a breath, exhale, and briefly hold it. ANS: C The correct technique for auscultating the carotid artery for bruits involves the nurse lightly applying the bell of the stethoscope over the carotid artery at three levels. While listening, the nurse asks the patient take a breath, exhale, and briefly hold it. Holding the breath on inhalation will also tense the levator scapulae muscles, which makes it hard to hear the carotid arteries. Examining only one carotid artery at a time will avoid compromising arterial blood flow to the brain. Pressure over the carotid sinus, which may lead to decreased heart rate, decreased blood pressure, and cerebral ischemia with syncope, should be avoided. DIF: Cognitive Level: Applying (Application) REF: dm. 476 MSC: Client Needs: Safe and Effective Care Environment: Management of Care 38. The nurse is preparing for a class on risk factors for hypertension and reviews recent statistics. Which racial group has the highest prevalence of hypertension in the world? a Blacks . b Whites . c American Indians . d Hispanics . ANS: A According to the American Heart Association, the prevalence of hypertension is higher among Blacks than in other racial groups. Chapter 7 Advanced Health Assessment of the Respiratory System MULTIPLE CHOICE 1. Which of these statements is true regarding the vertebra prominens? The vertebra prominens is: a The spinous process of C7. . b Usually nonpalpable in most individuals. . c Opposite the interior border of the scapula. . d Located next to the manubrium of the sternum. . ANS: A The spinous process of C7 is the vertebra prominens and is the most prominent bony spur protruding at the base of the neck. Counting ribs and intercostal spaces on the posterior thorax is difficult because of the muscles and soft tissue. The vertebra prominens is easier to identify and is used as a starting point in counting thoracic processes and identifying landmarks on the posterior chest. DIF: Cognitive Level: Remembering (Knowledge) REF: dm. 414 MSC: Client Needs: General 2. When performing a respiratory assessment on a patient, the nurse notices a costal angle of approximately 90 degrees. This characteristic is: a Observed in patients with kyphosis. . b Indicative of pectus excavatum. . c A normal finding in a healthy adult. . d An expected finding in a patient with a barrel chest. . ANS: C The right and left costal margins form an angle where they meet at the xiphoid process. Usually, this angle is 90 degrees or less. The angle increases when the rib cage is chronically overinflated, as in emphysema. DIF: Cognitive Level: Understanding (Comprehension) REF: dm. 414 MSC: Client Needs: Safe and Effective Care Environment: Management of Care 3. When assessing a patients lungs, the nurse recalls that the left lung: a Consists of two lobes. . b Is divided by the horizontal fissure. . c Primarily consists of an upper lobe on the posterior chest. . d Is shorter than the right lung because of the underlying stomach. . ANS: A The left lung has two lobes, and the right lung has three lobes. The right lung is shorter than the left lung because of the underlying liver. The left lung is narrower than the right lung because the heart bulges to the left. The posterior chest is almost all lower lobes. DIF: Cognitive Level: Remembering (Knowledge) REF: dm. 415 MSC: Client Needs: General 4. Which statement about the apices of the lungs is true? The apices of the lungs: a Are at the level of the second rib anteriorly. . b Extend 3 to 4 cm above the inner third of the clavicles. . c Are located at the sixth rib anteriorly and the eighth rib laterally. . d Rest on the diaphragm at the fifth intercostal space in the midclavicular line . (MCL). ANS: B The apex of the lung on the anterior chest is 3 to 4 cm above the inner third of the clavicles. On the posterior chest, the apices are at the level of C7. DIF: Cognitive Level: Remembering (Knowledge) REF: dm. 415 MSC: Client Needs: General 5. During an examination of the anterior thorax, the nurse is aware that the trachea bifurcates anteriorly at the: a Costal angle. . b . Sternal angle. c . Xiphoid process. d . Suprasternal notch. ANS: B The sternal angle marks the site of tracheal bifurcation into the right and left main bronchi; it corresponds with the upper borders of the atria of the heart, and it lies above the fourth thoracic vertebra on the back. DIF: Cognitive Level: Remembering (Knowledge) REF: dm. 416 MSC: Client Needs: Safe and Effective Care Environment: Management of Care 6. During an assessment, the nurse knows that expected assessment findings in the normal adult lung include the presence of: a Adventitious sounds and limited chest expansion. . b Increased tactile fremitus and dull percussion tones. . c Muffled voice sounds and symmetric tactile fremitus. . d Absent voice sounds and hyperresonant percussion tones. . ANS: C Normal lung findings include symmetric chest expansion, resonant percussion tones, vesicular breath sounds over the peripheral lung fields, muffled voice sounds, and no adventitious sounds. DIF: Cognitive Level: Understanding (Comprehension) REF: dm. 426 |dm. 431 MSC: Client Needs: Safe and Effective Care Environment: Management of Care 7. The primary muscles of respiration include the: a . Diaphragm and intercostals. b . Sternomastoids and scaleni. c . Trapezii and rectus abdominis. d External obliques and pectoralis major. . ANS: A The major muscle of respiration is the diaphragm. The intercostal muscles lift the sternum and elevate the ribs during inspiration, increasing the anteroposterior diameter. Expiration is primarily passive. Forced inspiration involves the use of other muscles, such as the accessory neck musclessternomastoid, scaleni, and trapezii muscles. Forced expiration involves the abdominal muscles. DIF: Cognitive Level: Remembering (Knowledge) REF: dm. 418 MSC: Client Needs: General 8. A 65-year-old patient with a history of heart failure comes to the clinic with complaints of being awakened from sleep with shortness of breath. Which action by the nurse is most appropriate? a . Obtaining a detailed health history of the patients allergies and a history of asthma b . Telling the patient to sleep on his or her right side to facilitate ease of respirations c . Assessing for other signs and symptoms of paroxysmal nocturnal dyspnea d . Assuring the patient that paroxysmal nocturnal dyspnea is normal and will probably resolve within the next week ANS: C The patient is experiencing paroxysmal nocturnal dyspneabeing awakened from sleep with shortness of breath and the need to be upright to achieve comfort. DIF: Cognitive Level: Applying (Application) REF: dm. 421 MSC: Client Needs: Safe and Effective Care Environment: Management of Care 9. When assessing tactile fremitus, the nurse recalls that it is normal to feel tactile fremitus most intensely over which location? a . Between the scapulae b . Third intercostal space, MCL c . Fifth intercostal space, midaxillary line (MAL) d Over the lower lobes, posterior side . ANS: A Normally, fremitus is most prominent between the scapulae and around the sternum. These sites are where the major bronchi are closest to the chest wall. Fremitus normally decreases as one progresses down the chest because more tissue impedes sound transmission. DIF: Cognitive Level: Understanding (Comprehension) REF: dm. 426 MSC: Client Needs: Safe and Effective Care Environment: Management of Care 10. The nurse is reviewing the technique of palpating for tactile fremitus with a new graduate. Which statement by the graduate nurse reflects a correct understanding of tactile fremitus? Tactile fremitus: a . Is caused by moisture in the alveoli. b . Indicates that air is present in the subcutaneous tissues. c . Is caused by sounds generated from the larynx. d . Reflects the blood flow through the pulmonary arteries. ANS: C Fremitus is a palpable vibration. Sounds generated from the larynx are transmitted through patent bronchi and the lung parenchyma to the chest wall where they are felt as vibrations. Crepitus is the term for air in the subcutaneous tissues. DIF: Cognitive Level: Understanding (Comprehension) REF: dm. 426 MSC: Client Needs: Safe and Effective Care Environment: Management of Care 11. During percussion, the nurse knows that a dull percussion note elicited over a lung lobe most likely results from: a . Shallow breathing. b . Normal lung tissue. c . Decreased adipose tissue. d . Increased density of lung tissue. ANS: D A dull percussion note indicates an abnormal density in the lungs, as with pneumonia, pleural effusion, atelectasis, or a tumor. Resonance is the expected finding in normal lung tissue. DIF: Cognitive Level: Understanding (Comprehension) REF: dm. 427 MSC: Client Needs: General 12. The nurse is observing the auscultation technique of another nurse. The correct method to use when progressing from one auscultatory site on the thorax to another is comparison. a . Side-to-side b . Top-to-bottom c . Posterior-to-anterior d . Interspace-by-interspace ANS: A Side-to-side comparison is most important when auscultating the chest. The nurse should listen to at least one full respiration in each location. The other techniques are not correct. DIF: Cognitive Level: Understanding (Comprehension) REF: dm. 435 MSC: Client Needs: Safe and Effective Care Environment: Management of Care 13. When auscultating the lungs of an adult patient, the nurse notes that low-pitched, soft breath sounds are heard over the posterior lower lobes, with inspiration being longer than expiration. The nurse interprets that these sounds are: a . Normally auscultated over the trachea. b . Bronchial breath sounds and normal in that location. c . Vesicular breath sounds and normal in that location. d . Bronchovesicular breath sounds and normal in that location. ANS: C Vesicular breath sounds are low-pitched, soft sounds with inspiration being longer than expiration. These breath sounds are expected over the peripheral lung fields where air flows through smaller bronchioles and alveoli. DIF: Cognitive Level: Applying (Application) REF: dm. 430 MSC: Client Needs: Safe and Effective Care Environment: Management of Care 14. The nurse is auscultating the chest in an adult. Which technique is correct? a . Instructing the patient to take deep, rapid breaths b . Instructing the patient to breathe in and out through his or her nose c . Firmly holding the diaphragm of the stethoscope against the chest d . Lightly holding the bell of the stethoscope against the chest to avoid friction ANS: C Firmly holding the diaphragm of the stethoscope against the chest is the correct way to auscultate breath sounds. The patient should be instructed to breathe through his or her mouth, a little deeper than usual, but not to hyperventilate. DIF: Cognitive Level: Understanding (Comprehension) REF: dm. 429 MSC: Client Needs: Safe and Effective Care Environment: Management of Care 15. The nurse is percussing over the lungs of a patient with pneumonia. The nurse knows that percussion over an area of atelectasis in the lungs will reveal: a . Dullness. b . Tympany. c . Resonance. d . Hyperresonance. ANS: A A dull percussion note signals an abnormal density in the lungs, as with pneumonia, pleural effusion, atelectasis, or a tumor. DIF: Cognitive Level: Understanding (Comprehension) REF: dm. 427 MSC: Client Needs: Physiologic Integrity: Physiologic Adaptation 16. During auscultation of the lungs, the nurse expects decreased breath sounds to be heard in which situation? a . When the bronchial tree is obstructed b . When adventitious sounds are present c . In conjunction with whispered pectoriloquy d . In conditions of consolidation, such as pneumonia ANS: A Decreased or absent breath sounds occur when the bronchial tree is obstructed, as in emphysema, and when sound transmission is obstructed, as in pleurisy, pneumothorax, or pleural effusion. DIF: Cognitive Level: Understanding (Comprehension) REF: dm. 430 MSC: Client Needs: Physiologic Integrity: Physiologic Adaptation 17. The nurse knows that a normal finding when assessing the respiratory system of an older adult is: a . Increased thoracic expansion. b . Decreased mobility of the thorax. c . Decreased anteroposterior diameter. d . Bronchovesicular breath sounds throughout the lungs. ANS: B The costal cartilages become calcified with aging, resulting in a less mobile thorax. Chest expansion may be somewhat decreased, and the chest cage commonly shows an increased anteroposterior diameter. DIF: Cognitive Level: Understanding (Comprehension) REF: dm. 438 MSC: Client Needs: Health Promotion and Maintenance 18. A mother brings her 3-month-old infant to the clinic for evaluation of a cold. She tells the nurse that he has had a runny nose for a week. When performing the physical assessment, the nurse notes that the child has nasal flaring and sternal and intercostal retractions. The nurses next action should be to: a . Assure the mother that these signs are normal symptoms of a cold. b . Recognize that these are serious signs, and contact the physician. c . Ask the mother if the infant has had trouble with feedings. d . Perform a complete cardiac assessment because these signs are probably indicative of early heart failure. ANS: B The infant is an obligatory nose breather until the age of 3 months. Normally, no flaring of the nostrils and no sternal or intercostal retraction occurs. Significant retractions of the sternum and intercostal muscles and nasal flaring indicate increased inspiratory effort, as in pneumonia, acute airway obstruction, asthma, and atelectasis; therefore, immediate referral to the physician is warranted. These signs do not indicate heart failure, and an assessment of the infants feeding is not a priority at this time. DIF: Cognitive Level: Analyzing (Analysis) REF: dm. 437 MSC: Client Needs: Physiologic Integrity: Physiologic Adaptation 19. When assessing the respiratory system of a 4-year-old child, which of these findings would the nurse expect? a . Crepitus palpated at the costochondral junctions b . No diaphragmatic excursion as a result of a childs decreased inspiratory volume c . Presence of bronchovesicular breath sounds in the peripheral lung fields d . Irregular respiratory pattern and a respiratory rate of 40 breaths per minute at rest ANS: C Bronchovesicular breath sounds in the peripheral lung fields of the infant and young child up to age 5 or 6 years are normal findings. Their thin chest walls with underdeveloped musculature do not dampen the sound, as do the thicker chest walls of adults; therefore, breath sounds are loud and harsh. DIF: Cognitive Level: Applying (Application) REF: dm. 437 MSC: Client Needs: Safe and Effective Care Environment: Management of Care 20. When inspecting the anterior chest of an adult, the nurse should include which assessment? a . Diaphragmatic excursion b . Symmetric chest expansion c . Presence of breath sounds d . Shape and configuration of the chest wall ANS: D Inspection of the anterior chest includes shape and configuration of the chest wall; assessment of the patients level of consciousness and the patients skin color and condition; quality of respirations; presence or absence of retraction and bulging of the intercostal spaces; and use of accessory muscles. Symmetric chest expansion is assessed by palpation. Diaphragmatic excursion is assessed by percussion of the posterior chest. Breath sounds are assessed by auscultation. DIF: Cognitive Level: Understanding (Comprehension) REF: dm. 432 MSC: Client Needs: Safe and Effective Care Environment: Management of Care 21. The nurse knows that auscultation of fine crackles would most likely be noticed in: a . A healthy 5-year-old child. b . A pregnant woman. c . The immediate newborn period. d . Association with a pneumothorax. ANS: C Fine crackles are commonly heard in the immediate newborn period as a result of the opening of the airways and a clearing of fluid. Persistent fine crackles would be noticed with pneumonia, bronchiolitis, or atelectasis. DIF: Cognitive Level: Applying (Application) REF: dm. 438 MSC: Client Needs: Safe and Effective Care Environment: Management of Care 22. During an assessment of an adult, the nurse has noted unequal chest expansion and recognizes that this occurs in which situation? a . In an obese patient b . When part of the lung is obstructed or collapsed c . When bulging of the intercostal spaces is present d . When accessory muscles are used to augment respiratory effort ANS: B Unequal chest expansion occurs when part of the lung is obstructed or collapsed, as with pneumonia, or when guarding to avoid postoperative incisional pain. DIF: Cognitive Level: Understanding (Comprehension) REF: dm. 432 MSC: Client Needs: Physiologic Integrity: Physiologic Adaptation 23. During auscultation of the lungs of an adult patient, the nurse notices the presence of bronchophony. The nurse should assess for signs of which condition? a . Airway obstruction b . Emphysema c . Pulmonary consolidation d . Asthma ANS: C Pathologic conditions that increase lung density, such as pulmonary consolidation, will enhance the transmission of voice sounds, such as bronchophony (see Table 18-7). DIF: Cognitive Level: Understanding (Comprehension) REF: dm. 449 MSC: Client Needs: Physiologic Integrity: Physiologic Adaptation 24. The nurse is reviewing the characteristics of breath sounds. Which statement about bronchovesicular breath sounds is true? Bronchovesicular breath sounds are: a Musical in quality. . b Usually caused by a pathologic disease. . c Expected near the major airways. . d Similar to bronchial sounds except shorter in duration. . ANS: C Bronchovesicular breath sounds are heard over major bronchi where fewer alveoli are located posteriorlybetween the scapulae, especially on the right; and anteriorly, around the upper sternum in the first and second intercostal spaces. The other responses are not correct. DIF: Cognitive Level: Understanding (Comprehension) REF: dm. 430 MSC: Client Needs: Safe and Effective Care Environment: Management of Care 25. The nurse is listening to the breath sounds of a patient with severe asthma. Air passing through narrowed bronchioles would produce which of these adventitious sounds? a Wheezes . b Bronchial sounds . c Bronchophony . d Whispered pectoriloquy . ANS: A Wheezes are caused by air squeezed or compressed through passageways narrowed almost to closure by collapsing, swelling, secretions, or tumors, such as with acute asthma or chronic emphysema. DIF: Cognitive Level: Understanding (Comprehension) REF: dm. 447 MSC: Client Needs: Physiologic Integrity: Physiologic Adaptation 26. A patient has a long history of chronic obstructive pulmonary disease (COPD). During the assessment, the nurse will most likely observe which of these? a . Unequal chest expansion b . Increased tactile fremitus c . Atrophied neck and trapezius muscles d Anteroposterior-to-transverse diameter ratio of 1:1 . ANS: D An anteroposterior-to-transverse diameter ratio of 1:1 or barrel chest is observed in individuals with COPD because of hyperinflation of the lungs. The ribs are more horizontal, and the chest appears as if held in continuous inspiration. Neck muscles are hypertrophied from aiding in forced respiration. Chest expansion may be decreased but symmetric. Decreased tactile fremitus occurs from decreased transmission of vibrations. DIF: Cognitive Level: Applying (Application) REF: dm. 448 MSC: Client Needs: Physiologic Integrity: Physiologic Adaptation 27. A teenage patient comes to the emergency department with complaints of an inability to breathe and a sharp pain in the left side of his chest. The assessment findings include cyanosis, tachypnea, tracheal deviation to the right, decreased tactile fremitus on the left, hyperresonance on the left, and decreased breath sounds on the left. The nurse interprets that these assessment findings are consistent with: a . Bronchitis. b . Pneumothorax. c . Acute pneumonia. d . Asthmatic attack. ANS: B With a pneumothorax, free air in the pleural space causes partial or complete lung collapse. If the pneumothorax is large, then tachypnea and cyanosis are evident. Unequal chest expansion, decreased or absent tactile fremitus, tracheal deviation to the unaffected side, decreased chest expansion, hyperresonant percussion tones, and decreased or absent breath sounds are found with the presence of pneumothorax. (See Table 18-8 for descriptions of the other conditions.) DIF: Cognitive Level: Analyzing (Analysis) REF: dm. 454 MSC: Client Needs: Physiologic Integrity: Physiologic Adaptation 28. An adult patient with a history of allergies comes to the clinic complaining of wheezing and difficulty in breathing when working in his yard. The assessment findings include tachypnea, the use of accessory neck muscles, prolonged expiration, intercostal retractions, decreased breath sounds, and expiratory wheezes. The nurse interprets that these assessment findings are consistent with: a . Asthma. b . Atelectasis. c . Lobar pneumonia. d . Heart failure. ANS: A Asthma is allergic hypersensitivity to certain inhaled particles that produces inflammation and a reaction of bronchospasm, which increases airway resistance, especially during expiration. An increased respiratory rate, the use of accessory muscles, a retraction of the intercostal muscles, prolonged expiration, decreased breath sounds, and expiratory wheezing are all characteristics of asthma. (See Table 18-8 for descriptions of the other conditions.) DIF: Cognitive Level: Analyzing (Analysis) REF: dm. 453 MSC: Client Needs: Physiologic Integrity: Physiologic Adaptation 29. The nurse is assessing the lungs of an older adult. Which of these changes are normal in the respiratory system of the older adult? a . Severe dyspnea is experienced on exertion, resulting from changes in the lungs. b . Respiratory muscle strength increases to compensate for a decreased vital capacity. c . Decrease in small airway closure occurs, leading to problems with atelectasis. d . Lungs are less elastic and distensible, which decreases their ability to collapse and recoil. ANS: D In the aging adult, the lungs are less elastic and distensible, which decreases their ability to collapse and recoil. Vital capacity is decreased, and a loss of intra-alveolar septa occurs, causing less surface area for gas exchange. The lung bases become less ventilated, and the older person is at risk for dyspnea with exertion beyond his or her usual workload. DIF: Cognitive Level: Understanding (Comprehension) REF: dm. 419 MSC: Client Needs: Health Promotion and Maintenance 30. A woman in her 26th week of pregnancy states that she is not really short of breath but feels that she is aware of her breathing and the need to breathe. What is the nurses best reply? a . The diaphragm becomes fixed during pregnancy, making it difficult to take in a deep breath. b . The increase in estrogen levels during pregnancy often causes a decrease in the diameter of the rib cage and makes it difficult to breathe. c . What you are experiencing is normal. Some women may interpret this as shortness of breath, but it is a normal finding and nothing is wrong. d . This increased awareness of the need to breathe is normal as the fetus grows because of the increased oxygen demand on the mothers body, which results in an increased respiratory rate. ANS: C During pregnancy, the woman may develop an increased awareness of the need to breathe. Some women may interpret this as dyspnea, although structurally nothing is wrong. Increases in estrogen relax the chest cage ligaments, causing an increase in the transverse diameter. Although the growing fetus increases the oxygen demand on the mothers body, this increased demand is easily met by the increasing tidal volume (deeper breathing). Little change occurs in the respiratory rate. DIF: Cognitive Level: Applying (Application) REF: dm. 419 MSC: Client Needs: Health Promotion and Maintenance 31. A 35-year-old recent immigrant is being seen in the clinic for complaints of a cough that is associated with rust-colored sputum, low-grade afternoon fevers, and night sweats for the past 2 months. The nurses preliminary analysis, based on this history, is that this patient may be suffering from: a . Bronchitis. b . Pneumonia. c . Tuberculosis. d . Pulmonary edema. ANS: C Sputum is not diagnostic alone, but some conditions have characteristic sputum production. Tuberculosis often produces rust-colored sputum in addition to other symptoms of night sweats and low-grade afternoon fevers (see Table 18-8). DIF: Cognitive Level: Analyzing (Analysis) REF: dm. 455 MSC: Client Needs: Physiologic Integrity: Physiologic Adaptation 32. A 70-year-old patient is being seen in the clinic for severe exacerbation of his heart failure. Which of these findings is the nurse most likely to observe in this patient? a . Shortness of breath, orthopnea, paroxysmal nocturnal dyspnea, and ankle edema b . Rasping cough, thick mucoid sputum, wheezing, and bronchitis c . Productive cough, dyspnea, weight loss, anorexia, and tuberculosis d . Fever, dry nonproductive cough, and diminished breath sounds ANS: A A person with heart failure often exhibits increased respiratory rate, shortness of breath on exertion, orthopnea, paroxysmal nocturnal dyspnea, nocturia, ankle edema, and pallor in light- skinned individuals. A patient with rasping cough, thick mucoid sputum, and wheezing may have bronchitis. Productive cough, dyspnea, weight loss, and dyspnea indicate tuberculosis; fever, dry nonproductive cough, and diminished breath sounds may indicate Pneumocystis jiroveci (dm. carinii) pneumonia (see Table 18-8). DIF: Cognitive Level: Applying (Application) REF: dm. 448 MSC: Client Needs: Physiologic Integrity: Physiologic Adaptation 33. A patient comes to the clinic complaining of a cough that is worse at night but not as bad during the day. The nurse recognizes that this cough may indicate: a . Pneumonia. b . Postnasal drip or sinusitis. c . Exposure to irritants at work. d . Chronic bronchial irritation from smoking. ANS: B A cough that primarily occurs at night may indicate postnasal drip or sinusitis. Exposure to irritants at work causes an afternoon or evening cough. Smokers experience early morning coughing. Coughing associated with acute illnesses such as pneumonia is continuous throughout the day. DIF: Cognitive Level: Applying (Application) REF: dm. 420 MSC: Client Needs: Physiologic Integrity: Physiologic Adaptation 34. During a morning assessment, the nurse notices that the patients sputum is frothy and pink. Which condition could this finding indicate? a . Croup b . Tuberculosis c . Viral infection d . Pulmonary edema ANS: D Sputum, alone, is not diagnostic, but some conditions have characteristic sputum production. Pink, frothy sputum indicates pulmonary edema or it may be a side effect of sympathomimetic medications. Croup is associated with a barking cough, not sputum production. Tuberculosis may produce rust-colored sputum. Viral infections may produce white or clear mucoid sputum. DIF: Cognitive Level: Understanding (Comprehension) REF: dm. 420 MSC: Client Needs: Physiologic Integrity: Physiologic Adaptation 35. During auscultation of breath sounds, the nurse should correctly use the stethoscope in which of the following ways? a . Listening to at least one full respiration in each location b . Listening as the patient inhales and then going to the next site during exhalation c . Instructing the patient to breathe in and out rapidly while listening to the breath sounds d . If the patient is modest, listening to sounds over his or her clothing or hospital gown ANS: A During auscultation of breath sounds with a stethoscope, listening to one full respiration in each location is important. During the examination, the nurse should monitor the breathing and offer times for the person to breathe normally to prevent possible dizziness. DIF: Cognitive Level: Applying (Application) REF: dm. 429 MSC: Client Needs: Safe and Effective Care Environment: Management of Care 36. A patient has been admitted to the emergency department with a possible medical diagnosis of pulmonary embolism. The nurse expects to see which assessment findings related to this condition? a . Absent or decreased breath sounds b . Productive cough with thin, frothy sputum c . Chest pain that is worse on deep inspiration and dyspnea d . Diffuse infiltrates with areas of dullness upon percussion ANS: C Findings for pulmonary embolism include chest pain that is worse on deep inspiration, dyspnea, apprehension, anxiety, restlessness, partial arterial pressure of oxygen (PaO2) less than 80 mm Hg, diaphoresis, hypotension, crackles, and wheezes. DIF: Cognitive Level: Analyzing (Analysis) REF: dm. 456 MSC: Client Needs: Physiologic Integrity: Physiologic Adaptation 37. During palpation of the anterior chest wall, the nurse notices a coarse, crackling sensation over the skin surface. On the basis of these findings, the nurse suspects: a . Tactile fremitus. b . Crepitus. c . Friction rub. d . Adventitious sounds. ANS: B Crepitus is a coarse, crackling sensation palpable over the skin surface. It occurs in subcutaneous emphysema when air escapes from the lung and enters the subcutaneous tissue, such as after open thoracic injury or surgery. DIF: Cognitive Level: Applying (Application) REF: dm. 427 MSC: Client Needs: Physiologic Integrity: Physiologic Adaptation 38. The nurse is auscultating the lungs of a patient who had been sleeping and notices short, popping, crackling sounds that stop after a few breaths. The nurse recognizes that these breath sounds are: a . Atelectatic crackles that do not have a pathologic cause. b . Fine crackles and may be a sign of pneumonia. c . Vesicular breath sounds. d . Fine wheezes. ANS: A One type of adventitious sound, atelectatic crackles, does not have a pathologic cause. They are short, popping, crackling sounds that sound similar to fine crackles but do not last beyond a few breaths. When sections of alveoli are not fully aerated (as in people who are asleep or in older adults), they deflate slightly and accumulate secretions. Crackles are heard when these sections are expanded by a few deep breaths. Atelectatic crackles are heard only in the periphery, usually in dependent portions of the lungs, and disappear after the first few breaths or after a cough. DIF: Cognitive Level: Analyzing (Analysis) REF: dm. 431 MSC: Client Needs: Safe and Effective Care Environment: Management of Care 39. A patient has been admitted to the emergency department for a suspected drug overdose. His respirations are shallow, with an irregular pattern, with a rate of 12 respirations per minute. The nurse interprets this respiration pattern as which of the following? a . Bradypnea b . Cheyne-Stokes respirations c . Hypoventilation d . Chronic obstructive breathing ANS: C Hypoventilation is characterized by an irregular, shallow pattern, and can be caused by an overdose of narcotics or anesthetics. Bradypnea is slow breathing, with a rate less than 10 respirations per minute. (See Table 18-4 for descriptions of Cheyne-Stokes respirations and chronic obstructive breathing.) DIF: Cognitive Level: Analyzing (Analysis) REF: dm. 444 MSC: Client Needs: Safe and Effective Care Environment: Management of Care 40. A patient with pleuritis has been admitted to the hospital and complains of pain with breathing. What other key assessment finding would the nurse expect to find upon auscultation? a Stridor . b Friction rub . c Crackles . d Wheezing . ANS: B A patient with pleuritis will exhibit a pleural friction rub upon auscultation. This sound is made when the pleurae become inflamed and rub together during respiration. The sound is superficial, coarse, and low-pitched, as if two pieces of leather are being rubbed together. Stridor is associated with croup, acute epiglottitis in children, and foreign body inhalation. Crackles are associated with pneumonia, heart failure, chronic bronchitis, and other diseases (see Table 18-6). Wheezes are associated with diffuse airway obstruction caused by acute asthma or chronic emphysema. Chapter 8 Advanced Health Assessment of the Abdomen, Rectum, and Anus MULTIPLE CHOICE 1. The nurse is percussing the seventh right intercostal space at the midclavicular line over the liver. Which sound should the nurse expect to hear? a . Dullness b . Tympany c . Resonance d . Hyperresonance ANS: A The liver is located in the right upper quadrant and would elicit a dull percussion note. DIF: Cognitive Level: Understanding (Comprehension) REF: dm. 550 MSC: Client Needs: Safe and Effective Care Environment: Management of Care 2. Which structure is located in the left lower quadrant of the abdomen? a Liver . b Duodenum . c Gallbladder . d Sigmoid colon . ANS: D The sigmoid colon is located in the left lower quadrant of the abdomen. DIF: Cognitive Level: Remembering (Knowledge) REF: dm. 540 MSC: Client Needs: General 3. A patient is having difficulty swallowing medications and food. The nurse would document that this patient has: a Aphasia. . b Dysphasia. . c Dysphagia. . d Anorexia. . ANS: C Dysphagia is a condition that occurs with disorders of the throat or esophagus and results in difficulty swallowing. Aphasia and dysphasia are speech disorders. Anorexia is a loss of appetite. DIF: Cognitive Level: Applying (Application) REF: dm. 542 MSC: Client Needs: Safe and Effective Care Environment: Management of Care 4. The nurse suspects that a patient has a distended bladder. How should the nurse assess for this condition? a Percuss and palpate in the lumbar region. . b . Inspect and palpate in the epigastric region. c . Auscultate and percuss in the inguinal region. d . Percuss and palpate the midline area above the suprapubic bone. ANS: D Dull percussion sounds would be elicited over a distended bladder, and the hypogastric area would seem firm to palpation. DIF: Cognitive Level: Applying (Application) REF: dm. 540 MSC: Client Needs: Safe and Effective Care Environment: Management of Care 5. The nurse is aware that one change that may occur in the gastrointestinal system of an aging adult is: a Increased salivation. . b Increased liver size. . c Increased esophageal emptying. . d Decreased gastric acid secretion. . ANS: D Gastric acid secretion decreases with aging. As one ages, salivation decreases, esophageal emptying is delayed, and liver size decreases. DIF: Cognitive Level: Understanding (Comprehension) REF: dm. 541 MSC: Client Needs: Health Promotion and Maintenance 6. A 22-year-old man comes to the clinic for an examination after falling off his motorcycle and landing on his left side on the handle bars. The nurse suspects that he may have injured his spleen. Which of these statements is trueregarding assessment of the spleen in this situation? a The spleen can be enlarged as a result of trauma. . b The spleen is normally felt on routine palpation. . c If an enlarged spleen is noted, then the nurse should thoroughly palpate to . determine its size. d An enlarged spleen should not be palpated because it can easily rupture. . ANS: D If an enlarged spleen is felt, then the nurse should refer the person and should not continue to palpate it. An enlarged spleen is friable and can easily rupture with overpalpation. DIF: Cognitive Level: Applying (Application) REF: dm. 558 MSC: Client Needs: Physiologic Integrity: Physiologic Adaptation 7. A patients abdomen is bulging and stretched in appearance. The nurse should describe this finding as: a Obese. . b Herniated. . c Scaphoid. . d Protuberant. . ANS: D A protuberant abdomen is rounded, bulging, and stretched (see Figure 21-7). A scaphoid abdomen caves inward. DIF: Cognitive Level: Remembering (Knowledge) REF: dm. 546 MSC: Client Needs: Physiologic Integrity: Physiologic Adaptation 8. The nurse is describing a scaphoid abdomen. To the horizontal plane, a scaphoid contour of the abdomen depicts a profile. a Flat . b Convex . c Bulging . d Concave . ANS: D Contour describes the profile of the abdomen from the rib margin to the pubic bone; a scaphoid contour is one that is concave from a horizontal plane (see Figure 21-7). DIF: Cognitive Level: Understanding (Comprehension) REF: dm. 546 MSC: Client Needs: Physiologic Integrity: Physiologic Adaptation 9. While examining a patient, the nurse observes abdominal pulsations between the xiphoid process and umbilicus. The nurse would suspect that these are: a Pulsations of the renal arteries. . b Pulsations of the inferior vena cava. . c Normal abdominal aortic pulsations. . d Increased peristalsis from a bowel obstruction. . ANS: C Normally, the pulsations from the aorta are observed beneath the skin in the epigastric area, particularly in thin persons who have good muscle wall relaxation. DIF: Cognitive Level: Applying (Application) REF: dm. 549 MSC: Client Needs: Safe and Effective Care Environment: Management of Care 10. A patient has hypoactive bowel sounds. The nurse knows that a potential cause of hypoactive bowel sounds is: a Diarrhea. . b Peritonitis. . c Laxative use. . d Gastroenteritis. . ANS: B Diminished or absent bowel sounds signal decreased motility from inflammation as exhibited with peritonitis, with paralytic ileus after abdominal surgery, or with late bowel obstruction. DIF: Cognitive Level: Understanding (Comprehension) REF: dm. 549 MSC: Client Needs: Physiologic Integrity: Physiologic Adaptation 11. The nurse is watching a new graduate nurse perform auscultation of a patients abdomen. Which statement by the new graduate shows a correct understanding of the reason auscultation precedes percussion and palpation of the abdomen? a We need to determine the areas of tenderness before using percussion and . palpation. b Auscultation prevents distortion of bowel sounds that might occur after . percussion and palpation. c Auscultation allows the patient more time to relax and therefore be more . comfortable with the physical examination. d Auscultation prevents distortion of vascular sounds, such as bruits and hums, . that might occur after percussion and palpation. ANS: B Auscultation is performed first (after inspection) because percussion and palpation can increase peristalsis, which would give a false interpretation of bowel sounds. DIF: Cognitive Level: Applying (Application) REF: dm. 548 MSC: Client Needs: Safe and Effective Care Environment: Management of Care 12. The nurse is listening to bowel sounds. Which of these statements is true of bowel sounds? Bowel sounds: a Are usually loud, high-pitched, rushing, and tinkling sounds. . b Are usually high-pitched, gurgling, and irregular sounds. . c Sound like two pieces of leather being rubbed together. . d Originate from the movement of air and fluid through the large intestine. . ANS: B Bowel sounds are high-pitched, gurgling, and cascading sounds that irregularly occur from 5 to 30 times per minute. They originate from the movement of air and fluid through the small intestine. DIF: Cognitive Level: Understanding (Comprehension) REF: dm. 549 MSC: Client Needs: Safe and Effective Care Environment: Management of Care 13. The physician comments that a patient has abdominal borborygmi. The nurse knows that this term refers to: a Loud continual hum. . b Peritoneal friction rub. . c Hypoactive bowel sounds. . d Hyperactive bowel sounds. . ANS: D Borborygmi is the term used for hyperperistalsis when the person actually feels his or her stomach growling. DIF: Cognitive Level: Understanding (Comprehension) REF: dm. 549 MSC: Client Needs: Safe and Effective Care Environment: Management of Care 14. During an abdominal assessment, the nurse would consider which of these findings as normal? a Presence of a bruit in the femoral area . b Tympanic percussion note in the umbilical region . c Palpable spleen between the ninth and eleventh ribs in the left midaxillary line . d Dull percussion note in the left upper quadrant at the midclavicular line . ANS: B Tympany should predominate in all four quadrants of the abdomen because air in the intestines rises to the surface when the person is supine. Vascular bruits are not usually present. Normally, the spleen is not palpable. Dullness would not be found in the area of lung resonance (left upper quadrant at the midclavicular line). DIF: Cognitive Level: Understanding (Comprehension) REF: dm. 550 MSC: Client Needs: Safe and Effective Care Environment: Management of Care 15. The nurse is assessing the abdomen of a pregnant woman who is complaining of having acid indigestion all the time. The nurse knows that esophageal reflux during pregnancy can cause: a Diarrhea. . b Pyrosis. . c Dysphagia. . d Constipation. . ANS: B Pyrosis, or heartburn, is caused by esophageal reflux during pregnancy. The other options are not correct. DIF: Cognitive Level: Applying (Application) REF: dm. 540 MSC: Client Needs: Physiologic Integrity: Physiologic Adaptation 16. The nurse is performing percussion during an abdominal assessment. Percussion notes heard during the abdominal assessment may include: a Flatness, resonance, and dullness. . b Resonance, dullness, and tympany. . c Tympany, hyperresonance, and dullness. . d Resonance, hyperresonance, and flatness. . ANS: C Percussion notes normally heard during the abdominal assessment may include tympany, which should predominate because air in the intestines rises to the surface when the person is supine; hyperresonance, which may be present with gaseous distention; and dullness, which may be found over a distended bladder, adipose tissue, fluid, or a mass. DIF: Cognitive Level: Understanding (Comprehension) REF: dm. 550 MSC: Client Needs: Safe and Effective Care Environment: Management of Care 17. An older patient has been diagnosed with pernicious anemia. The nurse knows that this condition could be related to: a Increased gastric acid secretion. . b . Decreased gastric acid secretion. c . Delayed gastrointestinal emptying time. d . Increased gastrointestinal emptying time. ANS: B Gastric acid secretion decreases with aging and may cause pernicious anemia (because it interferes with vitamin B12 absorption), iron-deficiency anemia, and malabsorption of calcium. DIF: Cognitive Level: Applying (Application) REF: dm. 541 MSC: Client Needs: Physiologic Integrity: Physiologic Adaptation 18. A patient is complaining of a sharp pain along the costovertebral angles. The nurse is aware that this symptom is most often indicative of: a Ovary infection. . b Liver enlargement. . c Kidney inflammation. . d Spleen enlargement. . ANS: C Sharp pain along the costovertebral angles occurs with inflammation of the kidney or paranephric area. The other options are not correct. DIF: Cognitive Level: Applying (Application) REF: dm. 552 MSC: Client Needs: Physiologic Integrity: Physiologic Adaptation 19. A nurse notices that a patient has ascites, which indicates the presence of: a Fluid. . b Feces. . c Flatus. . d Fibroid tumors. . ANS: A Ascites is free fluid in the peritoneal cavity and occurs with heart failure, portal hypertension, cirrhosis, hepatitis, pancreatitis, and cancer. DIF: Cognitive Level: Understanding (Comprehension) REF: dm. 553 MSC: Client Needs: Physiologic Integrity: Physiologic Adaptation 20. The nurse knows that during an abdominal assessment, deep palpation is used to determine: a Bowel motility. . b Enlarged organs. . c Superficial tenderness. . d Overall impression of skin surface and superficial musculature. . ANS: B With deep palpation, the nurse should notice the location, size, consistency, and mobility of any palpable organs and the presence of any abnormal enlargement, tenderness, or masses. DIF: Cognitive Level: Understanding (Comprehension) REF: dm. 554 MSC: Client Needs: Safe and Effective Care Environment: Management of Care 21. The nurse notices that a patient has had a black, tarry stool and recalls that a possible cause would be: a Gallbladder disease. . b Overuse of laxatives. . c Gastrointestinal bleeding. . d Localized bleeding around the anus. . ANS: C Black stools may be tarry as a result of occult blood (melena) from gastrointestinal bleeding. Red blood in stools occurs with localized bleeding around the anus. DIF: Cognitive Level: Understanding (Comprehension) REF: dm. 543 MSC: Client Needs: Physiologic Integrity: Physiologic Adaptation 22. During an abdominal assessment, the nurse elicits tenderness on light palpation in the right lower quadrant. The nurse interprets that this finding could indicate a disorder of which of these structures? a Spleen . b Sigmoid . c Appendix . d Gallbladder . ANS: C The appendix is located in the right lower quadrant. When the iliopsoas muscle is inflamed, which occurs with an inflamed or perforated appendix, pain is felt in the right lower quadrant. 23. A 62-year-old man is experiencing fever, chills, malaise, urinary frequency, and urgency. He also reports urethral discharge and a dull aching pain in the perineal and rectal area. These symptoms are most consistent with which condition? a Prostatitis . b Polyps . c Carcinoma of the prostate . d BPH . ANS: A The common presenting symptoms of prostatitis are fever, chills, malaise, and urinary frequency and urgency. The individual may also have dysuria, urethral discharge, and a dull aching pain in the perineal and rectal area. These symptoms are not consistent with polyps. (See Table 25-3 for the descriptions of carcinoma of the prostate and BPH.) DIF: Cognitive Level: Analyzing (Analysis) REF: dm. 735 MSC: Client Needs: Physiologic Integrity: Physiologic Adaptation 24. During a discussion for a mens health group, the nurse relates that the group with the highest incidence of prostate cancer is: a Asian Americans. . b Blacks. . c American Indians. . d Hispanics. . ANS: B According to the American Cancer Society (2010), black men have a higher rate of prostate cancer than other racial groups. DIF: Cognitive Level: Remembering (Knowledge) REF: dm. 723 MSC: Client Needs: Physiologic Integrity: Reduction of Risk Potential 25. Which characteristic of the prostate gland would the nurse recognize as an abnormal finding while palpating the prostate gland through the rectum? a Palpable central groove . b Tenderness to palpation . c Heart shaped . d Elastic and rubbery consistency . ANS: B The normal prostate gland should feel smooth, elastic, and rubbery; slightly movable; heart- shaped with a palpable central groove; and not be tender to palpation. DIF: Cognitive Level: Analyzing (Analysis) REF: dm. 728 MSC: Client Needs: Health Promotion and Maintenance 26. The nurse notices that a patient has had a pale, yellow, greasy stool, or steatorrhea, and recalls that this is caused by: a Occult bleeding. . b . Absent bile pigment. c . Increased fat content. d . Ingestion of bismuth preparations. ANS: C Steatorrhea (pale, yellow, greasy stool) is caused by increased fat content in the stools, as in malabsorption syndrome. Occult bleeding and ingestion of bismuth products cause a black stool, and absent bile pigment causes a gray-tan stool. DIF: Cognitive Level: Understanding (Comprehension) REF: dm. 729 MSC: Client Needs: Physiologic Integrity: Physiologic Adaptation 27. During a health history of a patient who complains of chronic constipation, the patient asks the nurse about high-fiber foods. The nurse relates that an example of a high-fiber food would be: a Broccoli. . b Hamburger. . c Iceberg lettuce. . d Yogurt. . ANS: A High-fiber foods are either soluble type (e.g., beans, prunes, barley, broccoli) or insoluble type (e.g., cereals, wheat germ). The other examples are not considered high-fiber foods. DIF: Cognitive Level: Understanding (Comprehension) REF: dm. 724 MSC: Client Needs: Health Promotion and Maintenance 28. While assessing a patient who is hospitalized and bedridden, the nurse notices that the patient has been incontinent of stool. The stool is loose and gray-tan in color. The nurse recognizes that this finding indicates which of the following? a Occult blood . b Inflammation . c Absent bile pigment . d Ingestion of iron preparations . ANS: C The presence of gray-tan stool indicates absent bile pigment, which can occur with obstructive jaundice. The ingestion of iron preparations and the presence of occult blood turns the stools to a black color. Jellylike mucus shreds mixed in the stool would indicate inflammation. DIF: Cognitive Level: Applying (Application) REF: dm. 729 MSC: Client Needs: Physiologic Integrity: Physiologic Adaptation 29. During a digital examination of the rectum, the nurse notices that the patient has hard feces in the rectum. The patient complains of feeling full, has a distended abdomen, and states that she has not had a bowel movement for several days. The nurse suspects which condition? a Rectal polyp . b Fecal impaction . c Rectal abscess . d Rectal prolapse . ANS: B A fecal impaction is a collection of hard, desiccated feces in the rectum. The obstruction often results from decreased bowel motility, in which more water is reabsorbed from the stool. (See Table 25-2 for the descriptions of rectal polyps and abscesses; See Table 25-1 for a description of rectal prolapse.) DIF: Cognitive Level: Applying (Application) REF: dm. 734 MSC: Client Needs: Physiologic Integrity: Physiologic Adaptation 30. During the taking of a health history, the patient states, It really hurts back there, and sometimes it itches, too. I have even seen blood on the tissue when I have a bowel movement. Is there something there? The nurse should expect to see which of these upon examination of the anus? a Rectal prolapse . b . Internal hemorrhoid c . External hemorrhoid that has resolved d . External hemorrhoid that is thrombosed ANS: D These symptoms are consistent with an external hemorrhoid. An external hemorrhoid, when thrombosed, contains clotted blood and becomes a painful, swollen, shiny blue mass that itches and bleeds with defecation. When the external hemorrhoid resolves, it leaves a flabby, painless skin sac around the anal orifice. An internal hemorrhoid is not palpable but may appear as a red mucosal mass when the person performs a Valsalva maneuver. A rectal prolapse appears as a moist, red doughnut with radiating lines. 31. A 45-year-old man is in the clinic for a physical examination. During the abdominal assessment, the nurse percusses the abdomen and notices an area of dullness above the right costal margin of approximately 11 cm. The nurse should: a Document the presence of hepatomegaly. . b Ask additional health history questions regarding his alcohol intake. . c Describe this dullness as indicative of an enlarged liver, and refer him to a . physician. d Consider this finding as normal, and proceed with the examination. . ANS: D A liver span of 10.5 cm is the mean for males and 7 cm for females. Men and taller individuals are at the upper end of this range. Women and shorter individuals are at the lower end of this range. A liver span of 11 cm is within normal limits for this individual. DIF: Cognitive Level: Analyzing (Analysis) REF: dm. 550 MSC: Client Needs: Safe and Effective Care Environment: Management of Care 32. When palpating the abdomen of a 20-year-old patient, the nurse notices the presence of tenderness in the left upper quadrant with deep palpation. Which of these structures is most likely to be involved? a Spleen . b . Sigmoid colon c . Appendix d . Gallbladder ANS: A The spleen is located in the left upper quadrant of the abdomen. The gallbladder is in the right upper quadrant, the sigmoid colon is in the left lower quadrant, and the appendix is in the right lower quadrant. DIF: Cognitive Level: Applying (Application) REF: dm. 540 MSC: Client Needs: Physiologic Integrity: Physiologic Adaptation 33. The nurse is reviewing statistics for lactose intolerance. In the United States, the incidence of lactose intolerance is higher in adults of which ethnic group? a Blacks . b Hispanics . c Whites . d Asians . ANS: A A recent study found estimates of lactose-intolerance prevalence as follows: 19.5% for Blacks, 10% for Hispanics, and 7.72% for Whites. DIF: Cognitive Level: Understanding (Comprehension) REF: dm. 541 MSC: Client Needs: Health Promotion and Maintenance 34. The nurse is assessing a patient for possible peptic ulcer disease. Which condition or history often causes this problem? a Hypertension . b Streptococcal infections . c Recurrent constipation with frequent laxative use . d Frequent use of nonsteroidal antiinflammatory drugs . ANS: D Peptic ulcer disease occurs with the frequent use of nonsteroidal antiinflammatory drugs, alcohol use, smoking, and Helicobacter pylori infection. DIF: Cognitive Level: Applying (Application) REF: dm. 543 MSC: Client Needs: Physiologic Integrity: Reduction of Risk Potential 35. During reporting, the student nurse hears that a patient has hepatomegaly and recognizes that this term refers to: a Enlarged liver. . b Enlarged spleen. . c Distended bowel. . d Excessive diarrhea. . ANS: A The term hepatomegaly refers to an enlarged liver. The term splenomegaly refers to an enlarged spleen. The other responses are not correct. DIF: Cognitive Level: Remembering (Knowledge) REF: dm. 551 MSC: Client Needs: Physiologic Integrity: Physiologic Adaptation 36. During an assessment, the nurse notices that a patients umbilicus is enlarged and everted. It is positioned midline with no change in skin color. The nurse recognizes that the patient may have which condition? a . Intra-abdominal bleeding b . Constipation c . Umbilical hernia d Abdominal tumor . ANS: C The umbilicus is normally midline and inverted with no signs of discoloration. With an umbilical hernia, the mass is enlarged and everted. The other responses are incorrect. DIF: Cognitive Level: Applying (Application) REF: dm. 546 MSC: Client Needs: Physiologic Integrity: Physiologic Adaptation 37. During an abdominal assessment, the nurse tests for a fluid wave. A positive fluid wave test occurs with: a . Splenomegaly. b . Distended bladder. c . Constipation. d . Ascites. ANS: D If ascites (fluid in the abdomen) is present, then the examiner will feel a fluid wave when assessing the abdomen. A fluid wave is not present with splenomegaly, a distended bladder, or constipation. DIF: Cognitive Level: Applying (Application) REF: dm. 553 MSC: Client Needs: Physiologic Integrity: Physiologic Adaptation 38. The nurse is preparing to examine a patient who has been complaining of right lower quadrant pain. Which technique is correct during the assessment? The nurse should: a . Examine the tender area first. b . Examine the tender area last. c . Avoid palpating the tender area. d . Palpate the tender area first, and then auscultate for bowel sounds. ANS: B The nurse should save the examination of any identified tender areas until last. This method avoids pain and the resulting muscle rigidity that would obscure deep palpation later in the examination. Auscultation is performed before percussion and palpation because percussion and palpation can increase peristalsis, which would give a false interpretation of bowel sounds. DIF: Cognitive Level: Analyzing (Analysis) REF: dm. 555 MSC: Client Needs: Safe and Effective Care Environment: Management of Care 39. During a health history, the patient tells the nurse, I have pain all the time in my stomach. Its worse 2 hours after I eat, but it gets better if I eat again! Based on these symptoms, the nurse suspects that the patient has which condition? a . Appendicitis b . Gastric ulcer c . Duodenal ulcer d . Cholecystitis ANS: C Pain associated with duodenal ulcers occurs 2 to 3 hours after a meal; it may relieved by more food. Chronic pain associated with gastric ulcers usually occurs on an empty stomach. Severe, acute pain would occur with appendicitis and cholecystitis. 40. The nurse suspects that a patient has appendicitis. Which of these procedures are appropriate for use when assessing for appendicitis or a perforated appendix? Select all that apply. a . Test for the Murphy sign b . Test for the Blumberg sign c . Test for shifting dullness d . Perform the iliopsoas muscle test e . Test for fluid wave ANS: B, D Testing for the Blumberg sign (rebound tenderness) and performing the iliopsoas muscle test should be used when assessing for appendicitis. The Murphy sign is used when assessing for an inflamed gallbladder or cholecystitis. Testing for a fluid wave and shifting dullness is performed when assessing for ascites. Chapter 9 Advanced Health Assessment of the Male Genitourinary System MULTIPLE CHOICE 1. The external male genital structures include the: a Testis. . b Scrotum. . c Epididymis. . d Vas deferens. . ANS: B The external male genital structures include the penis and scrotum. The testis, epididymis, and vas deferens are internal structures. DIF: Cognitive Level: Remembering (Knowledge) REF: dm. 691 MSC: Client Needs: General 2. An accessory glandular structure for the male genital organs is the: a Testis. . b Scrotum. . c Prostate. . d Vas deferens. . ANS: C Glandular structures accessory to the male genital organs are the prostate, seminal vesicles, and bulbourethral glands. DIF: Cognitive Level: Remembering (Knowledge) REF: dm. 691 MSC: Client Needs: General 3. Which of these statements is true regarding the penis? a The urethral meatus is located on the ventral side of the penis. . b The prepuce is the fold of foreskin covering the shaft of the penis. . c The penis is made up of two cylindrical columns of erectile tissue. . d The corpus spongiosum expands into a cone of erectile tissue called the glans. . ANS: D At the distal end of the shaft, the corpus spongiosum expands into a cone of erectile tissue, the glans. The penis is made up of three cylindrical columns of erectile tissue. The skin that covers the glans of the penis is the prepuce. The urethral meatus forms at the tip of the glans. DIF: Cognitive Level: Remembering (Knowledge) REF: dm. 691 MSC: Client Needs: General 4. When performing a genital examination on a 25-year-old man, the nurse notices deeply pigmented, wrinkled scrotal skin with large sebaceous follicles. On the basis of this information, the nurse would: a Squeeze the glans to check for the presence of discharge. . b Consider this finding as normal, and proceed with the examination. . c Assess the testicles for the presence of masses or painless lumps. . d Obtain a more detailed history, focusing on any scrotal abnormalities the patient . has noticed. ANS: B After adolescence, the scrotal skin is deeply pigmented and has large sebaceous follicles and appears corrugated. DIF: Cognitive Level: Applying (Application) REF: dm. 691 MSC: Client Needs: Health Promotion and Maintenance 5. Which statement concerning the testes is true? a The lymphatic vessels of the testes drain into the abdominal lymph nodes. . b The vas deferens is located along the inferior portion of each testis. . c The right testis is lower than the left because the right spermatic cord is longer. . d The cremaster muscle contracts in response to cold and draws the testicles . closer to the body. ANS: D When it is cold, the cremaster muscle contracts, which raises the scrotal sac and brings the testes closer to the body to absorb heat necessary for sperm viability. The lymphatic vessels of the testes drain into the inguinal lymph nodes. The vas deferens is located along the upper portion of each testis. The left testis is lower than the right because the left spermatic cord is longer. DIF: Cognitive Level: Remembering (Knowledge) REF: dm. 691 MSC: Client Needs: General 6. A male patient with possible fertility problems asks the nurse where sperm is produced. The nurse knows that sperm production occurs in the: a Testes. . b Prostate. . c Epididymis. . d Vas deferens. . ANS: A Sperm production occurs in the testes, not in the other structures listed. DIF: Cognitive Level: Remembering (Knowledge) REF: dm. 692 MSC: Client Needs: Physiologic Integrity 7. A 62-year-old man states that his physician told him that he has an inguinal hernia. He asks the nurse to explain what a hernia is. The nurse should: a Tell him not to worry and that most men his age develop hernias. . b . Explain that a hernia is often the result of prenatal growth abnormalities. c . Refer him to his physician for additional consultation because the physician made the initial diagnosis. d . Explain that a hernia is a loop of bowel protruding through a weak spot in the abdominal muscles. ANS: D A hernia is a loop of bowel protruding through a weak spot in the musculature. The other options are not correct responses to the patients question. DIF: Cognitive Level: Applying (Application) REF: dm. 692 MSC: Client Needs: Physiologic Integrity 8. The mother of a 10-year-old boy asks the nurse to discuss the recognition of puberty. The nurse should reply by saying: a Puberty usually begins around 15 years of age. . b The first sign of puberty is an enlargement of the testes. . c The penis size does not increase until about 16 years of age. . d The development of pubic hair precedes testicular or penis enlargement. . ANS: B Puberty begins sometime between age 9 for African Americans and age 10 for Caucasians and Hispanics. The first sign is an enlargement of the testes. Pubic hair appears next, and then penis size increases. DIF: Cognitive Level: Applying (Application) REF: dm. 693 MSC: Client Needs: Health Promotion and Maintenance 9. During an examination of an aging man, the nurse recognizes that normal changes to expect would be: a Enlarged scrotal sac. . b Increased pubic hair. . c Decreased penis size. . d Increased rugae over the scrotum. . ANS: C In the aging man, the amount of pubic hair decreases, the penis size decreases, and the rugae over the scrotal sac decreases. The scrotal sac does not enlarge. DIF: Cognitive Level: Understanding (Comprehension) REF: dm. 693 MSC: Client Needs: Health Promotion and Maintenance 10. An older man is concerned about his sexual performance. The nurse knows that in the absence of disease, a withdrawal from sexual activity later in life may be attributable to: a Side effects of medications. . b Decreased libido with aging. . c Decreased sperm production. . d Decreased pleasure from sexual intercourse. . ANS: A In the absence of disease, a withdrawal from sexual activity may be attributable to side effects of medications such as antihypertensives, antidepressants, sedatives, psychotropics, antispasmotics, tranquilizers or narcotics, and estrogens. The other options are not correct. DIF: Cognitive Level: Understanding (Comprehension) REF: dm. 693 MSC: Client Needs: Health Promotion and Maintenance 11. A 59-year-old patient has been diagnosed with prostatitis and is being seen at the clinic for complaints of burning and pain during urination. He is experiencing: a Dysuria. . b Nocturia. . c Polyuria. . d Hematuria. . ANS: A Dysuria (burning with urination) is common with acute cystitis, prostatitis, and urethritis. Nocturia is voiding during the night. Polyuria is voiding in excessive quantities. Hematuria is voiding with blood in the urine. DIF: Cognitive Level: Applying (Application) REF: dm. 695 MSC: Client Needs: Physiologic Integrity: Physiologic Adaptation 12. A 45-year-old mother of two children is seen at the clinic for complaints of losing my urine when I sneeze. The nurse documents that she is experiencing: a Urinary frequency. . b Enuresis. . c Stress incontinence. . d Urge incontinence. . ANS: C Stress incontinence is involuntary urine loss with physical strain, sneezing, or coughing that occurs as a result to weakness of the pelvic floor. Urinary frequency is urinating more times than usual (more than five to six times per day). Enuresis is involuntary passage of urine at night after age 5 to 6 years (bed wetting). Urge incontinence is involuntary urine loss from overactive detrusor muscle in the bladder. It contracts, causing an urgent need to void. DIF: Cognitive Level: Applying (Application) REF: dm. 696 MSC: Client Needs: Physiologic Integrity: Physiologic Adaptation 13. When the nurse is conducting sexual history from a male adolescent, which statement would be most appropriate to use at the beginning of the interview? a Do you use condoms? . b You dont masturbate, do you? . c Have you had sex in the last 6 months? . d Often adolescents your age have questions about sexual activity. . ANS: D The interview should begin with a permission statement, which conveys that it is normal and acceptable to think or feel a certain way. Sounding judgmental should be avoided. DIF: Cognitive Level: Analyzing (Analysis) REF: dm. 698 MSC: Client Needs: Health Promotion and Maintenance 14. Which of these statements is most appropriate when the nurse is obtaining a genitourinary history from an older man? a Do you need to get up at night to urinate? . b Do you experience nocturnal emissions, or wet dreams? . c Do you know how to perform a testicular self-examination? . d Has anyone ever touched your genitals when you did not want them to? . ANS: A The older male patient should be asked about the presence of nocturia. Awaking at night to urinate may be attributable to a diuretic medication, fluid retention from mild heart failure or varicose veins, or fluid ingestion 3 hours before bedtime, especially coffee and alcohol. The other questions are more appropriate for younger men. DIF: Cognitive Level: Analyzing (Analysis) REF: dm. 699 MSC: Client Needs: Health Promotion and Maintenance 15. When the nurse is performing a genital examination on a male patient, the patient has an erection. The nurses most appropriate action or response is to: a Ask the patient if he would like someone else to examine him. . b Continue with the examination as though nothing has happened. . c Stop the examination, leave the room while stating that the examination will . resume at a later time. d Reassure the patient that this is a normal response and continue with the . examination. ANS: D When the male patient has an erection, the nurse should reassure the patient that this is a normal physiologic response to touch and proceed with the rest of the examination. The other responses are not correct and may be perceived as judgmental. DIF: Cognitive Level: Applying (Application) REF: dm. 699 MSC: Client Needs: Psychosocial Integrity 16. The nurse is examining the glans and knows which finding is normal for this area? a The meatus may have a slight discharge when the glans is compressed. . b Hair is without pest inhabitants. . c The skin is wrinkled and without lesions. . d Smegma may be present under the foreskin of an uncircumcised male. . ANS: D The glans looks smooth and without lesions and does not have hair. The meatus should not have any discharge when the glans is compressed. Some cheesy smegma may have collected under the foreskin of an uncircumcised male. DIF: Cognitive Level: Understanding (Comprehension) REF: dm. 701 MSC: Client Needs: Safe and Effective Care Environment: Management of Care 17. When performing a genitourinary assessment, the nurse notices that the urethral meatus is ventrally positioned. This finding is: a Called hypospadias. . b A result of phimosis. . c Probably due to a stricture. . d Often associated with aging. . ANS: A Normally, the urethral meatus is positioned just about centrally. Hypospadias is the ventral location of the urethral meatus. The position of the meatus does not change with aging. Phimosis is the inability to retract the foreskin. A stricture is a narrow opening of the meatus. DIF: Cognitive Level: Applying (Application) REF: dm. 700 MSC: Client Needs: Safe and Effective Care Environment: Management of Care 18. The nurse is performing a genital examination on a male patient and notices urethral drainage. When collecting urethral discharge for microscopic examination and culture, the nurse should: a Ask the patient to urinate into a sterile cup. . b Ask the patient to obtain a specimen of semen. . c Insert a cotton-tipped applicator into the urethra. . d Compress the glans between the examiners thumb and forefinger, and collect . any discharge. ANS: D If urethral discharge is noticed, then the examiner should collect a smear for microscopic examination and culture by compressing the glans anteroposteriorly between the thumb and forefinger. The other options are not correct actions. DIF: Cognitive Level: Applying (Application) REF: dm. 701 MSC: Client Needs: Safe and Effective Care Environment: Management of Care 19. When assessing the scrotum of a male patient, the nurse notices the presence of multiple firm, nontender, yellow 1-cm nodules. The nurse knows that these nodules are most likely: a From urethritis. . b Sebaceous cysts. . c Subcutaneous plaques. . d From an inflammation of the epididymis. . ANS: B Sebaceous cysts are commonly found on the scrotum. These yellowish 1-cm nodules are firm, nontender, and often multiple. The other options are not correct. DIF: Cognitive Level: Analyzing (Analysis) REF: dm. 702 MSC: Client Needs: Physiologic Integrity: Physiologic Adaptation 20. When performing a scrotal assessment, the nurse notices that the scrotal contents show a red glow with transillumination. On the basis of this finding the nurse would: a Assess the patient for the presence of a hernia. . b Suspect the presence of serous fluid in the scrotum. . c Consider this finding normal, and proceed with the examination. . d Refer the patient for evaluation of a mass in the scrotum. . ANS: B Normal scrotal contents do not allow light to pass through the scrotum. However, serous fluid does transilluminate and shows as a red glow. Neither a mass nor a hernia would transilluminate. DIF: Cognitive Level: Analyzing (Analysis) REF: dm. 703 MSC: Client Needs: Physiologic Integrity: Physiologic Adaptation 21. When the nurse is performing a genital examination on a male patient, which action is correct? a Auscultating for the presence of a bruit over the scrotum . b Palpating for the vertical chain of lymph nodes along the groin, inferior to the . inguinal ligament c Palpating the inguinal canal only if a bulge is present in the inguinal region . during inspection d Having the patient shift his weight onto the left (unexamined) leg when . palpating for a hernia on the right side ANS: D When palpating for the presence of a hernia on the right side, the male patient is asked to shift his weight onto the left (unexamined) leg. Auscultating for a bruit over the scrotum is not appropriate. When palpating for lymph nodes, the horizontal chain is palpated. The inguinal canal should be palpated whether a bulge is present or not. DIF: Cognitive Level: Applying (Application) REF: dm. 706 MSC: Client Needs: Safe and Effective Care Environment: Management of Care 22. The nurse is aware of which statement to be true regarding the incidence of testicular cancer? a Testicular cancer is the most common cancer in men aged 30 to 50 years. . b The early symptoms of testicular cancer are pain and induration. . c Men with a history of cryptorchidism are at the greatest risk for the . development of testicular cancer. d The cure rate for testicular cancer is low. . ANS: C Men with undescended testicles (cryptorchidism) are at the greatest risk for the development of testicular cancer. The overall incidence of testicular cancer is rare. Although testicular cancer has no early symptoms, when detected early and treated before metastasizing, the cure rate is almost 100%. DIF: Cognitive Level: Understanding (Comprehension) REF: dm. 707 MSC: Client Needs: Health Promotion and Maintenance 23. The nurse is describing how to perform a testicular self-examination to a patient. Which statement is most appropriate? a A good time to examine your testicles is just before you take a shower. . b If you notice an enlarged testicle or a painless lump, call your health care . provider. c The testicle is egg shaped and movable. It feels firm and has a lumpy . consistency. d Perform a testicular examination at least once a week to detect the early stages . of testicular cancer. ANS: B If the patient notices a firm painless lump, a hard area, or an overall enlarged testicle, then he should call his health care provider for further evaluation. The testicle normally feels rubbery with a smooth surface. A good time to examine the testicles is during the shower or bath, when ones hands are warm and soapy and the scrotum is warm. Testicular self-examination should be performed once a month. DIF: Cognitive Level: Applying (Application) REF: dm. 704 MSC: Client Needs: Health Promotion and Maintenance 24. A 2-month-old uncircumcised infant has been brought to the clinic for a well-baby checkup. How would the nurse proceed with the genital examination? a Eliciting the cremasteric reflex is recommended. . b The glans is assessed for redness or lesions. . c Retracting the foreskin should be avoided until the infant is 3 months old. . d Any dirt or smegma that has collected under the foreskin should be noted. . ANS: C If uncircumcised, then the foreskin is normally tight during the first 3 months and should not be retracted because of the risk of tearing the membrane attaching the foreskin to the shaft. The other options are not correct. DIF: Cognitive Level: Applying (Application) REF: dm. 706 MSC: Client Needs: Safe and Effective Care Environment: Management of Care 25. A 2-year-old boy has been diagnosed with physiologic cryptorchidism. Considering this diagnosis, during assessment the nurse will most likely observe: a Testes that are hard and painful to palpation. . b Atrophic scrotum and a bilateral absence of the testis. . c Absence of the testis in the scrotum, but the testis can be milked down. . d Testes that migrate into the abdomen when the child squats or sits cross-legged. . ANS: C Migratory testes (physiologic cryptorchidism) are common because of the strength of the cremasteric reflex and the small mass of the prepubertal testes. The affected side has a normally developed scrotum and the testis can be milked down. The other responses are not correct. DIF: Cognitive Level: Applying (Application) REF: dm. 707 MSC: Client Needs: Physiologic Integrity: Physiologic Adaptation 26. The nurse knows that a common assessment finding in a boy younger than 2 years old is: a Inflamed and tender spermatic cord. . b . Presence of a hernia in the scrotum. c . Penis that looks large in relation to the scrotum. d . Presence of a hydrocele, or fluid in the scrotum. ANS: D A common scrotal finding in boys younger than 2 years of age is a hydrocele, or fluid in the scrotum. The other options are not correct. DIF: Cognitive Level: Applying (Application) REF: dm. 707 MSC: Client Needs: Health Promotion and Maintenance 27. During an examination of an aging man, the nurse recognizes that normal changes to expect would be: a Change in scrotal color. . b Decrease in the size of the penis. . c Enlargement of the testes and scrotum. . d Increase in the number of rugae over the scrotal sac. . ANS: B When assessing the genitals of an older man, the nurse may notice thinner, graying pubic hair and a decrease in the size of the penis. The size of the testes may be decreased, they may feel less firm, and the scrotal sac is pendulous with less rugae. No change in scrotal color is observed. DIF: Cognitive Level: Applying (Application) REF: dm. 708 MSC: Client Needs: Health Promotion and Maintenance 28. When performing a genital assessment on a middle-aged man, the nurse notices multiple soft, moist, painless papules in the shape of cauliflower-like patches scattered across the shaft of the penis. These lesions are characteristic of: a Carcinoma. . b Syphilitic chancres. . c Genital herpes. . d Genital warts. . ANS: D The lesions of genital warts are soft, pointed, moist, fleshy, painless papules that may be single or multiple in a cauliflower-like patch. They occur on the shaft of the penis, behind the corona, or around the anus, where they may grow into large grapelike clusters. (See Table 24-4 for more information and for the descriptions of the other options.) DIF: Cognitive Level: Analyzing (Analysis) REF: dm. 713 MSC: Client Needs: Physiologic Integrity: Physiologic Adaptation 29. A 15-year-old boy is seen in the clinic for complaints of dull pain and pulling in the scrotal area. On examination, the nurse palpates a soft, irregular mass posterior to and above the testis on the left. This mass collapses when the patient is supine and refills when he is upright. This description is consistent with: a Epididymitis. . b Spermatocele. . c Testicular torsion. . d Varicocele. . ANS: D A varicocele consists of dilated, tortuous varicose veins in the spermatic cord caused by incompetent valves within the vein. Symptoms include dull pain or a constant pulling or dragging feeling, or the individual may be asymptomatic. When palpating the mass, the examiner will feel a soft, irregular mass posterior to and above the testis that collapses when the individual is supine and refills when the individual is upright. (See Table 24-6 for more information and for the descriptions of the other options.) DIF: Cognitive Level: Analyzing (Analysis) REF: dm. 717 MSC: Client Needs: Physiologic Integrity: Physiologic Adaptation 30. When performing a genitourinary assessment on a 16-year-old male adolescent, the nurse notices a swelling in the scrotum that increases with increased intra-abdominal pressure and decreases when he is lying down. The patient complains of pain when straining. The nurse knows that this description is most consistent with a(n) hernia. a Femoral . b Incisional . c Direct inguinal . d Indirect inguinal . ANS: D With indirect inguinal hernias, pain occurs with straining and a soft swelling increases with increased intra-abdominal pressure, which may decrease when the patient lies down. These findings do not describe the other hernias. (See Table 24-7 for the descriptions of femoral, direct inguinal, and indirect inguinal hernias.) DIF: Cognitive Level: Analyzing (Analysis) REF: dm. 719 MSC: Client Needs: Physiologic Integrity: Physiologic Adaptation 31. When the nurse is performing a testicular examination on a 25-year-old man, which finding is considered normal? a Nontender subcutaneous plaques . b Scrotal area that is dry, scaly, and nodular . c Testes that feel oval and movable and are slightly sensitive to compression . d Single, hard, circumscribed, movable mass, less than 1 cm under the surface of . the testes ANS: C Testes normally feel oval, firm and rubbery, smooth, and bilaterally equal and are freely movable and slightly tender to moderate pressure. The scrotal skin should not be dry, scaly, or nodular or contain subcutaneous plaques. Any mass would be an abnormal finding. DIF: Cognitive Level: Applying (Application) REF: dm. 702 MSC: Client Needs: Safe and Effective Care Environment: Management of Care 32. The nurse is inspecting the scrotum and testes of a 43-year-old man. Which finding would require additional follow-up and evaluation? a Skin on the scrotum is taut. . b . Left testicle hangs lower than the right testicle. c . Scrotal skin has yellowish 1-cm nodules that are firm and nontender. d . Testes move closer to the body in response to cold temperatures. ANS: A Scrotal swelling may cause the skin to be taut and to display pitting edema. Normal scrotal skin is rugae, and asymmetry is normal with the left scrotal half usually lower than the right. The testes may move closer to the body in response to cold temperatures. DIF: Cognitive Level: Analyzing (Analysis) REF: dm. 718 MSC: Client Needs: Physiologic Integrity: Physiologic Adaptation 33. A 55-year-old man is experiencing severe pain of sudden onset in the scrotal area. It is somewhat relieved by elevation. On examination the nurse notices an enlarged, red scrotum that is very tender to palpation. Distinguishing the epididymis from the testis is difficult, and the scrotal skin is thick and edematous. This description is consistent with which of these? a Varicocele . b Epididymitis . c Spermatocele . d Testicular torsion . ANS: B Epididymitis presents as severe pain of sudden onset in the scrotum that is somewhat relieved by elevation. On examination, the scrotum is enlarged, reddened, and exquisitely tender. The epididymis is enlarged and indurated and may be hard to distinguish from the testis. The overlying scrotal skin may be thick and edematous. (See Table 24-6 for more information and for the descriptions of the other terms.) DIF: Cognitive Level: Analyzing (Analysis) REF: dm. 716 MSC: Client Needs: Physiologic Integrity: Physiologic Adaptation 34. The nurse is performing a genitourinary assessment on a 50-year-old obese male laborer. On examination, the nurse notices a painless round swelling close to the pubis in the area of the internal inguinal ring that is easily reduced when the individual is supine. These findings are most consistent with a(n) hernia. a Scrotal . b Femoral . c Direct inguinal . d Indirect inguinal . ANS: C Direct inguinal hernias occur most often in men over the age of 40 years. It is an acquired weakness brought on by heavy lifting, obesity, chronic cough, or ascites. The direct inguinal hernia is usually a painless, round swelling close to the pubis in the area of the internal inguinal ring that is easily reduced when the individual is supine. (See Table 24-6 for a description of scrotal hernia. See Table 24-7 for the descriptions of femoral hernias and indirect inguinal hernias.) DIF: Cognitive Level: Analyzing (Analysis) REF: dm. 719 MSC: Client Needs: Physiologic Integrity: Physiologic Adaptation 35. The nurse is providing patient teaching about an erectile dysfunction drug. One of the drugs potential side effects is prolonged, painful erection of the penis without sexual stimulation, which is known as: a Orchitis. . b Stricture. . c Phimosis. . d Priapism. . ANS: D Priapism is prolonged, painful erection of the penis without sexual desire. Orchitis is inflammation of the testes. Stricture is a narrowing of the opening of the urethral meatus. Phimosis is the inability to retract the foreskin. Chapter 10 Advanced Assessment of the Female Reproductive System MULTIPLE CHOICE 1. During a health history, a 22-year old woman asks, Can I get that vaccine for human papilloma virus (HPV)? I have genital warts and Id like them to go away! What is the nurses best response? a The HPV vaccine is for girls and women ages 9 to 26 years, so we can start that . today. b This vaccine is only for girls who have not yet started to become sexually . active. c Lets check with the physician to see if you are a candidate for this vaccine. . d The vaccine cannot protect you if you already have an HPV infection. . ANS: D The HPV vaccine is appropriate for girls and women age 9 to 26 years and is administered to prevent cervical cancer by preventing HPV infections before girls become sexually active. However, it cannot protect the woman if an HPV infection is already present. DIF: Cognitive Level: Analyzing (Analysis) REF: dm. 740 MSC: Client Needs: General 2. During an examination, the nurse observes a female patients vestibule and expects to see the: a Urethral meatus and vaginal orifice. . b Vaginal orifice and vestibular (Bartholin) glands. . c Urethral meatus and paraurethral (Skene) glands. . d Paraurethral (Skene) and vestibular (Bartholin) glands. . ANS: A The labial structures encircle a boat-shaped space, or cleft, termed the vestibule. Within the vestibule are numerous openings. The urethral meatus and vaginal orifice are visible. The ducts of the paraurethral (Skene) glands and the vestibular (Bartholin) glands are present but not visible. DIF: Cognitive Level: Understanding (Comprehension) REF: dm. 737 MSC: Client Needs: Physiologic Integrity: Physiologic Adaptation 3. During a speculum inspection of the vagina, the nurse would expect to see what at the end of the vaginal canal? a Cervix . b Uterus . c Ovaries . d Fallopian tubes . ANS: A At the end of the canal, the uterine cervix projects into the vagina. DIF: Cognitive Level: Remembering (Knowledge) REF: dm. 738 MSC: Client Needs: Physiologic Integrity: Physiologic Adaptation 4. The uterus is usually positioned tilting forward and superior to the bladder. This position is known as: a Anteverted and anteflexed. . b Retroverted and anteflexed. . c Retroverted and retroflexed. . d Superiorverted and anteflexed. . ANS: A The uterus is freely movable, not fixed, and usually tilts forward and superior to the bladder (a position labeled as anteverted and anteflexed). DIF: Cognitive Level: Remembering (Knowledge) REF: dm. 738 MSC: Client Needs: General 5. An 11-year-old girl is in the clinic for a sports physical examination. The nurse notices that she has begun to develop breasts, and during the conversation the girl reveals that she is worried about her development. The nurse should use which of these techniques to best assist the young girl in understanding the expected sequence for development? The nurse should: a Use the Tanner scale on the five stages of sexual development. . b . Describe her development and compare it with that of other girls her age. c . Use the Jacobsen table on expected development on the basis of height and weight data. d . Reassure her that her development is within normal limits and tell her not to worry about the next step. ANS: A The Tanner scale on the five stages of pubic hair development is helpful in teaching girls the expected sequence of sexual development (see Table 26-1). The other responses are not appropriate. DIF: Cognitive Level: Applying (Application) REF: dm. 739 MSC: Client Needs: Health Promotion and Maintenance 6. A woman who is 8 weeks pregnant is in the clinic for a checkup. The nurse reads on her chart that her cervix is softened and looks cyanotic. The nurse knows that the woman is exhibiting sign and sign. a Tanner; Hegar . b Hegar; Goodell . c Chadwick; Hegar . d Goodell; Chadwick . ANS: D Shortly after the first missed menstrual period, the female genitalia show signs of the growing fetus. The cervix softens (Goodell sign) at 4 to 6 weeks, and the vaginal mucosa and cervix look cyanotic (Chadwick sign) at 8 to 12 weeks. These changes occur because of increased vascularity and edema of the cervix and hypertrophy and hyperplasia of the cervical glands. Hegar sign occurs when the isthmus of the uterus softens at 6 to 8 weeks. Tanner sign is not a correct response. DIF: Cognitive Level: Understanding (Comprehension) REF: dm. 739 MSC: Client Needs: Health Promotion and Maintenance 7. Generally, the changes normally associated with menopause occur because the cells in the reproductive tract are: a Aging. . b Becoming fibrous. . c Estrogen dependent. . d Able to respond to estrogen. . ANS: C Because cells in the reproductive tract are estrogen dependent, decreased estrogen levels during menopause bring dramatic physical changes. The other options are not correct. DIF: Cognitive Level: Remembering (Knowledge) REF: dm. 740 MSC: Client Needs: Health Promotion and Maintenance 8. The nurse is reviewing the changes that occur with menopause. Which changes are associated with menopause? a Uterine and ovarian atrophy, along with a thinning of the vaginal epithelium . b Ovarian atrophy, increased vaginal secretions, and increasing clitoral size . c Cervical hypertrophy, ovarian atrophy, and increased acidity of vaginal . secretions d Vaginal mucosa fragility, increased acidity of vaginal secretions, and uterine . hypertrophy ANS: A The uterus shrinks because of its decreased myometrium. The ovaries atrophy to 1 to 2 cm and are not palpable after menopause. The sacral ligaments relax, and the pelvic musculature weakens; consequently, the uterus droops. The cervix shrinks and looks paler with a thick glistening epithelium. The vaginal epithelium atrophies, becoming thinner, drier, and itchy. The vaginal pH becomes more alkaline, and secretions are decreased, which results in a fragile mucosal surface that is at risk for vaginitis. DIF: Cognitive Level: Understanding (Comprehension) REF: dm. 740 MSC: Client Needs: Health Promotion and Maintenance 9. A 54-year-old woman who has just completed menopause is in the clinic today for a yearly physical examination. Which of these statements should the nurse include in patient education? A postmenopausal woman: a Is not at any greater risk for heart disease than a younger woman. . b Should be aware that she is at increased risk for dyspareunia because of . decreased vaginal secretions. c Has only stopped menstruating; there really are no other significant changes . with which she should be concerned. d Is likely to have difficulty with sexual pleasure as a result of drastic changes in . the female sexual response cycle. ANS: B Decreased vaginal secretions leave the vagina dry and at risk for irritation and pain with intercourse (dyspareunia). The other statements are incorrect. DIF: Cognitive Level: Applying (Application) REF: dm. 740 MSC: Client Needs: Health Promotion and Maintenance 10. A woman is in the clinic for an annual gynecologic examination. The nurse should plan to begin the interview with the: a Menstrual history, because it is generally nonthreatening. . b Obstetric history, because it includes the most important information. . c Urinary system history, because problems may develop in this area as well. . d Sexual history, because discussing it first will build rapport. . ANS: A Menstrual history is usually nonthreatening and therefore a good topic with which to begin the interview. Obstetric, urinary, and sexual histories are also part of the interview but not necessarily the best topics with which to start. DIF: Cognitive Level: Applying (Application) REF: dm. 740 MSC: Client Needs: Safe and Effective Care Environment: Management of Care 11. A patient has had three pregnancies and two live births. The nurse would record this information as grav , para , AB . a 2; 2; 1 . b 3; 2; 0 . c 3; 2; 1 . d 3; 3; 1 . ANS: C Gravida (grav) is the number of pregnancies. Para is the number of births. Abortions are interrupted pregnancies, including elective abortions and spontaneous miscarriages. DIF: Cognitive Level: Applying (Application) REF: dm. 741 MSC: Client Needs: Safe and Effective Care Environment: Management of Care 12. During the interview with a female patient, the nurse gathers data that indicate the patient is perimenopausal. Which of these statements made by this patient leads to this conclusion? a I have noticed that my muscles ache at night when I go to bed. . b I will be very happy when I can stop worrying about having a period. . c I have been noticing that I sweat a lot more than I used to, especially at night. . d I have only been pregnant twice, but both times I had breast tenderness as my . first symptom. ANS: C Hormone shifts occur during the perimenopausal period, and associated symptoms of menopause may occur, such as hot flashes, night sweats, numbness and tingling, headache, palpitations, drenching sweats, mood swings, vaginal dryness, and itching. The other responses are not correct. DIF: Cognitive Level: Analyzing (Analysis) REF: dm. 741 MSC: Client Needs: Health Promotion and Maintenance 13. A 50-year-old woman calls the clinic because she has noticed some changes in her body and breasts and wonders if these changes could be attributable to the hormone replacement therapy (HRT) she started 3 months earlier. The nurse should tell her: a HRT is at such a low dose that side effects are very unusual. . b HRT has several side effects, including fluid retention, breast tenderness, and . vaginal bleeding. c Vaginal bleeding with HRT is very unusual; I suggest you come into the clinic . immediately to have this evaluated. d It sounds as if your dose of estrogen is too high; I think you may need to . decrease the amount you are taking and then call back in a week. ANS: B Side effects of HRT include fluid retention, breast pain, and vaginal bleeding. The other responses are not correct. DIF: Cognitive Level: Applying (Application) REF: dm. 741 MSC: Client Needs: Physiologic Integrity: Pharmacologic and Parenteral Therapies 14. A 52-year-old patient states that when she sneezes or coughs she wets herself a little. She is very concerned that something may be wrong with her. The nurse suspects that the problem is: a Dysuria. . b Stress incontinence. . c Hematuria. . d Urge incontinence. . ANS: B Stress incontinence is involuntary urine loss with physical strain, sneezing, or coughing. Dysuria is pain or burning with urination. Hematuria is bleeding with urination. Urge incontinence is involuntary urine loss that occurs as a result of an overactive detrusor muscle in the bladder that contracts and causes an urgent need to void. DIF: Cognitive Level: Understanding (Comprehension) REF: dm. 742 MSC: Client Needs: Physiologic Integrity: Physiologic Adaptation 15. During the interview, a patient reveals that she has some vaginal discharge. She is worried that it may be a sexually transmitted infection. The nurses most appropriate response to this would be: a . Oh, dont worry. Some cyclic vaginal discharge is normal. b . Have you been engaging in unprotected sexual intercourse? c . Id like some information about the discharge. What color is it? d Have you had any urinary incontinence associated with the discharge? . ANS: C Questions that help the patient reveal more information about her symptoms should be asked in a nonthreatening manner. Asking about the amount, color, and odor of the vaginal discharge provides the opportunity for further assessment. Normal vaginal discharge is small, clear or cloudy, and always nonirritating. DIF: Cognitive Level: Analyzing (Analysis) REF: dm. 742 MSC: Client Needs: Physiologic Integrity: Physiologic Adaptation 16. A woman states that 2 weeks ago she had a urinary tract infection that was treated with an antibiotic. As a part of the interview, the nurse should ask, Have you noticed any: a . Changes in your urination patterns? b . Excessive vaginal bleeding? c . Unusual vaginal discharge or itching? d . Changes in your desire for intercourse? ANS: C Several medications may increase the risk of vaginitis. Broad-spectrum antibiotics alter the balance of normal flora, which may lead to the development of vaginitis. The other questions are not appropriate. DIF: Cognitive Level: Applying (Application) REF: dm. 742 MSC: Client Needs: Physiologic Integrity: Pharmacologic and Parenteral Therapies 17. Which statement would be most appropriate when the nurse is introducing the topic of sexual relationships during an interview? a . Now, it is time to talk about your sexual history. When did you first have intercourse? b . Women often feel dissatisfied with their sexual relationships. Would it be okay to discuss this now? c . Women often have questions about their sexual relationship and how it affects their health. Do you have any questions? d . Most women your age have had more than one sexual partner. How many would you say you have had? ANS: C The nurse should begin with an open-ended question to assess individual needs. The nurse should include appropriate questions as a routine part of the health history, because doing so communicates that the nurse accepts the individuals sexual activity and believes it is important. The nurses comfort with the discussion prompts the patients interest and, possibly, relief that the topic has been introduced. The initial discussion establishes a database for comparison with any future sexual activities and provides an opportunity to screen sexual problems. DIF: Cognitive Level: Applying (Application) REF: dm. 742 MSC: Client Needs: Psychosocial Integrity 18. A 22-year-old woman has been considering using oral contraceptives. As a part of her health history, the nurse should ask: a . Do you have a history of heart murmurs? b . Will you be in a monogamous relationship? c . Have you carefully thought this choice through? d . If you smoke, how many cigarettes do you smoke per day? ANS: D Oral contraceptives, together with cigarette smoking, increase the risk for cardiovascular side effects. If cigarettes are used, then the nurse should assess the patients smoking history. The other questions are not appropriate. DIF: Cognitive Level: Applying (Application) REF: dm. 743 MSC: Client Needs: Physiologic Integrity: Pharmacologic and Parenteral Therapies 19. A married couple has come to the clinic seeking advice on pregnancy. They have been trying to conceive for 4 months and have not been successful. What should the nurse do first? a . Ascertain whether either of them has been using broad-spectrum antibiotics. b . Explain that couples are considered infertile after 1 year of unprotected intercourse. c . Immediately refer the woman to an expert in pelvic inflammatory diseasethe most common cause of infertility. d . Explain that couples are considered infertile after 3 months of engaging in unprotected intercourse and that they will need a referral to a fertility expert. ANS: B Infertility is considered after 1 year of engaging in unprotected sexual intercourse without conceiving. The other actions are not appropriate. DIF: Cognitive Level: Applying (Application) REF: dm. 743 MSC: Client Needs: Psychosocial Integrity 20. A nurse is assessing a patients risk of contracting a sexually transmitted infection (STI). An appropriate question to ask would be: a . You know that its important to use condoms for protection, right? b . Do you use a condom with each episode of sexual intercourse? c . Do you have a sexually transmitted infection? d . You are aware of the dangers of unprotected sex, arent you? ANS: B In reviewing a patients risk for STIs, the nurse should ask in a nonconfrontational manner whether condoms are being used during each episode of sexual intercourse. Asking a person whether he or she has an infection does not address the risk. DIF: Cognitive Level: Understanding (Comprehension) REF: dm. 743 MSC: Client Needs: Physiologic Integrity: Reduction of Risk Potential 21. When the nurse is interviewing a preadolescent girl, which opening question would be least threatening? a . Do you have any questions about growing up? b . What has your mother told you about growing up? c . When did you notice that your body was changing? d . I remember being very scared when I got my period. How do you think youll feel? ANS: C Open-ended questions such as, When did you ? rather than Do you ? should be asked. Open- ended questions are less threatening because they imply that the topic is normal and unexceptional. DIF: Cognitive Level: Understanding (Comprehension) REF: dm. 743 MSC: Client Needs: Psychosocial Integrity 22. When the nurse is discussing sexuality and sexual issues with an adolescent, a permission statement helps convey that it is normal to think or feel a certain way. Which statement is the best example of a permission statement? a . It is okay that you have become sexually active. b . Girls your age often have questions about sexual activity. Do you have any questions? c . If it is okay with you, Id like to ask you some questions about your sexual history. d . Girls your age often engage in sexual activities. It is okay to tell me if you have had intercourse. ANS: B The examiner should start with a permission statement such as, Girls your age often experience A permission statement conveys the idea that it is normal to think or feel a certain way, and implying that the topic is normal and unexceptional is important. DIF: Cognitive Level: Understanding (Comprehension) REF: dm. 743 MSC: Client Needs: Psychosocial Integrity 23. The nurse is preparing to interview a postmenopausal woman. Which of these statements is true as it applies to obtaining the health history of a postmenopausal woman? a . The nurse should ask a postmenopausal woman if she has ever had vaginal bleeding. b . Once a woman reaches menopause, the nurse does not need to ask any history questions. c . The nurse should screen for monthly breast tenderness. d . Postmenopausal women are not at risk for contracting STIs; therefore, these questions can be omitted. ANS: A Postmenopausal bleeding warrants further workup and referral. The other statements are not true. DIF: Cognitive Level: Understanding (Comprehension) REF: dm. 744 MSC: Client Needs: Physiologic Integrity: Reduction of Risk Potential 24. During the examination portion of a patients visit, she will be in lithotomy position. Which statement reflects some things that the nurse can do to make this position more comfortable for her? a . Ask her to place her hands and arms over her head. b . Elevate her head and shoulders to maintain eye contact. c . Allow her to choose to have her feet in the stirrups or have them resting side by side on the edge of the table. d . Allow her to keep her buttocks approximately 6 inches from the edge of the table to prevent her from feeling as if she will fall off. ANS: B The nurse should elevate her head and shoulders to maintain eye contact. The patients arms should be placed at her sides or across the chest. Placing her hands and arms over her head only tightens the abdominal muscles. The feet should be placed into the stirrups, knees apart, and buttocks at the edge of the examining table. The stirrups are placed so that the legs are not abducted too far. DIF: Cognitive Level: Applying (Application) REF: dm. 745 MSC: Client Needs: Safe and Effective Care Environment: Management of Care 25. An 18-year-old patient is having her first pelvic examination. Which action by the nurse is appropriate? a . Inviting her mother to be present during the examination b . Avoiding the lithotomy position for this first time because it can be uncomfortable and embarrassing c . Raising the head of the examination table and giving her a mirror so that she can view the examination d . Fully draping her, leaving the drape between her legs elevated to avoid embarrassing her with eye contact ANS: C The techniques of the educational or mirror pelvic examination should be used. This is a routine examination with some modifications in attitude, position, and communication. First, the woman is considered an active participant, one who is interested in learning and in sharing decisions about her own health care. The woman props herself up on one elbow, or the head of the table is raised. Her other hand holds a mirror between her legs, above the examiners hands. The young woman can see all that the examiner is doing and has a full view of her genitalia. The mirror works well for teaching normal anatomy and its relationship to sexual behavior. The examiner can ask her if she would like to have a family member, friend, or chaperone present for the examination. The drape should be pushed down between the patients legs so that the nurse can see her face. DIF: Cognitive Level: Applying (Application) REF: dm. 746 MSC: Client Needs: Safe and Effective Care Environment: Management of Care 26. The nurse has just completed an inspection of a nulliparous womans external genitalia. Which of these would be a description of a finding within normal limits? a . Redness of the labia majora b . Multiple nontender sebaceous cysts c . Discharge that is foul smelling and irritating d . Gaping and slightly shriveled labia majora ANS: B No lesions should be noted, except for the occasional sebaceous cysts, which are yellowish 1-cm nodules that are firm, nontender, and often multiple. The labia majora are dark pink, moist, and symmetric; redness indicates inflammation or lesions. Discharge that is foul smelling and irritating may indicate infection. In the nulliparous woman, the labia majora meet in the midline, are symmetric and plump. DIF: Cognitive Level: Applying (Application) REF: dm. 747 MSC: Client Needs: Safe and Effective Care Environment: Management of Care 27. The nurse is preparing for an internal genitalia examination of a woman. Which order of the examination is correct? a . Bimanual, speculum, and rectovaginal b . Speculum, rectovaginal, and bimanual c . Speculum, bimanual, and rectovaginal d . Rectovaginal, bimanual, and speculum ANS: C The correct sequence is speculum examination, then bimanual examination after removing the speculum, and then rectovaginal examination. The examiner should change gloves before performing the rectovaginal examination to avoid spreading any possible infection. DIF: Cognitive Level: Analyzing (Analysis) REF: dm. 748 |dm. 754 |dm. 758 MSC: Client Needs: Safe and Effective Care Environment: Management of Care 28. During an internal examination of a womans genitalia, the nurse will use which technique for proper insertion of the speculum? a . The woman is instructed to bear down, the speculum blades are opened and applied in a swift, upward movement. b . The blades of the speculum are inserted on a horizontal plane, turning them to a 30-degree angle while continuing to insert them. The woman is asked to bear down after the speculum is inserted. c . The woman is instructed to bear down, the width of the blades are horizontally turned, and the speculum is inserted downward at a 45-degree angle toward the small of the womans back. d . The blades are locked open by turning the thumbscrew. Once the blades are open, pressure is applied to the introitus and the blades are inserted downward at a 45-degree angle to bring the cervix into view. ANS: C The examiner should instruct the woman to bear down, turn the width of the blades horizontally, and insert the speculum at a 45-degree angle downward toward the small of the womans back. (See the text under Speculum Examination for more detail.) DIF: Cognitive Level: Applying (Application) REF: dm. 749 MSC: Client Needs: Safe and Effective Care Environment: Management of Care 29. The nurse is examining a 35-year-old female patient. During the health history, the nurse notices that she has had two term pregnancies, and both babies were delivered vaginally. During the internal examination, the nurse observes that the cervical os is a horizontal slit with some healed lacerations and that the cervix has some nabothian cysts that are small, smooth, and yellow. In addition, the nurse notices that the cervical surface is granular and red, especially around the os. Finally, the nurse notices the presence of stringy, opaque, odorless secretions. Which of these findings are abnormal? a Nabothian cysts are present. . b The cervical os is a horizontal slit. . c The cervical surface is granular and red. . d Stringy and opaque secretions are present. . ANS: C Normal findings: Nabothian cysts may be present on the cervix after childbirth. The cervical os is a horizontal, irregular slit in the parous woman. Secretions vary according to the day of the menstrual cycle, and may be clear and thin or thick, opaque, and stringy. The surface is normally smooth, but cervical eversion, or ectropion, may occur where the endocervical canal is rolled out. Abnormal finding: The cervical surface should not be reddened or granular, which may indicate a lesion. DIF: Cognitive Level: Analyzing (Analysis) REF: dm. 750 MSC: Client Needs: Health Promotion and Maintenance 30. A patient calls the clinic for instructions before having a Papanicolaou (Pap) smear. The most appropriate instructions from the nurse are: a If you are menstruating, please use pads to avoid placing anything into the . vagina. b Avoid intercourse, inserting anything into the vagina, or douching within 24 . hours of your appointment. c If you suspect that you have a vaginal infection, please gather a sample of the . discharge to bring with you. d We would like you to use a mild saline douche before your examination. You . may pick this up in our office. ANS: B When instructing a patient before Pap smear is obtained, the nurse should follow these guidelines: Do not obtain during the womans menses or if a heavy infectious discharge is present. Instruct the woman not to douche, have intercourse, or put anything into the vagina within 24 hours before collecting the specimens. Any specimens will be obtained during the visit, not beforehand. DIF: Cognitive Level: Applying (Application) REF: dm. 752 MSC: Client Needs: Health Promotion and Maintenance 31. During an examination, which tests will the nurse collect to screen for cervical cancer? a Endocervical specimen, cervical scrape, and vaginal pool . b . Endocervical specimen, vaginal pool, and acetic acid wash c . Endocervical specimen, potassium hydroxide (KOH) preparation, and acetic acid wash d . Cervical scrape, acetic acid wash, saline mount (wet prep) ANS: A Laboratories may vary in method, but usually the test consists of three specimens: endocervical specimen, cervical scrape, and vaginal pool. The other tests (acetic acid wash, KOH preparation, and saline mount) are used to test for sexually transmitted infections. DIF: Cognitive Level: Understanding (Comprehension) REF: pp. 752-753 MSC: Client Needs: Safe and Effective Care Environment: Management of Care 32. When performing the bimanual examination, the nurse notices that the cervix feels smooth and firm, is round, and is fixed in place (does not move). When cervical palpation is performed, the patient complains of some pain. The nurses interpretation of these results should be which of these? a These findings are all within normal limits. . b Cervical consistency should be soft and velvetynot firm. . c The cervix should move when palpated; an immobile cervix may indicate . malignancy. d Pain may occur during palpation of the cervix. . ANS: C Normally, the cervix feels smooth and firm, similar to the consistency of the tip of the nose. It softens and feels velvety at 5 to 6 weeks of pregnancy (Goodell sign). The cervix should be evenly rounded. With a finger on either side, the examiner should be able to move the cervix gently from side to side, and doing so should produce no pain for the patient. Hardness of the cervix may occur with malignancy. Immobility may occur with malignancy, and pain may occur with inflammation or ectopic pregnancy. DIF: Cognitive Level: Analyzing (Analysis) REF: dm. 755 MSC: Client Needs: Health Promotion and Maintenance 33. The nurse is palpating a female patients adnexa. The findings include a firm, smooth uterine wall; the ovaries are palpable and feel smooth and firm. The fallopian tube is firm and pulsating. The nurses most appropriate course of action would be to: a Tell the patient that her examination is normal. . b Give her an immediate referral to a gynecologist. . c Suggest that she return in a month for a recheck to verify the findings. . d Tell the patient that she may have an ovarian cyst that should be evaluated . further. ANS: B Normally, the uterine wall feels firm and smooth, with the contour of the fundus rounded. Ovaries are not often palpable, but when they are, they normally feel smooth, firm, and almond shaped and are highly movable, sliding through the fingers. The fallopian tube is not normally palpable. No other mass or pulsation should be felt. Pulsation or palpable fallopian tube suggests ectopic pregnancy, which warrants immediate referral. DIF: Cognitive Level: Applying (Application) REF: dm. 757 MSC: Client Needs: Health Promotion and Maintenance 34. A 65-year-old woman is in the office for routine gynecologic care. She had a complete hysterectomy 3 months ago after cervical cancer was detected. Which statement does the nurse know to be true regarding this visit? a Her cervical mucosa will be red and dry looking. . b She will not need to have a Pap smear performed. . c The nurse can expect to find that her uterus will be somewhat enlarged and her . ovaries small and hard. d The nurse should plan to lubricate the instruments and the examining hand . adequately to avoid a painful examination. ANS: D In the aging adult woman, natural lubrication is decreased; therefore, to avoid a painful examination, the nurse should take care to lubricate the instruments and the examining hand adequately. Menopause, with the resulting decrease in estrogen production, shows numerous physical changes. The cervix shrinks and looks pale and glistening. With the bimanual examination, the uterus feels smaller and firmer and the ovaries are not normally palpable. Women should continue cervical cancer screening up to age 65 years if they have an intact cervix and are in good health. Women who have had a total hysterectomy do not need cervical cancer screening if they have 3 consecutive negative Pap tests or 2 or more consecutive negative HIV and Pap tests within the last 10 years. DIF: Cognitive Level: Applying (Application) REF: dm. 760 MSC: Client Needs: Health Promotion and Maintenance 35. The nurse is preparing to examine the external genitalia of a school-age girl. Which position would be most appropriate in this situation? a In the parents lap . b In a frog-leg position on the examining table . c In the lithotomy position with the feet in stirrups . d Lying flat on the examining table with legs extended . ANS: B For school-age children, placing them on the examining table in a frog-leg position is best. With toddlers and preschoolers, having the child on the parents lap in a frog-leg position is best. DIF: Cognitive Level: Applying (Application) REF: dm. 759 MSC: Client Needs: Health Promotion and Maintenance 36. When assessing a newborn infants genitalia, the nurse notices that the genitalia are somewhat engorged. The labia majora are swollen, the clitoris looks large, and the hymen is thick. The vaginal opening is difficult to visualize. The infants mother states that she is worried about the labia being swollen. The nurse should reply: a This is a normal finding in newborns and should resolve within a few weeks. . b This finding could indicate an abnormality and may need to be evaluated by a . physician. c We will need to have estrogen levels evaluated to ensure that they are within . normal limits. d We will need to keep close watch over the next few days to see if the genitalia . decrease in size. ANS: A It is normal for a newborns genitalia to be somewhat engorged. A sanguineous vaginal discharge or leukorrhea is normal during the first few weeks because of the maternal estrogen effect. During the early weeks, the genital engorgement resolves, and the labia minora atrophy and remain small until puberty. Chapter 11 Advanced Health Assessment of the Neurological System MULTIPLE CHOICE 1. The two parts of the nervous system are the: a Motor and sensory. . b Central and peripheral. . c Peripheral and autonomic. . d Hypothalamus and cerebral. . ANS: B The nervous system can be divided into two partscentral and peripheral. The central nervous system includes the brain and spinal cord. The peripheral nervous system includes the 12 pairs of cranial nerves (CNs), the 31 pairs of spinal nerves, and all of their branches. DIF: Cognitive Level: Remembering (Knowledge) REF: dm. 633 MSC: Client Needs: General 2. The wife of a 65-year-old man tells the nurse that she is concerned because she has noticed a change in her husbands personality and ability to understand. He also cries very easily and becomes angry. The nurse recalls that the cerebral lobe responsible for these behaviors is the lobe. a Frontal . b Parietal . c Occipital . d Temporal . ANS: A The frontal lobe has areas responsible for personality, behavior, emotions, and intellectual function. The parietal lobe has areas responsible for sensation; the occipital lobe is responsible for visual reception; and the temporal lobe is responsible for hearing, taste, and smell. DIF: Cognitive Level: Understanding (Comprehension) REF: dm. 633 MSC: Client Needs: Physiologic Integrity: Physiologic Adaptation 3. Which statement concerning the areas of the brain is true? a The cerebellum is the center for speech and emotions. . b The hypothalamus controls body temperature and regulates sleep. . c The basal ganglia are responsible for controlling voluntary movements. . d Motor pathways of the spinal cord and brainstem synapse in the thalamus. . ANS: B The hypothalamus is a vital area with many important functions: body temperature controller, sleep center, anterior and posterior pituitary gland regulator, and coordinator of autonomic nervous system activity and emotional status. The cerebellum controls motor coordination, equilibrium, and balance. The basal ganglia control autonomic movements of the body. The motor pathways of the spinal cord synapse in various areas of the spinal cord, not in the thalamus. DIF: Cognitive Level: Understanding (Comprehension) REF: dm. 634 MSC: Client Needs: General 4. The area of the nervous system that is responsible for mediating reflexes is the: a Medulla. . b Cerebellum. . c Spinal cord. . d Cerebral cortex. . ANS: C The spinal cord is the main highway for ascending and descending fiber tracts that connect the brain to the spinal nerves; it is responsible for mediating reflexes. DIF: Cognitive Level: Remembering (Knowledge) REF: dm. 635 MSC: Client Needs: General 5. While gathering equipment after an injection, a nurse accidentally received a prick from an improperly capped needle. To interpret this sensation, which of these areas must be intact? a Corticospinal tract, medulla, and basal ganglia . b Pyramidal tract, hypothalamus, and sensory cortex . c Lateral spinothalamic tract, thalamus, and sensory cortex . d Anterior spinothalamic tract, basal ganglia, and sensory cortex . ANS: C The spinothalamic tract contains sensory fibers that transmit the sensations of pain, temperature, and crude or light touch. Fibers carrying pain and temperature sensations ascend the lateral spinothalamic tract, whereas the sensations of crude touch form the anterior spinothalamic tract. At the thalamus, the fibers synapse with another sensory neuron, which carries the message to the sensory cortex for full interpretation. The other options are not correct. DIF: Cognitive Level: Applying (Application) REF: dm. 635 MSC: Client Needs: General 6. A patient with a lack of oxygen to his heart will have pain in his chest and possibly in the shoulder, arms, or jaw. The nurse knows that the best explanation why this occurs is which one of these statements? a A problem exists with the sensory cortex and its ability to discriminate the . location. b The lack of oxygen in his heart has resulted in decreased amount of oxygen to . the areas experiencing the pain. c The sensory cortex does not have the ability to localize pain in the heart; . consequently, the pain is felt elsewhere. d A lesion has developed in the dorsal root, which is preventing the sensation . from being transmitted normally. ANS: C The sensory cortex is arranged in a specific pattern, forming a corresponding map of the body. Pain in the right hand is perceived at a specific spot on the map. Some organs, such as the heart, liver, and spleen, are absent from the brain map. Pain originating in these organs is referred because no felt image exists in which to have pain. Pain is felt by proxy, that is, by another body part that does have a felt image. The other responses are not correct explanations. DIF: Cognitive Level: Understanding (Comprehension) REF: dm. 636 MSC: Client Needs: Physiologic Integrity: Basic Care and Comfort 7. The ability that humans have to perform very skilled movements such as writing is controlled by the: a Basal ganglia. . b Corticospinal tract. . c Spinothalamic tract. . d Extrapyramidal tract. . ANS: B Corticospinal fibers mediate voluntary movement, particularly very skilled, discrete, and purposeful movements, such as writing. The corticospinal tract, also known as the pyramidal tract, is a newer, higher motor system that humans have that permits very skilled and purposeful movements. The other responses are not related to skilled movements. DIF: Cognitive Level: Understanding (Comprehension) REF: dm. 636 MSC: Client Needs: General 8. A 30-year-old woman tells the nurse that she has been very unsteady and has had difficulty in maintaining her balance. Which area of the brain that is related to these findings would concern the nurse? a Thalamus . b Brainstem . c Cerebellum . d Extrapyramidal tract . ANS: C The cerebellar system coordinates movement, maintains equilibrium, and helps maintain posture. The thalamus is the primary relay station where sensory pathways of the spinal cord, cerebellum, and brainstem form synapses on their way to the cerebral cortex. The brainstem consists of the midbrain, pons, and medulla and has various functions, especially concerning autonomic centers. The extrapyramidal tract maintains muscle tone for gross automatic movements, such as walking. DIF: Cognitive Level: Understanding (Comprehension) REF: dm. 637 MSC: Client Needs: Physiologic Integrity: Physiologic Adaptation 9. Which of these statements about the peripheral nervous system is correct? a The CNs enter the brain through the spinal cord. . b Efferent fibers carry sensory input to the central nervous system through the . spinal cord. c The peripheral nerves are inside the central nervous system and carry impulses . through their motor fibers. d The peripheral nerves carry input to the central nervous system by afferent . fibers and away from the central nervous system by efferent fibers. ANS: D A nerve is a bundle of fibers outside of the central nervous system. The peripheral nerves carry input to the central nervous system by their sensory afferent fibers and deliver output from the central nervous system by their efferent fibers. The other responses are not related to the peripheral nervous system. DIF: Cognitive Level: Remembering (Knowledge) REF: dm. 637 MSC: Client Needs: General 10. A patient has a severed spinal nerve as a result of trauma. Which statement is true in this situation? a Because there are 31 pairs of spinal nerves, no effect results if only one nerve is . severed. b The dermatome served by this nerve will no longer experience any sensation. . c The adjacent spinal nerves will continue to carry sensations for the dermatome . served by the severed nerve. d A severed spinal nerve will only affect motor function of the patient because . spinal nerves have no sensory component. ANS: C A dermatome is a circumscribed skin area that is primarily supplied from one spinal cord segment through a particular spinal nerve. The dermatomes overlap, which is a form of biologic insurance; that is, if one nerve is severed, then most of the sensations can be transmitted by the spinal nerve above and the spinal nerve below the severed nerve. DIF: Cognitive Level: Applying (Application) REF: dm. 639 MSC: Client Needs: Physiologic Integrity: Physiologic Adaptation 11. A 21-year-old patient has a head injury resulting from trauma and is unconscious. There are no other injuries. During the assessment what would the nurse expect to find when testing the patients deep tendon reflexes? a Reflexes will be normal. . b Reflexes cannot be elicited. . c All reflexes will be diminished but present. . d Some reflexes will be present, depending on the area of injury. . ANS: A A reflex is a defense mechanism of the nervous system. It operates below the level of conscious control and permits a quick reaction to potentially painful or damaging situations. DIF: Cognitive Level: Applying (Application) REF: dm. 637 MSC: Client Needs: Safe and Effective Care Environment: Management of Care 12. A mother of a 1-month-old infant asks the nurse why it takes so long for infants to learn to roll over. The nurse knows that the reason for this is: a A demyelinating process must be occurring with her infant. . b Myelin is needed to conduct the impulses, and the neurons of a newborn are not . yet myelinated. c The cerebral cortex is not fully developed; therefore, control over motor . function gradually occurs. d The spinal cord is controlling the movement because the cerebellum is not yet . fully developed. ANS: B The infants sensory and motor development proceeds along with the gradual acquisition of myelin, which is needed to conduct most impulses. Very little cortical control exists, and the neurons are not yet myelinated. The other responses are not correct. DIF: Cognitive Level: Applying (Application) REF: dm. 640 MSC: Client Needs: Health Promotion and Maintenance 13. During an assessment of an 80-year-old patient, the nurse notices the following: an inability to identify vibrations at her ankle and to identify the position of her big toe, a slower and more deliberate gait, and a slightly impaired tactile sensation. All other neurologic findings are normal. The nurse should interpret that these findings indicate: a CN dysfunction. . b Lesion in the cerebral cortex. . c Normal changes attributable to aging. . d Demyelination of nerves attributable to a lesion. . ANS: C Some aging adults show a slower response to requests, especially for those calling for coordination of movements. The findings listed are normal in the absence of other significant abnormal findings. The other responses are incorrect. DIF: Cognitive Level: Analyzing (Analysis) REF: dm. 640 MSC: Client Needs: Health Promotion and Maintenance 14. A 70-year-old woman tells the nurse that every time she gets up in the morning or after shes been sitting, she gets really dizzy and feels like she is going to fall over. The nurses best response would be: a Have you been extremely tired lately? . b You probably just need to drink more liquids. . c Ill refer you for a complete neurologic examination. . d You need to get up slowly when youve been lying down or sitting. . ANS: D Aging is accompanied by a progressive decrease in cerebral blood flow. In some people, this decrease causes dizziness and a loss of balance with a position change. These individuals need to be taught to get up slowly. The other responses are incorrect. DIF: Cognitive Level: Analyzing (Analysis) REF: dm. 640 MSC: Client Needs: Health Promotion and Maintenance 15. During the taking of the health history, a patient tells the nurse that it feels like the room is spinning around me. The nurse would document this finding as: a Vertigo. . b Syncope. . c Dizziness. . d Seizure activity. . ANS: A True vertigo is rotational spinning caused by a neurologic dysfunction or a problem in the vestibular apparatus or the vestibular nuclei in the brainstem. Syncope is a sudden loss of strength or a temporary loss of consciousness. Dizziness is a lightheaded, swimming sensation. Seizure activity is characterized by altered or loss of consciousness, involuntary muscle movements, and sensory disturbances. DIF: Cognitive Level: Applying (Application) REF: dm. 641 MSC: Client Needs: Physiologic Integrity: Physiologic Adaptation 16. When taking the health history on a patient with a seizure disorder, the nurse assesses whether the patient has an aura. Which of these would be the best question for obtaining this information? a Does your muscle tone seem tense or limp? . b After the seizure, do you spend a lot of time sleeping? . c Do you have any warning sign before your seizure starts? . d Do you experience any color change or incontinence during the seizure? . ANS: C Aura is a subjective sensation that precedes a seizure; it could be auditory, visual, or motor. The other questions do not solicit information about an aura. DIF: Cognitive Level: Applying (Application) REF: dm. 641 MSC: Client Needs: Physiologic Integrity: Physiologic Adaptation 17. While obtaining a health history of a 3-month-old infant from the mother, the nurse asks about the infants ability to suck and grasp the mothers finger. What is the nurse assessing? a Reflexes . b Intelligence . c CNs . d Cerebral cortex function . ANS: A Questions regarding reflexes include such questions as, What have you noticed about the infants behavior, Are the infants sucking and swallowing seem coordinated, and Does the infant grasp your finger? The other responses are incorrect. DIF: Cognitive Level: Understanding (Comprehension) REF: dm. 642 MSC: Client Needs: Health Promotion and Maintenance 18. In obtaining a health history on a 74-year-old patient, the nurse notes that he drinks alcohol daily and that he has noticed a tremor in his hands that affects his ability to hold things. With this information, what response should the nurse make? a Does your family know you are drinking every day? . b Does the tremor change when you drink alcohol? . c Well do some tests to see what is causing the tremor. . d You really shouldnt drink so much alcohol; it may be causing your tremor. . ANS: B Senile tremor is relieved by alcohol, although not a recommended treatment. The nurse should assess whether the person is abusing alcohol in an effort to relieve the tremor. DIF: Cognitive Level: Analyzing (Analysis) REF: dm. 643 MSC: Client Needs: Health Promotion and Maintenance 19. A 50-year-old woman is in the clinic for weakness in her left arm and leg that she has noticed for the past week. The nurse should perform which type of neurologic examination? a Glasgow Coma Scale . b . Neurologic recheck examination c . Screening neurologic examination d . Complete neurologic examination ANS: D The nurse should perform a complete neurologic examination on an individual who has neurologic concerns (e.g., headache, weakness, loss of coordination) or who is showing signs of neurologic dysfunction. The Glasgow Coma Scale is used to define a persons level of consciousness. The neurologic recheck examination is appropriate for those who are demonstrating neurologic deficits. The screening neurologic examination is performed on seemingly well individuals who have no significant subjective findings from the health history. DIF: Cognitive Level: Applying (Application) REF: dm. 644 MSC: Client Needs: Health Promotion and Maintenance 20. During an assessment of the CNs, the nurse finds the following: asymmetry when the patient smiles or frowns, uneven lifting of the eyebrows, sagging of the lower eyelids, and escape of air when the nurse presses against the right puffed cheek. This would indicate dysfunction of which of these CNs? a Motor component of CN IV . b Motor component of CN VII . c Motor and sensory components of CN XI . d Motor component of CN X and sensory component of CN VII . ANS: B The findings listed reflect a dysfunction of the motor component of the facial nerve (CN VII). DIF: Cognitive Level: Analyzing (Analysis) REF: dm. 646 MSC: Client Needs: Health Promotion and Maintenance 21. The nurse is testing the function of CN XI. Which statement best describes the response the nurse should expect if this nerve is intact? The patient: a Demonstrates the ability to hear normal conversation. . b . Sticks out the tongue midline without tremors or deviation. c . Follows an object with his or her eyes without nystagmus or strabismus. d . Moves the head and shoulders against resistance with equal strength. ANS: D The following normal findings are expected when testing the spinal accessory nerve (CN XI): The patients sternomastoid and trapezius muscles are equal in size; the person can forcibly rotate the head both ways against resistance applied to the side of the chin with equal strength; and the patient can shrug the shoulders against resistance with equal strength on both sides. Checking the patients ability to hear normal conversation checks the function of CN VIII. Having the patient stick out the tongue checks the function of CN XII. Testing the eyes for nystagmus or strabismus is performed to check CNs III, IV, and VI. DIF: Cognitive Level: Applying (Application) REF: dm. 646 MSC: Client Needs: Health Promotion and Maintenance 22. During the neurologic assessment of a healthy 35-year-old patient, the nurse asks him to relax his muscles completely. The nurse then moves each extremity through full range of motion. Which of these results would the nurse expect to find? a Firm, rigid resistance to movement . b Mild, even resistance to movement . c Hypotonic muscles as a result of total relaxation . d Slight pain with some directions of movement . ANS: B Tone is the normal degree of tension (contraction) in voluntarily relaxed muscles. It shows a mild resistance to passive stretching. Normally, the nurse will notice a mild, even resistance to movement. The other responses are not correct. DIF: Cognitive Level: Applying (Application) REF: dm. 647 MSC: Client Needs: Health Promotion and Maintenance 23. When the nurse asks a 68-year-old patient to stand with his feet together and arms at his side with his eyes closed, he starts to sway and moves his feet farther apart. The nurse would document this finding as: a Ataxia. . b Lack of coordination. . c Negative Homans sign. . d Positive Romberg sign. . ANS: D Abnormal findings for the Romberg test include swaying, falling, and a widening base of the feet to avoid falling. A positive Romberg sign is a loss of balance that is increased by the closing of the eyes. Ataxia is an uncoordinated or unsteady gait. Homans sign is used to test the legs for deep-vein thrombosis. DIF: Cognitive Level: Analyzing (Analysis) REF: dm. 650 MSC: Client Needs: Health Promotion and Maintenance 24. The nurse is performing an assessment on a 29-year-old woman who visits the clinic complaining of always dropping things and falling down. While testing rapid alternating movements, the nurse notices that the woman is unable to pat both of her knees. Her response is extremely slow and she frequently misses. What should the nurse suspect? a Vestibular disease . b Lesion of CN IX . c Dysfunction of the cerebellum . d Inability to understand directions . ANS: C When a person tries to perform rapid, alternating movements, responses that are slow, clumsy, and sloppy are indicative of cerebellar disease. The other responses are incorrect. DIF: Cognitive Level: Analyzing (Analysis) REF: dm. 648 MSC: Client Needs: Physiologic Integrity: Physiologic Adaptation 25. During the taking of the health history of a 78-year-old man, his wife states that he occasionally has problems with short-term memory loss and confusion: He cant even remember how to button his shirt. When assessing his sensory system, which action by the nurse is most appropriate? a The nurse would not test the sensory system as part of the examination because . the results would not be valid. b The nurse would perform the tests, knowing that mental status does not affect . sensory ability. c The nurse would proceed with an explanation of each test, making certain that . the wife understands. d Before testing, the nurse would assess the patients mental status and ability to . follow directions. ANS: D The nurse should ensure the validity of the sensory system testing by making certain that the patient is alert, cooperative, comfortable, and has an adequate attention span. Otherwise, the nurse may obtain misleading and invalid results. DIF: Cognitive Level: Analyzing (Analysis) REF: dm. 644 MSC: Client Needs: Health Promotion and Maintenance 26. The assessment of a 60-year-old patient has taken longer than anticipated. In testing his pain perception, the nurse decides to complete the test as quickly as possible. When the nurse applies the sharp point of the pin on his arm several times, he is only able to identify these as one very sharp prick. What would be the most accurate explanation for this? a The patient has hyperesthesia as a result of the aging process. . b This response is most likely the result of the summation effect. . c The nurse was probably not poking hard enough with the pin in the other areas. . d The patient most likely has analgesia in some areas of arm and hyperalgesia in . others. ANS: B At least 2 seconds should be allowed to elapse between each stimulus to avoid summation. With summation, frequent consecutive stimuli are perceived as one strong stimulus. The other responses are incorrect. DIF: Cognitive Level: Analyzing (Analysis) REF: dm. 652 MSC: Client Needs: Safe and Effective Care Environment: Management of Care 27. The nurse is performing a neurologic assessment on a 41-year-old woman with a history of diabetes. When testing her ability to feel the vibrations of a tuning fork, the nurse notices that the patient is unable to feel vibrations on the great toe or ankle bilaterally, but she is able to feel vibrations on both patellae. Given this information, what would the nurse suspect? a Hyperalgesia . b Hyperesthesia . c Peripheral neuropathy . d Lesion of sensory cortex . ANS: C Loss of vibration sense occurs with peripheral neuropathy (e.g., diabetes and alcoholism). Peripheral neuropathy is worse at the feet and gradually improves as the examiner moves up the leg, as opposed to a specific nerve lesion, which has a clear zone of deficit for its dermatome. The other responses are incorrect. DIF: Cognitive Level: Analyzing (Analysis) REF: dm. 653 MSC: Client Needs: Physiologic Integrity: Physiologic Adaptation 28. The nurse places a key in the hand of a patient and he identifies it as a penny. What term would the nurse use to describe this finding? a Extinction . b Astereognosis . c Graphesthesia . d Tactile discrimination . ANS: B Stereognosis is the persons ability to recognize objects by feeling their forms, sizes, and weights. Astereognosis is an inability to identify objects correctly, and it occurs in sensory cortex lesions. Tactile discrimination tests fine touch. Extinction tests the persons ability to feel sensations on both sides of the body at the same point. DIF: Cognitive Level: Applying (Application) REF: dm. 654 MSC: Client Needs: Physiologic Integrity: Physiologic Adaptation 29. The nurse is testing the deep tendon reflexes of a 30-year-old woman who is in the clinic for an annual physical examination. When striking the Achilles heel and quadriceps muscle, the nurse is unable to elicit a reflex. The nurses next response should be to: a Ask the patient to lock her fingers and pull. . b Complete the examination, and then test these reflexes again. . c Refer the patient to a specialist for further testing. . d Document these reflexes as 0 on a scale of 0 to 4+. . ANS: A Sometimes the reflex response fails to appear. Documenting the reflexes as absent is inappropriate this soon in the examination. The nurse should try to further encourage relaxation, varying the persons position or increasing the strength of the blow. Reinforcement is another technique to relax the muscles and enhance the response. The person should be asked to perform an isometric exercise in a muscle group somewhat away from the one being tested. For example, to enhance a patellar reflex, the person should be asked to lock the fingers together and pull. DIF: Cognitive Level: Applying (Application) REF: dm. 656 MSC: Client Needs: Health Promotion and Maintenance 30. In assessing a 70-year-old patient who has had a recent cerebrovascular accident, the nurse notices right-sided weakness. What might the nurse expect to find when testing his reflexes on the right side? a Lack of reflexes . b Normal reflexes . c Diminished reflexes . d Hyperactive reflexes . ANS: D Hyperreflexia is the exaggerated reflex observed when the monosynaptic reflex arc is released from the influence of higher cortical levels. This response occurs with upper motor neuron lesions (e.g., a cerebrovascular accident). The other responses are incorrect. DIF: Cognitive Level: Applying (Application) REF: dm. 655 MSC: Client Needs: Physiologic Integrity: Physiologic Adaptation 31. When the nurse is testing the triceps reflex, what is the expected response? a Flexion of the hand . b Pronation of the hand . c Extension of the forearm . d Flexion of the forearm . ANS: C The normal response of the triceps reflex is extension of the forearm. The normal response of the biceps reflex causes flexion of the forearm. The other responses are incorrect. DIF: Cognitive Level: Remembering (Knowledge) REF: dm. 657 MSC: Client Needs: Safe and Effective Care Environment: Management of Care 32. The nurse is testing superficial reflexes on an adult patient. When stroking up the lateral side of the sole and across the ball of the foot, the nurse notices the plantar flexion of the toes. How should the nurse document this finding? a Positive Babinski sign . b Plantar reflex abnormal . c Plantar reflex present . d Plantar reflex 2+ on a scale from 0 to 4+ . ANS: C With the same instrument, the nurse should draw a light stroke up the lateral side of the sole of the foot and across the ball of the foot, similar to an upside-down J. The normal response is plantar flexion of the toes and sometimes of the entire foot. A positive Babinski sign is abnormal and occurs with the response of dorsiflexion of the big toe and fanning of all toes. The plantar reflex is not graded on a 0 to 4+ scale. DIF: Cognitive Level: Analyzing (Analysis) REF: dm. 660 MSC: Client Needs: Safe and Effective Care Environment: Management of Care 33. In the assessment of a 1-month-old infant, the nurse notices a lack of response to noise or stimulation. The mother reports that in the last week he has been sleeping all of the time, and when he is awake all he does is cry. The nurse hears that the infants cries are very high pitched and shrill. What should be the nurses appropriate response to these findings? a Refer the infant for further testing. . b Talk with the mother about eating habits. . c Do nothing; these are expected findings for an infant this age. . d Tell the mother to bring the baby back in 1 week for a recheck. . ANS: A A high-pitched, shrill cry or cat-sounding screech occurs with central nervous system damage. Lethargy, hyporeactivity, and hyperirritability, as well as the parents report of significant changes in behavior all warrant referral. The other options are not correct responses. DIF: Cognitive Level: Analyzing (Analysis) REF: dm. 661 MSC: Client Needs: Health Promotion and Maintenance 34. Which of these tests would the nurse use to check the motor coordination of an 11-month-old infant? a Denver II . b Stereognosis . c Deep tendon reflexes . d Rapid alternating movements . ANS: A To screen gross and fine motor coordination, the nurse should use the Denver II with its age- specific developmental milestones. Stereognosis tests a persons ability to recognize objects by feeling them and is not appropriate for an 11-month-old infant. Testing the deep tendon reflexes is not appropriate for checking motor coordination. Testing rapid alternating movements is appropriate for testing coordination in adults. DIF: Cognitive Level: Understanding (Comprehension) REF: dm. 662 MSC: Client Needs: Health Promotion and Maintenance 35. To assess the head control of a 4-month-old infant, the nurse lifts up the infant in a prone position while supporting his chest. The nurse looks for what normal response? The infant: a Raises the head, and arches the back. . b Extends the arms, and drops down the head. . c Flexes the knees and elbows with the back straight. . d Holds the head at 45 degrees, and keeps the back straight. . ANS: A At 3 months of age, the infant raises the head and arches the back as if in a swan dive. This response is the Landau reflex, which persists until 1 years of age (see Figure 23-43). The other responses are incorrect. DIF: Cognitive Level: Applying (Application) REF: dm. 663 MSC: Client Needs: Health Promotion and Maintenance 36. While assessing a 7-month-old infant, the nurse makes a loud noise and notices the following response: abduction and flexion of the arms and legs; fanning of the fingers, and curling of the index finger and thumb in a C position, followed by the infant bringing in the arms and legs to the body. What does the nurse know about this response? a This response could indicate brachial nerve palsy. . b This reaction is an expected startle response at this age. . c This reflex should have disappeared between 1 and 4 months of age. . d This response is normal as long as the movements are bilaterally symmetric. . ANS: C The Moro reflex is present at birth and usually disappears at 1 to 4 months. Absence of the Moro reflex in the newborn or its persistence after 5 months of age indicates severe central nervous system injury. The other responses are incorrect. DIF: Cognitive Level: Analyzing (Analysis) REF: dm. 666 MSC: Client Needs: Health Promotion and Maintenance 37. To test for gross motor skill and coordination of a 6-year-old child, which of these techniques would be appropriate? Ask the child to: a Hop on one foot. . b Stand on his head. . c Touch his finger to his nose. . d Make funny faces at the nurse. . ANS: A Normally, a child can hop on one foot and can balance on one foot for approximately 5 seconds by 4 years of age and can balance on one foot for 8 to 10 seconds at 5 years of age. Children enjoy performing these tests. Failure to hop after 5 years of age indicates incoordination of gross motor skills. Asking the child to touch his or her finger to the nose checks fine motor coordination; and asking the child to make funny faces tests CN VII. Asking a child to stand on his or her head is not appropriate. DIF: Cognitive Level: Applying (Application) REF: dm. 668 MSC: Client Needs: Health Promotion and Maintenance 38. During the assessment of an 80-year-old patient, the nurse notices that his hands show tremors when he reaches for something and his head is always nodding. No associated rigidity is observed with movement. Which of these statements is most accurate? a These findings are normal, resulting from aging. . b These findings could be related to hyperthyroidism. . c These findings are the result of Parkinson disease. . d This patient should be evaluated for a cerebellar lesion. . ANS: A Senile tremors occasionally occur. These benign tremors include an intention tremor of the hands, head nodding (as if saying yes or no), and tongue protrusion. Tremors associated with Parkinson disease include rigidity, slowness, and a weakness of voluntary movement. The other responses are incorrect. Chapter 12 Advanced Health Assessment of the Musculoskeletal System MULTIPLE CHOICE 1. A patient is being assessed for range-of-joint movement. The nurse asks him to move his arm in toward the center of his body. This movement is called: a Flexion. . b Abduction. . c Adduction. . d Extension. . ANS: C Moving a limb toward the midline of the body is called adduction; moving a limb away from the midline of the body is called abduction. Flexion is bending a limb at a joint; and extension is straightening a limb at a joint. DIF: Cognitive Level: Understanding (Comprehension) REF: dm. 578 MSC: Client Needs: Physiologic Integrity: Physiologic Adaptation 2. A patient tells the nurse that she is having a hard time bringing her hand to her mouth when she eats or tries to brush her teeth. The nurse knows that for her to move her hand to her mouth, she must perform which movement? a Flexion . b Abduction . c Adduction . d Extension . ANS: A Flexion, or bending a limb at a joint, is required to move the hand to the mouth. Extension is straightening a limb at a joint. Moving a limb toward the midline of the body is called adduction; abduction is moving a limb away from the midline of the body. DIF: Cognitive Level: Understanding (Comprehension) REF: dm. 578 MSC: Client Needs: Physiologic Integrity: Physiologic Adaptation 3. The functional units of the musculoskeletal system are the: a Joints. . b Bones. . c Muscles. . d Tendons. . ANS: A Joints are the functional units of the musculoskeletal system because they permit the mobility needed to perform the activities of daily living. The skeleton (bones) is the framework of the body. The other options are not correct. DIF: Cognitive Level: Remembering (Knowledge) REF: dm. 577 MSC: Client Needs: General 4. When reviewing the musculoskeletal system, the nurse recalls that hematopoiesis takes place in the: a Liver. . b Spleen. . c Kidneys. . d Bone marrow. . ANS: D The musculoskeletal system functions to encase and protect the inner vital organs, to support the body, to produce red blood cells in the bone marrow (hematopoiesis), and to store minerals. The other options are not correct. DIF: Cognitive Level: Remembering (Knowledge) REF: dm. 577 MSC: Client Needs: General 5. Fibrous bands running directly from one bone to another that strengthen the joint and help prevent movement in undesirable directions are called: a Bursa. . b Tendons. . c Cartilage. . d Ligaments. . ANS: D Fibrous bands running directly from one bone to another that strengthen the joint and help prevent movement in undesirable directions are called ligaments. The other options are not correct. DIF: Cognitive Level: Remembering (Knowledge) REF: dm. 577 MSC: Client Needs: General 6. The nurse notices that a woman in an exercise class is unable to jump rope. The nurse is aware that to jump rope, ones shoulder has to be capable of: a Inversion. . b Supination. . c Protraction. . d Circumduction. . ANS: D Circumduction is defined as moving the arm in a circle around the shoulder. The other options are not correct. DIF: Cognitive Level: Applying (Application) REF: dm. 578 MSC: Client Needs: Physiologic Integrity: Physiologic Adaptation 7. The articulation of the mandible and the temporal bone is known as the: a . Intervertebral foramen. b . Condyle of the mandible. c . Temporomandibular joint. d Zygomatic arch of the temporal bone. . ANS: C The articulation of the mandible and the temporal bone is the temporomandibular joint. The other responses are not correct. DIF: Cognitive Level: Remembering (Knowledge) REF: dm. 578 MSC: Client Needs: General 8. To palpate the temporomandibular joint, the nurses fingers should be placed in the depression of the ear. a . Distal to the helix b . Proximal to the helix c . Anterior to the tragus d . Posterior to the tragus ANS: C The temporomandibular joint can be felt in the depression anterior to the tragus of the ear. The other locations are not correct. DIF: Cognitive Level: Understanding (Comprehension) REF: dm. 578 MSC: Client Needs: Safe and Effective Care Environment: Management of Care 9. Of the 33 vertebrae in the spinal column, there are: a 5 lumbar. . b 5 thoracic. . c 7 sacral. . d 12 cervical. . ANS: A There are 7 cervical, 12 thoracic, 5 lumbar, 5 sacral, and 3 to 4 coccygeal vertebrae in the spinal column. DIF: Cognitive Level: Remembering (Knowledge) REF: dm. 579 MSC: Client Needs: General 10. An imaginary line connecting the highest point on each iliac crest would cross the vertebra. a First sacral . b Fourth lumbar . c Seventh cervical . d Twelfth thoracic . ANS: B An imaginary line connecting the highest point on each iliac crest crosses the fourth lumbar vertebra. The other options are not correct. DIF: Cognitive Level: Remembering (Knowledge) REF: dm. 579 MSC: Client Needs: General 11. The nurse is explaining to a patient that there are shock absorbers in his back to cushion the spine and to help it move. The nurse is referring to his: a Vertebral column. . b Nucleus pulposus. . c Vertebral foramen. . d Intervertebral disks. . ANS: D Intervertebral disks are elastic fibrocartilaginous plates that cushion the spine similar to shock absorbers and help it move. The vertebral column is the spinal column itself. The nucleus pulposus is located in the center of each disk. The vertebral foramen is the channel, or opening, for the spinal cord in the vertebrae. DIF: Cognitive Level: Understanding (Comprehension) REF: dm. 580 MSC: Client Needs: Physiologic Integrity: Physiologic Adaptation 12. The nurse is providing patient education for a man who has been diagnosed with a rotator cuff injury. The nurse knows that a rotator cuff injury involves the: a Nucleus pulposus. . b Articular processes. . c Medial epicondyle. . d Glenohumeral joint. . ANS: D A rotator cuff injury involves the glenohumeral joint, which is enclosed by a group of four powerful muscles and tendons that support and stabilize it. The nucleus pulposus is located in the center of each intervertebral disk. The articular processes are projections in each vertebral disk that lock onto the next vertebra, thereby stabilizing the spinal column. The medial epicondyle is located at the elbow. DIF: Cognitive Level: Applying (Application) REF: dm. 581 MSC: Client Needs: Physiologic Integrity: Physiologic Adaptation 13. During an interview the patient states, I can feel this bump on the top of both of my shouldersit doesnt hurt but I am curious about what it might be. The nurse should tell the patient that it is his: a Subacromial bursa. . b Acromion process. . c Glenohumeral joint. . d Greater tubercle of the humerus. . ANS: B The bump of the scapulas acromion process is felt at the very top of the shoulder. The other options are not correct. DIF: Cognitive Level: Applying (Application) REF: dm. 581 MSC: Client Needs: Physiologic Integrity: Physiologic Adaptation 14. The nurse is checking the range of motion in a patients knee and knows that the knee is capable of which movement(s)? a Flexion and extension . b Supination and pronation . c Circumduction . d Inversion and eversion . ANS: A The knee is a hinge joint, permitting flexion and extension of the lower leg on a single plane. The knee is not capable of the other movements listed. DIF: Cognitive Level: Understanding (Comprehension) REF: dm. 583 MSC: Client Needs: Physiologic Integrity: Physiologic Adaptation 15. A patient is visiting the clinic for an evaluation of a swollen, painful knuckle. The nurse notices that the knuckle above his ring on the left hand is swollen and that he is unable to remove his wedding ring. This joint is called the joint. a Interphalangeal . b Tarsometatarsal . c Metacarpophalangeal . d Tibiotalar . ANS: C The joint located just above the ring on the finger is the metacarpophalangeal joint. The interphalangeal joint is located distal to the metacarpophalangeal joint. The tarsometatarsal and tibiotalar joints are found in the foot and ankle. (See Figure 22-10 for a diagram of the bones and joints of the hand and fingers.) DIF: Cognitive Level: Understanding (Comprehension) REF: dm. 582 MSC: Client Needs: Physiologic Integrity: Physiologic Adaptation 16. The nurse is assessing a patients ischial tuberosity. To palpate the ischial tuberosity, the nurse knows that it is best to have the patient: a Standing. . b Flexing the hip. . c Flexing the knee. . d Lying in the supine position. . ANS: B The ischial tuberosity lies under the gluteus maximus muscle and is palpable when the hip is flexed. The other options are not correct. DIF: Cognitive Level: Understanding (Comprehension) REF: dm. 582 MSC: Client Needs: Safe and Effective Care Environment: Management of Care 17. The nurse is examining the hip area of a patient and palpates a flat depression on the upper, lateral side of the thigh when the patient is standing. The nurse interprets this finding as the: a Ischial tuberosity. . b Greater trochanter. . c Iliac crest. . d Gluteus maximus muscle. . ANS: B The greater trochanter of the femur is palpated when the person is standing, and it appears as a flat depression on the upper lateral side of the thigh. The iliac crest is the upper part of the hip bone; the ischial tuberosity lies under the gluteus maximus muscle and is palpable when the hip is flexed; and the gluteus muscle is part of the buttocks. DIF: Cognitive Level: Understanding (Comprehension) REF: dm. 582 MSC: Client Needs: Physiologic Integrity: Physiologic Adaptation 18. The ankle joint is the articulation of the tibia, fibula, and: a Talus. . b . Cuboid. c . Calcaneus. d . Cuneiform bones. ANS: A The ankle or tibiotalar joint is the articulation of the tibia, fibula, and talus. The other bones listed are foot bones and not part of the ankle joint. DIF: Cognitive Level: Remembering (Knowledge) REF: dm. 583 MSC: Client Needs: General 19. The nurse is explaining the mechanism of the growth of long bones to a mother of a toddler. Where does lengthening of the bones occur? a Bursa . b Calcaneus . c Epiphyses . d Tuberosities . ANS: C Lengthening occurs at the epiphyses, or growth plates. The other options are not correct. DIF: Cognitive Level: Understanding (Comprehension) REF: dm. 584 MSC: Client Needs: Health Promotion and Maintenance 20. A woman who is 8 months pregnant comments that she has noticed a change in her posture and is having lower back pain. The nurse tells her that during pregnancy, women have a posture shift to compensate for the enlarging fetus. This shift in posture is known as: a Lordosis. . b Scoliosis. . c Ankylosis. . d Kyphosis. . ANS: A Lordosis compensates for the enlarging fetus, which would shift the center of balance forward. This shift in balance, in turn, creates a strain on the low back muscles, felt as low back pain during late pregnancy by some women. Scoliosis is lateral curvature of portions of the spine; ankylosis is extreme flexion of the wrist, as observed with severe rheumatoid arthritis; and kyphosis is an enhanced thoracic curvature of the spine. DIF: Cognitive Level: Understanding (Comprehension) REF: dm. 584 MSC: Client Needs: Health Promotion and Maintenance 21. An 85-year-old patient comments during his annual physical examination that he seems to be getting shorter as he ages. The nurse should explain that decreased height occurs with aging because: a Long bones tend to shorten with age. . b The vertebral column shortens. . c A significant loss of subcutaneous fat occurs. . d A thickening of the intervertebral disks develops. . ANS: B Postural changes are evident with aging; decreased height is most noticeable and is due to shortening of the vertebral column. Long bones do not shorten with age. Intervertebral disks actually get thinner with age. Subcutaneous fat is not lost but is redistributed to the abdomen and hips. DIF: Cognitive Level: Applying (Application) REF: pp. 584-585 MSC: Client Needs: Health Promotion and Maintenance 22. A patient has been diagnosed with osteoporosis and asks the nurse, What is osteoporosis? The nurse explains that osteoporosis is defined as: a Increased bone matrix. . b Loss of bone density. . c New, weaker bone growth. . d Increased phagocytic activity. . ANS: B After age 40 years, a loss of bone matrix (resorption) occurs more rapidly than new bone formation. The net effect is a gradual loss of bone density, or osteoporosis. The other options are not correct. DIF: Cognitive Level: Remembering (Knowledge) REF: dm. 584 MSC: Client Needs: Physiologic Integrity: Physiologic Adaptation 23. The nurse is teaching a class on preventing osteoporosis to a group of perimenopausal women. Which of these actions is the best way to prevent or delay bone loss in this group? a Taking calcium and vitamin D supplements . b Taking medications to prevent osteoporosis . c Performing physical activity, such as fast walking . d Assessing bone density annually . ANS: C Physical activity, such as fast walking, delays or prevents bone loss in perimenopausal women. The faster the pace of walking, the higher the preventive effect is on the risk of hip fracture. The other options are not correct. DIF: Cognitive Level: Applying (Application) REF: dm. 585 MSC: Client Needs: Health Promotion and Maintenance 24. A teenage girl has arrived complaining of pain in her left wrist. She was playing basketball when she fell and landed on her left hand. The nurse examines her hand and would expect a fracture if the girl complains of a: a . Dull ache. b . Deep pain in her wrist. c . Sharp pain that increases with movement. d Dull throbbing pain that increases with rest. . ANS: C A fracture causes sharp pain that increases with movement. The other types of pain do not occur with a fracture. DIF: Cognitive Level: Analyzing (Analysis) REF: dm. 586 MSC: Client Needs: Physiologic Integrity: Physiologic Adaptation 25. A patient is complaining of pain in his joints that is worse in the morning, better after he moves around for a while, and then gets worse again if he sits for long periods. The nurse should assess for other signs of what problem? a . Tendinitis b . Osteoarthritis c . Rheumatoid arthritis d . Intermittent claudication ANS: C Rheumatoid arthritis is worse in the morning when a person arises. Movement increases most joint pain, except the pain with rheumatoid arthritis, which decreases with movement. The other options are not correct. DIF: Cognitive Level: Analyzing (Analysis) REF: dm. 586 MSC: Client Needs: Physiologic Integrity: Physiologic Adaptation 26. A patient states, I can hear a crunching or grating sound when I kneel. She also states that it is very difficult to get out of bed in the morning because of stiffness and pain in my joints. The nurse should assess for signs of what problem? a . Crepitation b . Bone spur c . Loose tendon d . Fluid in the knee joint ANS: A Crepitation is an audible and palpable crunching or grating that accompanies movement and occurs when articular surfaces in the joints are roughened, as with rheumatoid arthritis. The other options are not correct. DIF: Cognitive Level: Analyzing (Analysis) REF: dm. 590 MSC: Client Needs: Physiologic Integrity: Physiologic Adaptation 27. A patient is able to flex his right arm forward without difficulty or pain but is unable to abduct his arm because of pain and muscle spasms. The nurse should suspect: a . Crepitation. b . Rotator cuff lesions. c . Dislocated shoulder. d . Rheumatoid arthritis. ANS: B Rotator cuff lesions may limit range of motion and cause pain and muscle spasms during abduction, whereas forward flexion remains fairly normal. The other options are not correct. DIF: Cognitive Level: Analyzing (Analysis) REF: dm. 594 MSC: Client Needs: Physiologic Integrity: Physiologic Adaptation 28. A professional tennis player comes into the clinic complaining of a sore elbow. The nurse will assess for tenderness at the: a . Olecranon bursa. b . Annular ligament. c . Base of the radius. d . Medial and lateral epicondyle. ANS: D The epicondyles, the head of the radius, and the tendons are common sites of inflammation and local tenderness, commonly referred to as tennis elbow. The other locations are not affected. DIF: Cognitive Level: Analyzing (Analysis) REF: dm. 595 MSC: Client Needs: Physiologic Integrity: Physiologic Adaptation 29. The nurse suspects that a patient has carpal tunnel syndrome and wants to perform the Phalen test. To perform this test, the nurse should instruct the patient to: a . Dorsiflex the foot. b . Plantarflex the foot. c . Hold both hands back to back while flexing the wrists 90 degrees for 60 seconds. d . Hyperextend the wrists with the palmar surface of both hands touching, and wait for 60 seconds. ANS: C For the Phalen test, the nurse should ask the person to hold both hands back to back while flexing the wrists 90 degrees. Acute flexion of the wrist for 60 seconds produces no symptoms in the normal hand. The Phalen test reproduces numbness and burning in a person with carpal tunnel syndrome. The other actions are not correct when testing for carpal tunnel syndrome. DIF: Cognitive Level: Applying (Application) REF: dm. 599 MSC: Client Needs: Physiologic Integrity: Physiologic Adaptation 30. An 80-year-old woman is visiting the clinic for a checkup. She states, I cant walk as much as I used to. The nurse is observing for motor dysfunction in her hip and should ask her to: a . Internally rotate her hip while she is sitting. b . Abduct her hip while she is lying on her back. c . Adduct her hip while she is lying on her back. d . Externally rotate her hip while she is standing. ANS: B Limited abduction of the hip while supine is the most common motion dysfunction found in hip disease. The other options are not correct. DIF: Cognitive Level: Analyzing (Analysis) REF: dm. 600 MSC: Client Needs: Physiologic Integrity: Physiologic Adaptation 31. The nurse has completed the musculoskeletal examination of a patients knee and has found a positive bulge sign. The nurse interprets this finding to indicate: a . Irregular bony margins. b . Soft-tissue swelling in the joint. c . Swelling from fluid in the epicondyle. d . Swelling from fluid in the suprapatellar pouch. ANS: D A positive bulge sign confirms the presence of swelling caused by fluid in the suprapatellar pouch. The other options are not correct. DIF: Cognitive Level: Analyzing (Analysis) REF: dm. 602 MSC: Client Needs: Physiologic Integrity: Physiologic Adaptation 32. During an examination, the nurse asks a patient to bend forward from the waist and notices that the patient has lateral tilting. When his leg is raised straight up, the patient complains of a pain going down his buttock into his leg. The nurse suspects: a . Scoliosis. b . Meniscus tear. c . Herniated nucleus pulposus. d . Spasm of paravertebral muscles. ANS: C Lateral tilting and sciatic pain with straight leg raising are findings that occur with a herniated nucleus pulposus. The other options are not correct. DIF: Cognitive Level: Analyzing (Analysis) REF: dm. 606 |dm. 609 MSC: Client Needs: Physiologic Integrity: Physiologic Adaptation 33. The nurse is examining a 3-month-old infant. While the nurse holds his or her thumbs on the infants inner mid thighs and the fingers on the outside of the infants hips, touching the greater trochanter, the nurse adducts the legs until the his or her thumbs touch and then abducts the legs until the infants knees touch the table. The nurse does not notice any clunking sounds and is confident to record a: a . Positive Allis test. b . Negative Allis test. c . Positive Ortolani sign. d . Negative Ortolani sign. ANS: D Normally, this maneuver feels smooth and has no sound. With a positive Ortolani sign, however, the nurse will feel and hear a clunk, as the head of the femur pops back into place. A positive Ortolani sign also reflects hip instability. The Allis test also tests for hip dislocation but is performed by comparing leg lengths. DIF: Cognitive Level: Analyzing (Analysis) REF: dm. 611 MSC: Client Needs: Physiologic Integrity: Physiologic Adaptation 34. During a neonatal examination, the nurse notices that the newborn infant has six toes. This finding is documented as: a . Unidactyly. b . Syndactyly. c . Polydactyly. d . Multidactyly. ANS: C Polydactyly is the presence of extra fingers or toes. Syndactyly is webbing between adjacent fingers or toes. The other terms are not correct. DIF: Cognitive Level: Understanding (Comprehension) REF: dm. 611 MSC: Client Needs: Physiologic Integrity: Physiologic Adaptation 35. A mother brings her newborn baby boy in for a checkup; she tells the nurse that he does not seem to be moving his right arm as much as his left and that he seems to have pain when she lifts him up under the arms. The nurse suspects a fractured clavicle and would observe for: a . Negative Allis test. b . Positive Ortolani sign. c . Limited range of motion during the Moro reflex. d . Limited range of motion during Lasgue test. ANS: C For a fractured clavicle, the nurse should observe for limited arm range of motion and unilateral response to the Moro reflex. The other tests are not appropriate for this type of fracture. DIF: Cognitive Level: Analyzing (Analysis) REF: dm. 611 MSC: Client Needs: Health Promotion and Maintenance 36. A 40-year-old man has come into the clinic with complaints of extreme pain in his toes. The nurse notices that his toes are slightly swollen, reddened, and warm to the touch. His complaints would suggest: a . Osteoporosis. b . Acute gout. c . Ankylosing spondylitis. d . Degenerative joint disease. ANS: B Clinical findings for acute gout consist of redness, swelling, heat, and extreme pain like a continuous throbbing. Gout is a metabolic disorder of disturbed purine metabolism, associated with elevated serum uric acid. (See Table 22-1 for descriptions of the other terms.) DIF: Cognitive Level: Analyzing (Analysis) REF: dm. 627 MSC: Client Needs: Physiologic Integrity: Physiologic Adaptation 37. A young swimmer comes to the sports clinic complaining of a very sore shoulder. He was running at the pool, slipped on some wet concrete, and tried to catch himself with his outstretched hand. He landed on his outstretched hand and has not been able to move his shoulder since. The nurse suspects: a . Joint effusion. b . Tear of rotator cuff. c . Adhesive capsulitis. d . Dislocated shoulder. ANS: D A dislocated shoulder occurs with trauma involving abduction, extension, and external rotation (e.g., falling on an outstretched arm or diving into a pool). (See Table 22-2 for descriptions of the other conditions.) DIF: Cognitive Level: Analyzing (Analysis) REF: dm. 621 MSC: Client Needs: Physiologic Integrity: Physiologic Adaptation 38. A 68-year-old woman has come in for an assessment of her rheumatoid arthritis, and the nurse notices raised, firm, nontender nodules at the olecranon bursa and along the ulna. These nodules are most commonly diagnosed as: a . Epicondylitis. b . Gouty arthritis. c . Olecranon bursitis. d . Subcutaneous nodules. ANS: D Subcutaneous nodules are raised, firm, and nontender and occur with rheumatoid arthritis in the olecranon bursa and along the extensor surface of the ulna. (See Table 22-3 for a description of the other conditions.) DIF: Cognitive Level: Analyzing (Analysis) REF: dm. 623 MSC: Client Needs: Physiologic Integrity: Physiologic Adaptation 39. A woman who has had rheumatoid arthritis for years is starting to notice that her fingers are drifting to the side. The nurse knows that this condition is commonly referred to as: a Radial drift. . b . Ulnar deviation. c . Swan-neck deformity. d . Dupuytren contracture. ANS: B Fingers drift to the ulnar side because of stretching of the articular capsule and muscle imbalance caused by chronic rheumatoid arthritis. A radial drift is not observed. (See Table 22-4 for descriptions of swan-neck deformity and Dupuytren contracture.) DIF: Cognitive Level: Applying (Application) REF: dm. 624 MSC: Client Needs: Physiologic Integrity: Physiologic Adaptation 40. A patient who has had rheumatoid arthritis for years comes to the clinic to ask about changes in her fingers. The nurse will assess for signs of what problems? a Heberden nodes . b Bouchard nodules . c Swan-neck deformities . d Dupuytren contractures . ANS: C Changes in the fingers caused by chronic rheumatoid arthritis include swan-neck and boutonniere deformities. Heberden nodes and Bouchard nodules are associated with osteoarthritis. Dupuytren contractures of the digits occur because of chronic hyperplasia of the palmar fascia (see Table 22-4). DIF: Cognitive Level: Applying (Application) REF: dm. 624 MSC: Client Needs: Physiologic Integrity: Physiologic Adaptation 41. A patients annual physical examination reveals a lateral curvature of the thoracic and lumbar segments of his spine; however, this curvature disappears with forward bending. The nurse knows that this abnormality of the spine is called: a Structural scoliosis. . b . Functional scoliosis. c . Herniated nucleus pulposus. d . Dislocated hip. ANS: B Functional scoliosis is flexible and apparent with standing but disappears with forward bending. Structural scoliosis is fixed; the curvature shows both when standing and when bending forward. (See Table 22-7 for description of herniated nucleus pulposus.) These findings are not indicative of a dislocated hip. DIF: Cognitive Level: Analyzing (Analysis) REF: dm. 629 MSC: Client Needs: Physiologic Integrity: Physiologic Adaptation 42. A 14-year-old boy who has been diagnosed with Osgood-Schlatter disease reports painful swelling just below the knee for the past 5 months. Which response by the nurse is appropriate? a If these symptoms persist, you may need arthroscopic surgery. . b You are experiencing degeneration of your knee, which may not resolve. . c Your disease is due to repeated stress on the patellar tendon. It is usually self- . limited, and your symptoms should resolve with rest. d Increasing your activity and performing knee-strengthening exercises will help . decrease the inflammation and maintain mobility in the knee. ANS: C Osgood-Schlatter disease is a painful swelling of the tibial tubercle just below the knee and most likely due to repeated stress on the patellar tendon. It is usually self-limited, occurring during rapid growth and most often in boys. The symptoms resolve with rest. The other responses are not appropriate. DIF: Cognitive Level: Applying (Application) REF: dm. 626 MSC: Client Needs: Physiologic Integrity: Physiologic Adaptation 43. When assessing muscle strength, the nurse observes that a patient has complete range of motion against gravity with full resistance. What grade of muscle strength should the nurse record using a 0- to 5-point scale? a 2 . b 3 . c 4 . d 5 . ANS: D Complete range of motion against gravity is normal muscle strength and is recorded as grade 5 muscle strength. The other options are not correct. DIF: Cognitive Level: Applying (Application) REF: dm. 590 MSC: Client Needs: Physiologic Integrity: Physiologic Adaptation 44. The nurse is examining a 6-month-old infant and places the infants feet flat on the table and flexes his knees up. The nurse notes that the right knee is significantly lower than the left. Which of these statements is true of this finding? a This finding is a positive Allis sign and suggests hip dislocation. . b The infant probably has a dislocated patella on the right knee. . c This finding is a negative Allis sign and normal for an infant of this age. . d The infant should return to the clinic in 2 weeks to see if his condition has . changed. ANS: A Finding one knee significantly lower than the other is a positive Allis sign and suggests hip dislocation. Normally, the tops of the knees are at the same elevation. The other statements are not correct. DIF: Cognitive Level: Analyzing (Analysis) REF: dm. 611 MSC: Client Needs: Health Promotion and Maintenance 45. The nurse is assessing a 1-week-old infant and is testing his muscle strength. The nurse lifts the infant with hands under the axillae and notices that the infant starts to slip between the hands. The nurse should: a Suspect a fractured clavicle. . b Suspect that the infant may have a deformity of the spine. . c Suspect that the infant may have weakness of the shoulder muscles. . d Conclude that this is a normal finding because the musculature of an infant at . this age is undeveloped. ANS: C An infant who starts to slip between the nurses hands shows weakness of the shoulder muscles. An infant with normal muscle strength wedges securely between the nurses hands. The other responses are not correct. [Show More]

Last updated: 1 year ago

Preview 1 out of 416 pages

Reviews( 0 )

$18.00

Add to cart

Instant download

Can't find what you want? Try our AI powered Search

OR

GET ASSIGNMENT HELP
36
0

Document information


Connected school, study & course


About the document


Uploaded On

Jan 17, 2022

Number of pages

416

Written in

Seller


seller-icon
Nolan19

Member since 2 years

10 Documents Sold


Additional information

This document has been written for:

Uploaded

Jan 17, 2022

Downloads

 0

Views

 36

Document Keyword Tags

Recommended For You


$18.00
What is Browsegrades

In Browsegrades, a student can earn by offering help to other student. Students can help other students with materials by upploading their notes and earn money.

We are here to help

We're available through e-mail, Twitter, Facebook, and live chat.
 FAQ
 Questions? Leave a message!

Follow us on
 Twitter

Copyright © Browsegrades · High quality services·